UWorld Child Health

Ace your homework & exams now with Quizwiz!

The nurse has assessed 4 children. Which finding requires immediate follow-up with the health care provider? 1. Child who had a surgical repair of hypospadias earlier today with no urinary output in the past two hours 2. Child who is awaiting a neurological consult for suspected absence seizures and is sleeping soundly 3. Child who returned from a bronchoscopy an hour ago and coughed up a scant amount of blood-tinged sputum 4. Child with gastroenteritis, serum sodium of 131 mEq/L (131 mmol/L), and temperature of 100 F (37.8 C)

1 Hypospadias is a congenital defect in which the urethral opening is on the underside of the penis. Except in very mild cases, the condition is typically corrected around age 6-12 months by surgically redirecting the urethra to the penis tip. Circumcision is delayed so the foreskin can be used to reconstruct the urethra. If not corrected, clients may have toilet-training difficulties, more frequent urinary tract infections, and inability to achieve erections later in life. Postoperatively, the client will have a catheter or stent to maintain patency while the new meatus heals. Urinary output is an important indication of urethral patency. Fluids are encouraged, and the hourly output is documented. Absence of urinary output for over an hour indicates that a kink or obstruction may have occurred and requires immediate follow-up (Option 1). (Option 2) Absence seizures are brief moments of staring accompanied by a lapse in awareness. These seizures often go undetected and do not cause physical injury. (Option 3) Small amounts of blood-tinged sputum are normal after bronchoscopy; however, frank bleeding or clots should be reported. (Option 4) Electrolyte disturbances commonly occur with gastroenteritis due to prolonged or heavy vomiting and diarrhea. This client's sodium and temperature are slightly abnormal but not critical. Educational objective: Hypospadias, a condition in which the urethral opening is on the underside of the penis, is surgically corrected by rerouting the urethra and inserting a temporary stent to aid healing. Urinary output is closely monitored postoperatively to ensure patency.

A mother reports to the pediatric nurse that her 3-year-old child coughs at night and at times until he vomits. The symptoms have not improved over the past 2 months despite multiple over-the-counter cough medications. What should the nurse explore related to a possible etiology? 1. Ask about exposure to triggers such as pet dander 2. Assess for the presence of a butterfly rash 3. History of intolerance to wheat food products 4. Palpate for an abdominal mass from pyloric stenosis

1 Asthma is a chronic inflammatory disease of the lungs in genetically susceptible children. Frequent cough, especially at night, is the warning signal that the child's airway is very sensitive to stimuli; it may be the only sign in "silent" asthma. Common triggers include indoor contaminants (eg, tobacco smoke, pet dander, cockroach feces), outdoor contaminants (eg, air pollution), and allergic disease (eg, hay fever, food allergies). (Option 2) A red or pink butterfly rash across the cheeks and bridge of the nose is classic for systemic lupus erythematosus (SLE), an autoimmune disease that affects connective tissue. The child has no symptoms of SLE. Manifestations are acute (eg, nephritis, arthritis, vasculitis) or involve a gradual onset of nonspecific symptoms. (Option 3) Celiac, or gluten-sensitive, enteropathy is a chronic malabsorption syndrome. There is intolerance for gluten, a protein found in wheat, barley, rye, and oats. This condition affects absorption of nutrients; it does not cause nausea. (Option 4) Pyloric stenosis is a hypertrophy of the pylorus that results in stenosis of the passage between the stomach and the duodenum. Symptoms become evident 2-8 weeks after birth. It starts with occasional vomiting that eventually becomes forceful/projectile vomiting as the obstruction becomes complete. Dehydration and electrolyte imbalance result. The thickened pyloric muscle can sometimes be palpated and can be confirmed with ultrasound. This child is too old for this complication. Educational objective: Pediatric asthma can present as night coughing until the child vomits.

The clinic nurse supervises a graduate nurse who is teaching the parents of a 2-year-old with acute diarrhea about home management. The nurse would need to intervene when the graduate nurse provides which instruction? 1. "Do not administer antidiarrheal medications to your child." 2. "Follow the bananas, rice, applesauce, and toast diet for the next few days." 3. "Record the number of wet diapers and return to the clinic if you notice a decrease." 4. "Use a skin barrier cream such as zinc oxide in the diaper area until diarrhea subsides."

2 During bouts of acute diarrhea and dehydration, treatment focuses on maintaining adequate fluid and electrolyte balance. The first-line treatment is oral rehydration therapy, using oral rehydration solutions (ORSs) to increase reabsorption of water and sodium. Even if the diarrhea is accompanied by vomiting, ORS should still be offered in small amounts at frequent intervals. Continuing the child's normal diet (solid foods) is encouraged as it shortens the duration and severity of the diarrhea. The BRAT (bananas, rice, applesauce, and toast) diet is not recommended as it does not provide sufficient protein or energy. (Option 1) Use of antidiarrheal medications is discouraged as these have little effect in controlling diarrhea and may actually be harmful by prolonging some bacterial infections and causing fatal paralytic ileus in children. (Option 3) Parents should be taught to monitor their child for signs of dehydration by checking the amount of fluid intake, number of wet diapers, presence of sunken eyes, and the condition of the mucous membranes. (Option 4) Protecting the perineal skin from breakdown during bouts of diarrhea can be accomplished by using skin barrier creams (eg, petrolatum or zinc oxide). Educational objective: When a child is experiencing acute diarrhea, the priority is to monitor for dehydration. Treatment is accomplished with oral rehydration solutions and early reintroduction of the child's normal diet (usual foods).

The nurse is caring for a 4-year-old who was hospitalized with influenza. Which nursing action would be most effective to maintain psychosocial integrity? 1. Encouraging use of puzzles for play 2. Offering the child stacking blocks for diversion 3. Providing crayons to draw noses on facemasks 4. Suggesting that playmates visit the child

3 Clients with influenza are maintained on droplet precautions, and anyone entering the room must wear a facemask. Medical play during the preschool period (age 3-5 years) facilitates psychosocial integrity. Crayons are age-appropriate toys. Drawing noses on facemasks will help the child feel more comfortable with procedures and provides a developmentally appropriate diversion. (Option 1) Puzzles would be more appropriate for the school-age child (6-12 years). (Option 2) Stacking blocks would be more appropriate for the toddler (age 1-3 years). (Option 4) Maintaining contact with peers would be more appropriate for the adolescent (age 12-19 years). Educational objective: Maintaining psychosocial integrity is a priority when a child is hospitalized. This is achieved by integrating age-appropriate diversions (eg, medical play for a preschooler) in the nursing care.

The nurse taught the caregiver of a child with a ventriculoperitoneal (VP) shunt about when to contact the health care provider (HCP). The caregiver shows understanding of the instructions by contacting the HCP about which symptom? 1. A temperature of 99 F (37 C) that occurs during the evening 2. The child cannot recall items eaten for lunch the previous day 3. The child vomits after awakening from a nap and 1 hour later 4. The VP shunt is palpated along the posterior-lateral portion of the skull

3 The caregiver of a child with a VP shunt must understand symptoms of increased intracranial pressure (ICP), which indicate shunt malfunction. Vomiting may be a sign of increased ICP and would require that the HCP be contacted. (Option 1) Fever may indicate shunt infection, but a temperature of 99 F (37 C) remains within acceptable parameters. Contacting the HCP is not indicated. (Option 2) Memory lapse or changes in mental status may indicate increased ICP. The inability to remember one meal would not indicate a change of mental status. (Option 4) A VP shunt is tunneled under the scalp and can be palpated. Educational objective: Increased ICP may occur with VP shunt malfunctions. The caregiver must recognize symptoms of vomiting, headaches, vision changes, and changes in mental status. Early intervention by the HCP will decrease the risk of damage to the brain tissue.

A child is scheduled to have an electroencephalogram (EEG). Which statement by the parent indicates understanding of the teaching? 1. "I will let my child drink cocoa as usual the morning of the procedure." 2. "I will wash my child's hair using shampoo the morning of the procedure." 3. "My child may have scalp tenderness where the electrodes were applied." 4. "My child will not remember the procedure."

2 An electroencephalogram (EEG) is a diagnostic procedure used to evaluate the presence of abnormal electrical discharges in the brain, which may result in a seizure disorder. The EEG can be done in a variety of ways, such as with the child asleep or awake with or without stimulation. Teaching for the parent includes the following: Hair should be washed to remove oils and hair care products, and accessories such as ribbons or barrettes should be removed. Hair may need to be washed after the procedure to remove electrode gel. Avoid caffeine, stimulants, and central nervous system depressants prior to the test. The test is not painful, and no analgesia is required. (Option 1) Food and liquids are not restricted prior to an EEG except for caffeinated beverages. Cocoa contains caffeine. (Option 3) This test (EEG) is not painful as it only records brain electrical activity. Electrode gel is nonirritating to the skin. (Option 4) A routine EEG is not performed under sedation, and so the child should remember the procedure. Educational objective: An EEG is used to diagnose the presence of a seizure disorder. Electrodes are secured to the scalp to observe for abnormal electrical discharges in the brain. Preprocedure teaching includes avoiding stimulants and CNS depressants and washing the hair.

The parent of an 8-year-old client asks the nurse for guidance on how to help the client cope with the recent death of the other parent. When developing a response to the parent, the nurse considers that a school-aged child is most likely to do what? 1. React anxiously to altered daily routines 2. Realize that death eventually affects everyone 3. Think about the religious or spiritual aspects of death 4. Understand that death is permanent but be curious about it

4 Understanding a child's perception of illness and death can empower caregivers (eg, parents) to support the child during the loss of a loved one. A child's developmental stage as well as the caregiver's view of death and relationship with the child will influence coping during bereavement. The nurse should educate the parent of an 8-year-old client about how to assist with coping based on the knowledge that school-aged children (age 6-12 years) most likely have both a curiosity and fear about the implications of death (eg, process of dying, funeral services) and understand that death is permanent (Option 4). Therefore, it is important for the parents to be honest during discussions about death, talk about the lost loved one, and provide anticipatory guidance to reduce fears. (Option 1) Infants (age 1-12 months) and toddlers (age 12-36 months) mostly react to separation from caregivers, both temporary and permanent, because it affects daily routines. (Option 2) A child will most likely be aware that death affects everyone and also perceive it as evil by age 10-12 years. (Option 3) Adolescents are most likely to think about the religious and spiritual aspects of death, although this may occur earlier for some children. Educational objective: The nurse should understand how children perceive illness and death within each age group to provide guidance for caregivers. School-aged clients most likely understand the concrete finality of death and are curious and fearful about its implications (eg, process of dying, funeral services).

The clinic nurse reviews teaching provided to the parent of a child being considered for growth hormone replacement therapy at home. Which statement by the parent indicates that teaching has been effective? 1. "Treatment will be considered a success when my child grows at a rate equal to peers." 2. "Treatment will be required throughout my child's life." 3. "Treatment will begin when my child becomes an adolescent." 4. "Treatment will require a daily injection under my child's skin."

4 A child who demonstrates a slow growth pattern will undergo diagnostic evaluation to determine the cause. If the cause is found to be growth hormone deficiency, the child may undergo growth hormone replacement therapy. The biosynthetic hormone is administered via subcutaneous injection on a daily basis. Despite replacement therapy, the child may still have a final height less than "normal." Treatment is most successful when diagnosis and replacement therapy begin early in the child's life. When to stop therapy is decided by the client, family, and provider. However, growth less than 1 inch (2.5 cm) per year and bone age of 14 years in girls and 16 years in boys are the criteria often used to stop therapy. (Option 1) Growth hormone replacement does not guarantee that a child will grow at a rate equal to peers. Treated children often remain shorter than their peers. (Option 2) Replacement therapy is not continued throughout a child's life. It is stopped when bone growth begins to cease or when the child, parents, and provider make the decision. (Option 3) Replacement therapy is most successful when treatment begins early, as soon as growth delays are noted. Educational objective: Growth hormone replacement is an option for children who are not growing according to accepted standards. The treatment should begin as soon as delays are noted and continue until bone growth begins to cease despite replacement therapy. Replacement is administered via subcutaneous injections. Physiological Adaptation

A 3-month-old who weighs 8.8 lb (4 kg) has just returned to the intensive care unit after surgical repair of a congenital heart defect. Which finding by the nurse should be reported immediately to the health care provider? 1. Chest tube output of 50 mL in the past hour 2. Heart rate of 150/min 3. Temperature of 97.5 F (36.4 C) 4. Urine output of 8 mL in the past hour

1 Chest tubes may be placed during cardiac surgery to help drain fluid and air and to ensure room for lung expansion. The chest tube and chamber should be assessed every hour for color and quantity of drainage. Drainage >3 mL/kg/hr for 3 consecutive hours or >5-10 mL/kg in 1 hour should be reported immediately to the health care provider (Option 1). This could indicate postoperative hemorrhage and requires immediate intervention. Cardiac tamponade can develop rapidly in children and can be life-threatening. This child weighs 4 kg and an output of 50 mL in 1 hour is excessive. (Option 2) For infants age 1-12 months, the normal heart rate is 90-160/min. (Option 3) Hypothermia is common after surgery. Warmers may be used to correct the client's temperature. (Option 4) Hourly urine output should be measured in the postoperative infant. A urinary catheter is often placed during surgery, allowing for accurate measurement. Urine output should be 1-2 mL/kg/hr. Educational objective: The nurse should immediately report chest tube drainage >3 mL/kg/hr over 3 consecutive hours or >5-10 mL/kg over 1 hour, which could indicate postoperative hemorrhage. Cardiac tamponade can occur rapidly in children and can be life-threatening.

A nurse is teaching the parent of an 8-month-old infant who had a febrile seizure about management of future fevers. Which instruction is appropriate to include in the teaching? 1. "Give acetaminophen or ibuprofen every 6 hours to control the fever." 2. "Give the infant frequent tepid sponge baths to control the fever." 3. "If the infant develops another seizure, wait 15 minutes to see if it subsides." 4. "Place ice bags under the arms and around the neck to reduce the fever."

1 Febrile seizures are an alarming experience for parents. They most commonly occur in children between ages 6 months to 6 years, with the peak of incidence occurring at age 18 months. The etiology is unknown. Simple febrile seizure management typically involves reassurance regarding the benign nature of most febrile seizures, and education about the risk of recurrence and seizure safety precautions (eg, side-lying positioning, removal from harmful environments). Parents should use antipyretics such as acetaminophen or ibuprofen (in children age >6 months) to control fever and make the child more comfortable (Option 1). However, there is no evidence that antipyretics reduce the risk of future febrile seizures. After the administration of antipyretics, additional cooling methods that may be beneficial for reducing fever include applying cool, damp compresses to the forehead; increasing air circulation in the room; and wearing loose or minimal clothing. (Options 2 and 4) Bathing an infant in tepid water and placing ice bags under the arms and around the neck are not recommended techniques as these induce shivering, increase metabolic activity, have no antiseizure effects, and cause discomfort for the child. (Option 3) Parents should be instructed to call 911 and seek medical assistance for a seizure lasting more than 5 minutes. Neurologic damage can occur with frequent and prolonged seizures. Educational objective: Febrile seizures, although alarming, are generally benign. Parents should be instructed on appropriate cooling methods (eg, antipyretics, cool compresses), seizure safety precautions, and the avoidance of shivering.

What play behavior would the nurse be most likely to observe in a group of 4-year-old children? 1. Children playing and borrowing blocks from each other without directing others 2. Children playing and working together to build a castle out of blocks 3. Children playing next to each other with blocks, but not interacting 4. Children playing with blocks by themselves in separate areas of the room

1 Play is an important developmental task of childhood and is an indication of physical, social, and emotional health. Preschoolers (age 3-6) enjoy associative play, in which they engage in similar activities or play with the same or similar items, but the play is unorganized without specific goals or rules. They often borrow items from each other without directing each other's play. Preschoolers also enjoy play involving motor activities and imaginative, pretend play. (Option 2) Cooperative play is common in school-age children (age 6-12). These children play with one another with a specific goal (eg, building a castle from blocks), often within a rigid set of rules. Cooperative play is likely too advanced for preschool-age children, as it involves more organizational skills. (Option 3) Parallel play is more common in toddlers (age 1-3). During parallel play, these children play next to each other and are happy to be in the presence of peers, but they do not play directly with one another. (Option 4) Solitary play is common in infants (birth to 1 year). Children at this stage are focused on their own activity and will play alone in the presence of others. Educational objective: Preschoolers (age 3-6) enjoy associative play, in which children engage in the same type of activity, but the play is unorganized without specific goals or rules. Infants play alone (solitary), toddlers play next to each other (parallel), and school-age children play with others with a common goal (cooperative). Health Promotion and Maintenance

The nurse is caring for a hospitalized 6-month-old client. Which of the following interventions should the nurse implement to provide developmentally appropriate care for this client? Select all that apply. 1. Adhere to the child's home routine when possible during hospitalization 2. Encourage parents to bring the child's favorite toy from home 3. Have the parents step out of the room during procedures 4. Promote a quiet sleep environment with reduced stimuli 5. Provide a parent's shirt for the child to hold during procedures

1, 2, 4, 5 Around 6 months of age, infants begin to experience separation anxiety. This anxiety may be heightened during hospitalization because of exposure to many unfamiliar stressors. Appropriate nursing care can play a significant role in reducing the infant's physiologic and psychologic stress. Key interventions include: - Adhering to the infant's home routine (eg, meal and sleep times) as closely as possible (Option 1) - Providing a favorite toy or pacifier (Option 2) - Encouraging caregivers to remain whenever possible during hospitalization - Providing a quiet sleep environment with reduced stimulation to promote restful sleep (Option 4) - Offering a familiar object (eg, caregiver's shirt, blanket, voice recording) during stressful situations (Option 5) (Option 3) The presence of parents or the primary caregiver during hospitalization reduces separation anxiety and decreases the infant's stress response. Therefore, caregivers should remain whenever possible throughout all the client's care (eg, procedures, medication administration, scans). Educational objective: To reduce separation anxiety in infants during hospitalization, the nurse should adhere to the infant's home routine, provide a favorite toy or pacifier, provide a quiet sleep environment, encourage the presence of the primary caregivers, and expose the infant to familiar objects during stressful situations. Health Promotion and Maintenance

The nurse plans care for a pediatric client who has just undergone a cleft palate repair. Which of the following interventions should the nurse include in the plan of care? Select all that apply. 1. Assist and encourage caregivers to hold and comfort the child 2. Offer a pacifier in between feedings to promote the child's comfort 3. Position the child supine with an elevated head of bed after feedings 4. Remove elbow restraints per policy for skin and circulatory assessment 5. Use tongue blade and penlight to assess surgical site every 4 hours

1, 3, 4 A cleft palate is a malformation of the roof (palate) of the mouth occurring from incomplete fusion of the palatine bones and maxilla during fetal development. Cleft palate causes an opening (cleft) in the mouth into the nasal cavity, which leads to difficulty in sucking and feeding. Clients with cleft palate typically undergo surgical repair between age 6-24 months. Postoperative nursing interventions for clients with a cleft palate repair include: - Implementing pharmacological and nonpharmacological pain management (eg, encouraging caregiver soothing), as uncontrolled pain leads to crying, which stresses the surgical site and promotes hemorrhage (Option 1) - Positioning the child in an upright, supine position, particularly after feedings, to prevent airway compromise and obstruction from secretions and/or feedings (Option 3) - Utilizing elbow restraints to prevent the child from disrupting the surgical site by placing hands or objects into the mouth, and monitoring skin and neurovascular status by removing elbow restraints per agency policy (Option 4) (Options 2 and 5) Hard objects (eg, utensils, tongue depressors, pacifiers, straws) should not be placed into the mouth as they may damage the surgical site, which can lead to hemorrhage. Educational objective: Following cleft palate repair, the nurse should position the client supine with elevated head of bed, implement pain reduction measures, and remove elbow restraints per facility policy for skin and neurovascular assessment. To prevent damage to the surgical site, hard objects should not be placed into the mouth. Physiological Adaptation

The nurse assists with a staff education conference about appropriate nonpharmacological pain-management interventions for newborns and infants. Which of the following strategies should be included in the presentation? Select all that apply. 1. Administer an oral sucrose solution to a newborn during a circumcision procedure 2. Apply a cold pack to a newborn's heel 30 minutes before performing a heel stick 3. Assist the parent to hold a newborn skin-to-skin during an immunization injection 4. Offer a pacifier to an infant while performing venipuncture 5. Swaddle an infant while leaving one arm unwrapped during an IV dressing change

1, 3, 4, 5 Painful procedures (eg, capillary heel sticks, immunizations) are frequently required to provide optimal care but may cause considerable stress or alterations in a client's status (eg, vital sign changes, instability) without proper management. Nonpharmacological pain management is a method for stopping or reducing the sensation of pain and may eliminate or decrease the need for pharmacological intervention. Appropriate nonpharmacological pain-management interventions for infants and newborns include: - Offering concentrated sucrose, if prescribed, which is associated with reduced indicators of pain (eg, presence and duration of crying, grimacing) (Option 1) - Assisting the parent to hold the infant skin-to-skin (kangaroo care), which provides sensory stimulation that is calming and reduces indicators of pain (Option 3) - Offering nonnutritive sucking interventions (eg, pacifiers), which help calm the infant during painful procedures (Option 4) - Swaddling the infant, which provides a sense of comfort and security and reduces the heart rate and incidences of crying (Option 5) (Option 2) Before a heel stick is performed, a warm (not cold) pack should be applied to help facilitate blood flow to the area. Although a cold pack is a nonpharmacological pain-management technique, it causes vasoconstriction and impedes blood flow, which may result in a need to apply pressure to obtain blood or to perform multiple heel sticks. Educational objective: Nurses performing painful procedures (eg, capillary heel sticks) should implement pain-management techniques to promote the client's comfort and stability. Appropriate nonpharmacological interventions for newborns and infants include concentrated sucrose solutions, skin-to-skin contact, nonnutritive sucking (eg, pacifiers), and swaddling. Basic Care and Comfort

The nurse is educating a group of parents about ways to decrease the risk of sudden infant death syndrome. Which of the following recommendations should the nurse suggest? Select all that apply. 1. Breastfeed the infant 2. Cosleep with the infant in the parent's bed 3. Ensure the infant's vaccinations are updated 4. Maintain a smoke-free environment 5. Place the infant to sleep in a side-lying position 6. Provide a firm sleep surface for the infant

1, 3, 4, 6 Sudden infant death syndrome (SIDS) is the unexpected, unexplained death of an infant age <1 year, occurring most frequently in those age <6 months during sleep/naps. The nurse should recommend that parents place their infant to sleep on the back in a safe place (eg, crib). The sleep surface (eg, mattress) should be firm with no loose or soft items (eg, blankets, toys, stuffed animals) to prevent suffocation (Option 6). Environmental factors such as smoking may also increase the infant's risk for SIDS; therefore, parents should maintain a smoke-free environment (Option 4). In addition, breastfeeding and updated vaccinations help to keep infants healthy and are protective against SIDS (Options 1 and 3). (Option 2) Parents should avoid cosleeping with their infant (ie, bed sharing) because it increases the infant's risk for suffocation and falls. Encouraging room sharing without bed sharing is appropriate, however. (Option 5) Due to the infant's body shape (ie, barrel chest; flat, uncurved spine), side-lying positions facilitate rolling over to a prone position. Instead, the nurse may recommend supervised time during the day for the infant to lay on the stomach while awake (ie, tummy time) to promote muscle development and prevent positional plagiocephaly. Educational objective: To reduce the incidence of sudden infant death syndrome (ie, the unexpected, unexplained death of an infant age <1 year), the nurse should teach parents to avoid cosleeping and prone/side-lying sleeping positions and encourage safe sleep practices (eg, firm sleeping surface); breastfeeding; a smoke-free environment; and updated vaccinations. Health Promotion and Maintenance

The parent of a 21-day-old male infant reports that the infant is "throwing up a lot." Which assessments should the nurse make to help determine if pyloric stenosis is an issue? Select all that apply. 1. Assess the parent's feeding technique 2. Check for family history of gluten enteropathy 3. Check for history of physiological hyperbilirubinemia 4. Check if the vomiting is projectile 5. Compare current weight to birth weight

1, 4, 5 In pyloric stenosis, there is gradual hypertrophy of the pylorus until symptom onset at age 3-5 weeks. It is common in first-born boys and the etiology is unclear. Pyloric stenosis presents with postprandial projectile vomiting (ejected up to 3 feet) followed by hunger (eg, "hungry vomiter"). This is clearly distinguished from the "wet burps" infants have due to a weak lower esophageal sphincter. The emesis is nonbilious as the obstruction is proximal to the bile duct. Infants have poor weight gain and are often dehydrated (eg, sunken fontanelle, decreased skin turgor, delayed capillary refill). The amount of milk consumed (particularly with bottle feedings) along with the mother's technique (mainly adequate burping) should be assessed to ensure there is no excessive air swallowing or overfeeding as an etiology. (Option 2) At times, formula intolerance or allergy is suspected initially when the infant first starts vomiting. However, celiac disease or gluten enteropathy is related to intolerance to gluten, a protein in barley, rye, oats, and wheat (BROW). Clients with celiac disease cannot eat these foods. A 3-week-old infant would only consume milk; this history would not be a factor at this time. (Option 3) Physiological hyperbilirubinemia occurs due to the newborn's immature liver that is unable to metabolize hemoglobin byproducts. This is a "normal" finding that is unrelated to pyloric stenosis. Educational objective: Pyloric stenosis is a hypertrophy of the pyloric sphincter that eventually causes complete obstruction. Classic signs include projectile nonbilious vomiting, an olive-shaped right upper quadrant mass, weight loss, dehydration, and/or electrolyte imbalance (metabolic alkalosis). Physiological Adaptation

A nurse is talking with the parent of a 6-year-old regarding sleep and rest. Which information should be included? 1. Active play before bedtime promotes restful sleep 2. Bedtime hours should be established 3. Rest needs are related to the high rate of growth in this age group 4. Seven to 8 hours of sleep are required

2 During the school-age years (6-12), sleep needs of a child depend on health status, activity level, and age. Children in this age group need approximately 11 hours of sleep daily at age 5 and 9 hours at age 12 (Option 4). Children are often unaware of their level of fatigue. Bedtimes should be established to prevent fatigue the next day. Bedtime issues are usually not a concern, although many children retain bedtime rituals such as reading or listening to music. (Option 1) Quiet activity (eg, coloring, reading) prior to bedtime should be planned to promote restful sleep. (Option 3) Growth rate is slowed during the school-age years, which accounts for variations in sleep needs. Educational objective: Sleep needs of school-age children are dependent on health status, activity level, and age. Required sleep averages 11 hours (for 5-year-olds) to 9 hours (for 12-year-olds). It is important to establish bedtime hours and bedtime rituals. These children usually do not need daytime naps if they have slept well at night. Health Promotion and Maintenance

The nurse is caring for a 7-year-old client diagnosed with nephrotic syndrome who will be discharged soon. Which statement by the parent indicates the need for further teaching? 1. "I'll provide a healthy diet without added salt for my child." 2. "I'll organize playdates to keep my child's spirits up during relapses." 3. "I'll restrict my child's fluids if I notice swelling or rapid weight gain." 4. "I'll test for protein in my child's urine every day."

2 Nephrotic syndrome, an autoimmune disease, affects children age 2-7 and is characterized by increased permeability of the glomerulus to proteins (eg, albumin, immunoglobulins, natural anticoagulants). Loss of albumin in urine leads to hypoalbuminemia; this causes decreased plasma oncotic pressure, which allows fluid to leak out of the vascular spaces. Reduced plasma volume (hypovolemia) activates kidneys to retain salt and water (via the renin-angiotensin-aldosterone system). Clients experience generalized edema, weight gain, loss of appetite (from ascites), and decreased urine output. The loss of immunoglobulins causes increased susceptibility to infection. Caregivers should minimize the risk of infection during relapses (eg, limiting visitors) (Option 2). Treatment typically includes: - Corticosteroids and other immunosuppressants (eg, cyclosporine) - Loss of appetite management (eg, making foods fun and attractive) - Infection prevention (eg, limiting social interaction until the child is in remission) (Option 1) A regular diet without added salt is prescribed to prevent edema while in remission. More stringent sodium restrictions are necessary when symptoms are present. (Option 3) Fluid restriction is needed in cases of edema or rapid weight gain. (Option 4) There is a high risk for recurrence after recovery, and relapses may occur several times per year. The parent/caregiver should test daily for proteinuria, weigh the child weekly, and keep a diary of results. Educational objective: Nephrotic syndrome is characterized by massive proteinuria, edema, and hypoalbuminemia. Home management includes a low-sodium diet with attractive foods; infection prevention; fluid restriction for severe edema; and monitoring of weight gain and proteinuria to detect relapse. Reduction of Risk Potential

The nurse is performing a physical examination on a 10-year-old client with abdominal discomfort. Which actions would be appropriate during the examination? Select all that apply. 1. Ask the accompanying parent to rate and describe the client's pain 2. Ask the client to describe the chief symptom 3. Conduct a head-to-toe assessment in the same manner as an adult assessment 4. Explain the outcome of the examination to the parent without the child present 5. Honor the client's request to be examined without a parent present

2, 3, 5 Assessment based on the client's developmental age includes the following: - Clients as young as 3 can tell and/or show the examiner where they hurt or how they feel in their own terms - 10-year-olds are capable of understanding and assisting in their physical examination. In fact, school-age clients are usually quite interested in equipment and how it works. - 10-year-olds are becoming modest and do not want a parent, especially of the opposite sex, in the room with them during a physical examination (Option 1) Pain is the fifth vital sign and is subjective data. A 10-year-old can describe and rate pain accurately. (Option 4) 10-year-olds will think there is something seriously wrong with them if the nurse and parent will not explain the results of the examination to them. Educational objective: When performing a physical assessment on a child, it is imperative that the examiner understand the developmental age so that the child will be more comfortable and cooperative during the examination. Health Promotion and Maintenance

The nurse is admitting an infant who has severe growth deficiency and facial characteristics of indistinct philtrum, a thin upper lip, and short palpebral fissures. Which question should the nurse ask to assess the cause of these clinical findings? 1. "Is the mother of advanced age?" 2. "Is there a history of cigarette use during pregnancy?" 3. "Is there a history of exposure to alcohol in utero?" 4. "Is there a maternal history of valproate use?"

3 Fetal alcohol syndrome (FAS) is a leading cause of intellectual disability and developmental delay in the United States. Diagnosis includes history of prenatal exposure to any amount of alcohol, growth deficiency, neurological symptoms (eg, microcephaly), or specific facial characteristics (indistinct philtrum, thin upper lip, epicanthal folds, flat midface, and short palpebral fissures). Asking about alcohol use during pregnancy can identify newborns and infants who are at risk for FAS. Family support, early intervention, and prevention for subsequent pregnancies are important for families with an infant with this diagnosis. (Option 1) Advanced maternal age has been associated with a higher incidence of trisomy 21 (Down syndrome). Characteristic features include a single palmar crease and a short neck with excess skin (nuchal fold). (Option 2) Cigarette smoking is linked to perinatal loss, sudden infant death syndrome, low birth weight, and prematurity. Specific facial characteristics or syndromes are not typically caused by tobacco exposure in utero. (Option 4) Valproate (Depakote), a medication used to control seizures, is an FDA pregnancy category D drug that can cause neural tube defects such as spina bifida, but not the distinct facial features of FAS. Educational objective: Consumption of any amount of alcohol during pregnancy places a fetus at risk for fetal alcohol syndrome. Clinical manifestations include intellectual disability, developmental delay, and distinct facial characteristics (indistinct philtrum, thin upper lip, epicanthal folds, flat midface, and short palpebral fissures).

The nurse is assessing an 8-month-old client during a well-child visit. Which assessment finding should the nurse report to the health care provider? 1. Infant responds to their name when called but has not spoken any words 2. Infant was gaining 5 oz (140 g) per week at age 6 months and is now gaining 3 oz (84 g) per week 3. Infant's head stays behind the shoulders when raised from a supine to a sitting position 4. Infant's posterior fontanel is not palpable when performing assessment of the head

3 Infants and children undergo rapid physical, sensory, and cognitive development in relatively uniform patterns based on age. Failure to achieve expected milestones may be an early indicator of a chronic medical condition. Head and neck strength are critical assessments in an infant. In general, infants display head lag (ie, head falling behind the shoulders when raised from a supine to a seated position) from birth through age 4-6 months. Head lag remaining after age 6 months is an abnormal finding often associated with cerebral palsy or autism (Option 3). (Option 1) An infant age 6 to 7 months may start responding to their name by turning toward the sound when spoken. An infant may not speak first words with meaning (eg, mamma, dada) until approximately age 9-11 months. (Option 2) During the first 6 months, infants should grow 5-7 oz (140-196 g) per week. From age 6-12 months, infants should grow 3-5 oz (84-140 g) per week. (Option 4) The posterior fontanel is a membrane-filled space between the parietal and occipital bones that normally closes by age 2 months. Therefore, the posterior fontanel should not be palpable in an infant age 8 months. Educational objective: After age 6 months, infants should not exhibit head lag when raised from a supine to a seated position. An infant may not speak first words with meaning (eg, mamma, dada) until approximately age 9-11 months. Infant weight gain decreases after age 6 months, from 5-7 oz (140-196 g) to 3-5 oz (84-140 g) per week. Health Promotion and Maintenance

A nurse is discussing parallel play with the parent of a 2-year-old. Which statement by the parent indicates understanding of the discussion? 1. "I encourage working in a group to build towers with large blocks." 2. "I have a chalk board available to teach the alphabet and numbers." 3. "I set out a basket of various balls in the backyard when other children come to play." 4. "I try to organize games that involve a team approach."

3 Parallel play is typical behavior of a toddler and involves activities focused on improving motor skills, imitative efforts, and the use of multiple senses. Toddlers play alongside, rather than with, other children. Having a variety of different balls for a group of children allows each child to be present with others and participate as they desire. Other examples of parallel play activities include pushing and pulling large toys; smearing paint; playing with dolls or toy cars; and digging in a sandbox. (Option 1) Working in groups is an appropriate play activity for children in the preschooler period. (Option 2) The classroom approach does not promote parallel play. Using large chalk to draw allows the child creative expression in an unstructured manner. (Option 4) A toddler is challenged by the concept of team games, which requires attention to the group's effort. Educational objective: Toddlers engage in parallel play, which involves playing alongside, not with, other children. Activities such as playing with dolls or toy cars, pushing and pulling large toys, smearing paint, and digging in a sandbox encourage parallel play. Health Promotion and Maintenance

A 5-year-old child is receiving morphine sulfate for pain. Which statement by the caregiver indicates that further teaching is necessary? 1. "I will call the nurse if my child begins to act aggressively." 2. "I'm concerned that my child thinks the pain is punishment." 3. "My child is playing and so does not need pain medication." 4. "The FACES pain scale seems to be working very well."

3 The child who is playing or sleeping might still be experiencing pain but is using distraction as a coping mechanism. This statement by the caregiver indicates that further teaching is needed. (Option 1) Preschool-age children may become physically or verbally aggressive when in pain. (Option 2) The preschool-age child experiences magical thinking and might feel that pain is a punishment for wrongdoing. (Option 4) Age-appropriate pain scales can be used to assess pain in children. The FACES pain rating scale consists of 6 cartoon faces with expressions from no pain to worst pain. Educational objective: A child's expression of pain varies based on developmental stage and past experiences with pain. The nurse should use age-appropriate pain scales. A child who is asleep or playing may be experiencing pain. Pharmacological and Parenteral Therapies

The nurse is teaching a group of new parents about oral hygiene for their children. One of the parents asks, "When should I take my child to the dentist?" What would be the best response from the nurse? 1. "It is recommended that your child's first dental visit be after age 1." 2. "The first visit should be when all of your child's baby teeth have come in." 3. "The initial dentist visit should be soon after the child's first tooth appears." 4. "Your child will need to be taken to the dentist before starting preschool."

3 The recommendation from pediatric professional organizations, including the American Dental Association, American Academy of Pediatrics, and Canadian Dental Association, is that children have their first dental visit within 6 months of first tooth eruption or by their first birthday (Option 3). A child's first tooth usually erupts around age 6 months, and the child should be seen by a dentist soon after. The purposes of the first visit include: - Assessing risk for dental disease - Providing dental care and treatment of dental caries - Providing anticipatory guidance about dental hygiene, fluoride, diet and dietary habits, and non-nutritive sucking - Establishing care with a licensed dentist and scheduling future visits Taking a child to the dentist at an early age also helps the child become accustomed to the dentist's office, oral examinations, and dental care. (Option 1) Children should have their first dental visit by their first birthday. (Option 2) Most children have all of their primary teeth by age 2 or 3. Dental care is needed at a much earlier age. (Option 4) Most children start preschool around age 4. A routine of regular dental visits and care needs to be established at an earlier age. Educational objective: Regular visits to the dentist, starting within 6 months of first tooth eruption or by the first birthday, are recommended by pediatric professional organizations. Early assessment, dental care, anticipatory guidance, and education about periodontal disease prevention facilitate good oral health for children. Health Promotion and Maintenance

A nurse receives report on a group of clients. Which client should the nurse assess first? 1. A preschool-age child with a harsh cough, expiratory wheezes, and mild intercostal retractions 2. A toddler playing with small toys who appears in distress, has circumoral cyanosis, and cannot speak 3. A toddler with a barking cough, infrequent inspiratory stridor, and oxygen saturation of 94% on room air 4. An infant with an axillary temperature of 100.1 F (37.8 C) who is tugging at the left ear

2 Aspiration of a foreign body occurs most often in the toddler age group. Swallowing of objects such as buttons, small parts of toys, or food particles can be life-threatening and result in airway obstruction due to the small diameter of the airway. Manifestations include choking, gagging, cyanosis, and inability to speak when the object is lodged in the larynx. (Option 1) Although the client has mild retractions with wheezing and a harsh cough, a patent airway is present. This client may be experiencing expected manifestations of asthma, but this is not a life-threatening condition. (Option 3) The client's manifestations are consistent with laryngotracheobronchitis (croup), which is generally caused by a parainfluenza virus. There is no respiratory challenge indicated by a 94% oxygen saturation on room air, and this not an emergency situation. (Option 4) Otitis media is an infection or inflammation of the middle ear with the highest incidence at age 6-36 months; it occurs during the winter months. Acute onset presents with ear pain, irritability, fever, and pulling on the affected ear. Fluid can accumulate in the middle ear and create an environment for bacterial growth. Respiratory distress is not seen. Educational objective: Using the priorities of airway, breathing, and circulation, maintenance of airway function requires immediate intervention by a nurse.

A nurse auscultates a loud cardiac murmur on a newborn with suspected trisomy 21 (Down syndrome). A genetic screen and an echocardiogram are scheduled that day. The neonate's vital signs are shown in the exhibit. What would be an appropriate action for the nurse to complete next? Click on the exhibit button for additional information. Vital signs = Temperature 98.6 F (37 C); Heart rate 146/min Respirations 42/min; O2 saturation 98% 1. Call the health care provider (HCP) immediately 2. Document the assessment finding 3. Place the neonate in a knee-chest position 4. Provide oxygen to the neonate

2 Atrioventricular (AV) canal defect is a cardiac anomaly often associated with trisomy 21 (Down syndrome). As an echocardiogram is already scheduled for that day, documenting the assessment finding would be the appropriate action for the nurse to complete at this time. (Option 1) The neonate has stable vital signs and the echocardiogram will be completed. This is not an emergency and the HCP does not need to be contacted immediately. (Option 3) A knee-chest position is used to treat episodes of hypoxia and cyanosis in infants and young children with tetralogy of Fallot (TOF). This neonate likely has an AV canal defect, not TOF. There is also no indication of cyanosis or hypoxia that would necessitate knee-chest positioning. (Option 4) The normal respiratory rate in a neonate is 30-60/min; pulse can be up to 160/min. The vital signs are stable and the oxygen saturation level is appropriate for a neonate. Educational objective: Trisomy 21 (Down syndrome) is often associated with the cardiac anomaly AV canal defect. Assessment typically includes a loud murmur that requires no immediate action when vital signs are stable. Surgery will correct the anomaly when the neonate grows in size and can tolerate the invasive procedure better.

The nurse prepares a 7-year-old client for an influenza injection. The nurse explains that the client will receive "medicine under the skin," and the client is visibly anxious. Which nursing intervention is appropriate? 1. Ask the child to count to 10 during injection 2. Ask the parent to hold the child's arms tightly 3. Explain to the child that the injection will not hurt 4. Keep the injection needle out of the child's view

1 Children are often fearful of injections, exhibiting unpredictable and/or uncooperative behavior. The nurse should explain the procedure to the child using simple, age-appropriate language (eg, "medicine under the skin") to reduce anxiety. According to Piaget's cognitive developmental stages, school-age children develop concrete thought and may fear a loss of control. To improve the child's sense of control, the nurse should offer a specific, task-based coping technique (eg, counting aloud, deep breathing) (Option 1). (Option 2) A caregiver should hold or embrace a child during the injection process, with the child on the caregiver's lap or standing in front of a seated caregiver. Tightly holding the child's arms is extreme and may distress the child and caregiver. (Option 3) The child should be told the truth about pain that accompanies an injection. The nurse should use appropriate language, such as "the skin may hurt for a minute," and emphasize that the pain is quick and transient. (Option 4) Keeping objects that may alarm the child out of view is an appropriate intervention for a toddler but not for a school-age child. Hiding a procedural object from a 7-year-old will hinder rapport with the nurse and may heighten the child's anxiety. Educational objective: School-age children possess concrete thinking and fear loss of control. When administering an injection to a school-age child, the nurse should offer a specific, task-based coping technique (eg, instruct the child to count aloud or breathe deeply) to increase the child's sense of control and thereby reduce anxiety. Health Promotion and Maintenance

The parents of a hospitalized 3-month-old have to leave the infant while they work. One parent fears that the baby will cry as soon as they walk out. The nurse teaches both parents about separation anxiety. Which statement by the parent indicates that the teaching has been effective? 1. "At this age, my baby will not cry because we are leaving." 2. "I know my baby will feel abandoned when we leave." 3. "My baby is too young to sense my anxiety about leaving." 4. "My baby understands that we will return later in the day."

1 Separation or stranger anxiety occurs when the primary caregivers leave the child in the care of others who are not familiar to the child. This behavior starts around age 6 months, peaks at age 10-18 months, and can last until age 3 years. Separation anxiety produces more stress than any other factor (eg, pain, injury, change in surroundings) for children in this age range. However, this reaction is normal and resolves as the child approaches age 3 years. A 3-month-old can be soothed by any comforting voice (Option 1). (Option 2) A 3-month-old is not developmentally capable of fearing abandonment. (Option 3) A 3-month-old might sense a parent's anxiety but is cognitively unable to process it. (Option 4) A 3-month-old cannot tell time and would not understand the concept of returning later in the day. Educational objective: Separation anxiety starts around age 6 months, peaks at age 10-18 months, and can last until age 3 years. It produces more stress than any other factor (eg, pain, injury, change in surroundings) for children in this age range. However, separation anxiety is normal and resolves by age 3 years. Health Promotion and Maintenance

A parent brings a 6-month-old child to the primary health care provider after the child abruptly started crying and grabbing intermittently at the abdomen. The client's stool has a red, currant jelly appearance. What intervention does the nurse anticipate? 1. Administer epoetin alfa (erythropoietin) 2. Give air (pneumatic) enema 3. Have the parent give 2 ounces of extra juice a day for constipation 4. Perform hemoccult test on stool

2 Intussusception is a process in which one part of the intestine prolapses and then telescopes into another part. It is one of the most frequent causes of intestinal obstruction during infancy. Initially, the telescoping is intermittent, resulting in periodic pain in association with the legs drawn up toward the abdomen. Pain is severe, progressive, and associated with inconsolable crying. Ongoing obstruction can compromise circulation, causing mucosal ischemia, occult bleeding, and, if untreated, grossly bloody "currant jelly" stools (mixture of blood and mucus). A contrast enema is used for diagnostic purposes and often reduces the intussusceptions. An air enema is considered safer than a barium enema. (Option 1) Human recombinant erythropoietin (epoetin alfa [Epogen, Procrit]) stimulates bone marrow to form red blood cells and is used to combat the effects of chemotherapy (due to bone marrow suppression) and/or kidney disease (erythropoietin is secreted by the kidneys). Human recombinant erythropoietin is not indicated in this client. (Option 3) Constipation during infancy usually can be corrected by increasing fluids or adding 2 ounces of pear or apple juice to the daily diet. In addition, eliminating constipating foods and increasing high-fiber foods can help. In this client, it is more important to treat the intussusception as there is no evidence of constipation. (Option 4) A hemoccult test is performed typically when occult (hidden) blood is suspected due to a dark and tarry stool. Blood is evident in intussusception, and so the priority in this client is to treat the cause of the bloody mucus stool. Educational objective: Intussusception (the intestine telescoping into itself) causes intermittent cramping and progressive abdominal pain, inconsolable crying, and currant jelly stool (from blood or mucus). It is often treated successfully with an air enema. Physiological Adaptation

The nurse is caring for an infant diagnosed with Hirschsprung disease who is awaiting surgery. Which assessment finding requires the nurse's immediate action? 1. Abdominal distension with no change in girth for 8 hours 2. Did not pass meconium or stool within 48 hours after birth 3. Episode of foul-smelling diarrhea and fever 4. Excessive crying and greenish vomiting

3 Hirschsprung disease (HD) occurs when a child is born with some sections of the distal large intestine missing nerve cells, rendering the internal anal sphincter unable to relax. As a result, there is no peristalsis and stool is not passed. These newborns exhibit symptoms of distal intestinal obstruction. They have a distended abdomen and will not pass meconium within the expected 24-48 hours. They also have difficulty feeding and often vomit green bile. Surgical removal of the defective section of bowel is necessary and colostomy may be required. A potentially fatal complication is Hirschsprung enterocolitis, an inflammation of the colon, which can lead to sepsis and death. Enterocolitis will present with fever; lethargy; explosive, foul-smelling diarrhea; and rapidly worsening abdominal distension. (Option 1) Mild to moderate abdominal distension is an expected finding with a diagnosis of HD; however, increasing abdominal girth is a serious finding that must be reported. (Option 2) Failure to pass meconium or stool within 24-48 hours after birth is an expected finding of HD. (Option 4) Bilious vomiting and excessive crying are expected findings of HD. In enterocolitis, vomiting can occur more frequently and the client appears more ill. Educational objective: Enterocolitis, a potentially fatal complication of Hirschsprung disease, is characterized by explosive, foul-smelling diarrhea; fever; and worsening abdominal distension. Physiological Adaptation

A 10-year-old is implementing behavioral strategies to manage nocturnal enuresis. The client tells the nurse, "I want to go to sleep-away camp during the summer, but if I have an 'accident,' I'm afraid that other kids will tease me." What is the best response by the nurse? 1. "Don't worry. Your problem will be resolved by then." 2. "It would be better if you thought about going to day camp instead." 3. "We can ask your health care provider about a medication trial that may help." 4. "You could always wear a pull-up just in case."

3 Pharmacological interventions are often used as second-line treatment for nocturnal enuresis in children age >5 years; this is done when there has been little or no response to behavioral approaches and/or when short-term improvement of enuresis is desired for attending sleepovers or overnight camp. A trial run is usually done at least 6 weeks before camp to determine the appropriate drug dose and effectiveness. However, there is a high risk of relapse once the drug is discontinued. Medications used to treat nocturnal enuresis include the following: 1) Desmopressin reduces urine production during sleep. 2) Tricyclic antidepressants such as imipramine, amitriptyline, and desipramine improve functional bladder capacity. (Option 1) This statement gives the client false reassurance. Although nocturnal enuresis resolves eventually, there is no guaranteed time frame. (Option 2) This response ignores the child's desire to go to overnight camp and dismisses any possibility of helpful treatment. (Option 4) Wearing a pull-up could embarrass the child at overnight camp. Educational objective: Pharmacological interventions such as desmopressin and tricyclic antidepressants are often used for nocturnal enuresis treatment in children age >5 years when there has been little or no response to behavioral approaches and/or when short-term improvement of enuresis is desired for attending sleepovers or overnight camp. Health Promotion and Maintenance

A nurse in the neonatal intensive care unit discovers a cyanotic newborn with excessive frothy mucus in the mouth. What should be the nurse's first action? 1. Administer 100% oxygen 2. Auscultate the lungs 3. Place infant in knee-chest position 4. Suction the infant's mouth

4 The initial nursing action for a client experiencing cyanosis and excess oral secretions is suctioning the mouth (ie, oropharynx) to clear the airway (Option 4). Excessive frothy mucus and cyanosis in a newborn could be due to esophageal atresia (EA) and tracheoesophageal fistula (TEF). If EA/TEF is suspected, the infant should be kept supine with the head elevated at least 30 degrees to prevent aspiration. A nasogastric tube should be inserted and connected to continuous or intermittent suction until surgical repair. (Option 1) Oxygen cannot be delivered to the lungs if secretions obstruct the airway. Therefore, suctioning is a priority. (Option 2) This infant is aspirating and in immediate distress, which should be addressed without delay. After suctioning the excess saliva and ensuring a clear airway, the nurse may perform further assessments. (Option 3) This infant's cyanosis is a result of aspirating secretions and does not indicate a circulatory problem. The knee-chest position is appropriate to increase pulmonary blood flow in infants with a cyanotic heart defect (eg, tetralogy of Fallot). Educational objective: The initial nursing action for a client experiencing cyanosis and excess oral secretions is oropharyngeal suctioning to ensure airway patency. Physiological Adaptation

A nurse is caring for a child with acute glomerulonephritis. Frequent monitoring of which of the following is a priority? 1. Blood pressure 2. Hematuria 3. Intake and output 4. Peripheral edema

1 Acute glomerulonephritis (AGN) in children is an immune complex disease most commonly induced by prior group A beta-hemolytic streptococcal infection of the skin or throat. A latent period of 2-3 weeks occurs between the streptococcal infection (eg, pharyngitis) and the symptoms of AGN. Clinical manifestations include periorbital and facial/generalized edema, hypertension, and oliguria, which are primarily due to fluid retention (decreased kidney filtration). The urine is tea-colored and cloudy due to the presence of protein and blood. Although most clients recover spontaneously within days, severe hypertension is an anticipated complication that must be identified early. Monitoring and control of blood pressure are most important as they prevent further progression of kidney injury and development of hypertensive encephalopathy or pulmonary edema. (Option 2) Hematuria is common with AGN. It is usually minimal and resolves spontaneously. Monitoring is important but not a priority. (Option 3) The most important measure of fluid status is a daily weight as it identifies fluid retention and response to treatment. Monitoring intake and output is important but is not the priority action over hypertension monitoring and control. (Option 4) Monitoring for edema is important but not the priority. Moderate sodium restriction is needed, especially if hypertension and edema are present. Otherwise, avoiding high-sodium foods and having no added salt in the diet may be adequate measures. Educational objective: Acute glomerulonephritis is most often caused by recent streptococcal infection. Nursing care is focused on monitoring vital signs (particularly blood pressure) and fluid status, avoiding salt in the diet, and conserving energy.

The nurse is teaching a 9-year-old child with asthma how to use a metered-dose inhaler (MDI). Place the instructions in the appropriate order. All options must be used. - Exhale completely - Deliver one puff of medication into spacer - Place lips tightly around the mouth piece - Rinse mouth with water - Shake MDI and attach it to spacer - Take a slow deep breath and hold for 10 seconds

1. Shake MDI and attach it to spacer 2. Exhale completely 3. Place lips tightly around the mouth piece 4. Deliver one puff of medication into spacer 5. Take a slow deep breath and hold for 10 seconds 6. Rinse mouth with water Educational objective: Any child under age 12 should use a spacer with the MDI to ensure the entire dose is inhaled appropriately.

A nurse is evaluating a client's understanding about infant formula preparation. Which of the following client statements indicate proper understanding? Select all that apply. 1. "I can add extra water to powdered formula if it seems that my baby wants to feed longer." 2. "I can heat formula in the microwave for less than 1 minute." 3. "I must wash the top of concentrated formula cans before opening." 4. "Leftover milk in the bottle may be refrigerated and used at a later feeding." 5. "Unused, prepared formula should be kept in the refrigerator and discarded after 48 hours."

3, 5 Infant formula is readily available as ready-to-feed, concentrate, or powder. Parents should exactly follow the manufacturer's recommendations for preparation. Basic guidelines for preparation, safe storage, and handling of formula include: - Keep bottles, nipples, caps, and other parts as clean as possible (ie, boil or wash in dishwasher). - Wash the tops of formula cans (eg, concentrated formula) with hot water and soap prior to opening to prevent contamination (Option 3). - Refrigerate any unused, prepared formula or unused, opened formula (eg, ready-to-feed, concentrated), but use within 48 hours or discard to reduce the risk of bacterial growth (Option 5). - Warm bottles in a pan of hot water or under warm tap water for several minutes. - Test formula temperature on the inner wrist before serving to the infant (should feel lukewarm, not hot). (Option 1) Never overdilute or overconcentrate formula. Dilution reduces necessary calories, vitamins, and minerals, which hinders growth and development. Overconcentration results in intake of excessive proteins and minerals beyond the excretory ability of the infant's immature kidneys. (Option 2) Never microwave formula as it causes "hot spots" in the milk that can burn the infant's mouth. (Option 4) Any formula in a bottle left over after feeding should be discarded because the infant's saliva has mixed with it, which can foster bacterial growth. Educational objective: Infant formula should never be overdiluted, overconcentrated, or microwaved. Unused, prepared formula should be stored in the refrigerator and, if unused, discarded after 48 hours. Formula left over in a bottle after feeding should be discarded. Health Promotion and Maintenance

The nurse receives new prescriptions for a 6-month-old client with bacterial meningitis. Which action is the priority of care? 1. Administer 400 mg ceftriaxone IV every 12 hours 2. Apply padding to the crib siderails 3. Implement low-stimulation environment 4. Monitor head circumference daily

1 Bacterial meningitis is an inflammation of the membranes covering the brain and spinal cord (ie, meninges) caused by a bacterial infection. The inflammatory process and bacterial growth within the meninges lead to increased volumes of cerebrospinal fluid and, subsequently, increased intracranial pressure (ICP). Without intervention, increased ICP may lead to nerve ischemia, permanent functional impairment (eg, hearing loss, visual impairment, paralysis), brain damage, herniation, and death. The initial priority of nursing care is protecting other clients and staff from exposure, as bacterial meningitis is highly contagious and transmitted by droplets. After isolating the client, the nurse should initiate prescribed antibiotics as quickly as possible, as bacterial meningitis can progress rapidly and lead to death without treatment (Option 1). (Options 2 and 3) Clients with meningitis are at increased risk for seizures. Implementation of a low-stimulation environment (eg, low lighting, minimal noise, uninterrupted rest periods) and seizure precautions (eg, padded side rails) are important interventions. However, initiating antibiotic therapy is the priority. (Option 4) An increasing head circumference may indicate increasing volumes of cerebrospinal fluid and increased ICP. Monitoring for new or worsening signs of increased ICP is important but may be performed after initiating prescribed antibiotics. Educational objective: Bacterial meningitis is an inflammation of the membranes covering the brain and spinal cord that can lead to severe complications (eg, hearing loss, brain damage) or death without treatment. To reduce the risk of complications, the nurse should prioritize initiation of prescribed antibiotic therapy as soon as possible.

The emergency nurse is admitting a 12-year-old client who reports palpitations. Which action should the nurse anticipate? Click the exhibit button for additional information. Vital signs = Temperature 97.1 F (36.2 C); Blood pressure 114/74 mm Hg; Heart rate 234/min; Respirations 24/min; SpO2 97% Physical assessment Neurologic: Alert and oriented to person, place, and time. Follows commands. Cardiovascular: Tachycardia noted. +1 radial pulses. +1 carotid pulses. Jugular veins 1 cm at 45-degree angle. Respiratory: Clear to auscultation. No adventitious sounds. Even and unlabored breathing. Skin: Color appropriate for ethnicity. No cyanosis or mottling. Extremities: No edema in upper or lower extremities. 1. Administering epinephrine by rapid IV push 2. Assisting the client to a tripod position 3. Instructing the client to hold their breath and bear down 4. Sedating the client for immediate asynchronous defibrillation

3 Supraventricular tachycardia (SVT) is the most common tachyarrhythmia of childhood and refers to a rapid heart rate of 200-300/min with no variation in rate during activity. It can lead to life-threatening congestive heart failure if left untreated. Symptoms in children may include palpitations, dizziness, or chest pain. Once an ECG confirms SVT, the nurse should anticipate nonpharmacological interventions (ie, vagal maneuvers) to convert SVT to sinus rhythm if the client is stable. Placing an ice bag to the client's face and instructing the client to hold their breath while bearing down (Valsalva) are vagal maneuvers that can slow electrical conduction through the heart's atrioventricular node (Option 3). If these maneuvers are ineffective, or if the client becomes unstable, administration of adenosine or synchronized cardioversion is indicated. (Option 1) IV epinephrine is not an appropriate treatment for a stable client with tachycardia and would further increase the client's heart rate. Epinephrine is typically used for clients with pulseless arrhythmias (eg, asystole). (Option 2) The tripod position opens the airway and promotes airflow, particularly for clients with significant airway obstruction (eg, epiglottitis). The child with palpitations may assume any position of comfort. (Option 4) Asynchronous defibrillation is indicated for the treatment of lethal cardiac arrhythmias (eg, ventricular fibrillation, pulseless ventricular tachycardia). Educational objective: Supraventricular tachycardia refers to a rapid heart rate of 200-300/min with no variation in rate during activity. The nurse should anticipate instructing the client to perform vagal maneuvers (eg, Valsalva) first if the client is stable.

A 6-month-old client has been diagnosed with cystic fibrosis. Which of the following would be appropriate for the registered nurse to teach to the parents? 1. Monitor for and report development of a "white pupil" 2. Perform manual chest physiotherapy 3. Place child in knee-chest position during hypercyanotic episode 4. Provide a low-calorie diet to prevent obesity

2 Cystic fibrosis is an inherited autosomal recessive disorder of the exocrine glands that results in physiologic alterations in the respiratory, gastrointestinal, and reproductive systems. It is theorized that the chloride transport alternation and resulting thickened mucus inhibit normal ciliary action and cough clearance, and the lungs become clogged with mucus. The thickened mucus harbors bacteria. Over time, airways develop chronic colonization and frequent respiratory infections result. Bronchial hygiene therapy, such as manual chest physiotherapy, is used. For physiotherapy, various positions are used, and this should be performed before meals to avoid a full stomach and resultant regurgitation or vomiting. (Option 1) A white pupil (leukocoria, or cat's-eye reflex) is one of the first signs of retinoblastoma, an intraocular malignancy of the retina. Other symptoms include an absent red reflex, asymmetric or of a differing color in the affected eye, and fixed strabismus (constant deviation of one eye from the other). This disease is not related to cystic fibrosis. (Option 3) Hypercyanotic episodes are associated with tetralogy of Fallot. The knee-chest position increases systemic vascular resistance in the lower extremities. In addition, irritating stimuli should be limited, and supplemental oxygen should be provided. (Option 4) The pancreatic ducts become damaged, and there is a decreased ability to digest fats and proteins and absorb fat-soluble vitamins. Pancreatic enzyme supplements are used. Children with cystic fibrosis tend to be hungry but underweight due to a decreased ability to use fat and its calories. Educational objective: Cystic fibrosis causes thickened mucus, making respiratory infections common. Treatment includes chest physiotherapy performed usually before meals.

The nurse is providing teaching to the parents of a child with Marfan syndrome. Which topic is the priority for the nurse to address? 1. Avoiding participation in contact sports 2. Informing the dentist of the child's condition 3. Monitoring for development of scoliosis 4. Scheduling annual eye examinations

1 Marfan syndrome is an autosomal dominant disorder affecting the connective tissues of the body. Abnormalities are mainly seen in the cardiovascular, musculoskeletal, and ocular systems. Clients with Marfan syndrome are very tall and thin, with disproportionately long arms, legs, and fingers. Cardiovascular manifestations of Marfan syndrome include abnormalities of the aorta and cardiac valves, including aneurysms, tears (dissection), and leaky heart valves that may require replacement or repair. Therefore, competitive or contact sports are discouraged due to the risk of cardiac injury and sudden death (Option 1). (Option 2) The client may also experience crowding of the teeth from a very high-arched palate. Preventive antibiotics prior to dental work may be needed to provide prophylaxis against infective endocarditis, especially in clients with an artificial valve replacement. However, this is not a priority. (Option 3) These clients have an increased risk for scoliosis, especially during the adolescent years of increased growth; therefore, the child should be monitored regularly for curvature of the spine. This is not a priority. (Option 4) Ocular problems (eg, lens dislocation [ectopia lentis], retinal detachment, cataracts, glaucoma) can be common for the child with Marfan syndrome. Annual eye examinations with an ophthalmologist are important to monitor for developing issues. Educational objective: Aortic root disease (ie, aneurysm, dissection) is the major cause of morbidity and mortality in Marfan syndrome. Therefore, competitive or contact sports are discouraged due to the risk of cardiac injury and sudden death. Health Promotion and Maintenance

The nurse is reinforcing discharge teaching for the parents of a 1-year-old with a newly diagnosed cow's milk allergy. Which nutrients normally provided by milk should be obtained from other sources? Select all that apply. 1. Calcium 2. Fiber 3. Iron 4. Vitamin D 5. Vitamin K

1, 4 Calcium and vitamin D are nutrients in cow's milk that are essential for proper bone development in children and adolescents (Options 1 and 4). To obtain the recommended 500 mg of daily calcium (for ages 1-3 years), the parents should serve foods such as beans, dark green vegetables, and calcium-fortified cereals and juices. Vitamin D, which enhances the absorption of calcium, is synthesized in the skin by exposure to direct sunlight. Alternate dietary sources include fish oils, egg yolks, and vitamin D-fortified foods (eg, orange juice). (Option 2) Fiber, which is important for digestive health, is found in only small amounts in cow's milk. Fiber-rich foods include whole grains, beans, and berries. (Option 3) Cow's milk is not a significant source of iron. Dietary sources of iron include meats and spinach. (Option 5) Vitamin K is an important nutrient for coagulation. Vitamin K is produced by bacteria in the large intestine and is found in food sources such as dark green vegetables, fish, and eggs, not in cow's milk. Educational objective: Calcium and vitamin D are nutrients in cow's milk that are essential for proper bone development in children and should be obtained from other sources for clients with a cow's milk allergy. Alternate sources of calcium include beans, dark greens, and calcium-fortified cereal and juices. Vitamin D is synthesized in the skin when exposed to sunlight and can be obtained in foods such as fish, egg yolks, and vitamin D-fortified foods. Basic Care and Comfort

What communication strategies would the nurse have in place when establishing rapport with the caregiver and an 8-year-old during a health history interview? Select all that apply. 1. Ask only closed-ended questions to obtain information 2. Allow the child to describe their current issue 3. Isolate the child from the parents and interview them separately 4. Maintain an eye level position when speaking with the child 5. Use language that both the child and caregiver can understand

2, 4, 5 The first step in effective communication is to establish trust between the nurse, the child, and the parent. By actively including a school-age child in the health history interview, the nurse shows respect to that child and obtains valuable insight into their health status. Allowing the child to describe how they feel or where they hurt gives the nurse a better understanding of the issue. Using clear, age-appropriate explanations will enhance communication with the child while maintaining the participation of the caregiver. Open-ended questions allow the child or caregiver to elaborate on the question, giving the nurse detailed information to guide further assessment. Non-verbal cues also play an important role in communication (eg, staying at eye level with the child to ease any potential nervousness). (Option 1) Closed-ended questions usually result in a "yes" or "no" answer. There are times in an interview that closed-ended questions are appropriate to gather specific information, but broader, more descriptive answers are generally desired when conducting a health history interview. (Option 3) The nurse should interview a school-age child together with their caregiver unless there is an indication of child abuse. The child may feel more at ease, and a more complete assessment may be obtained through answers from both the child and caregiver. Educational objective: It is important for the nurse to develop a rapport with the child and the caregiver based on mutual trust and respect. Strategies for building this relationship include actively including the child in the interview, using age-appropriate explanations, maintaining an eye level position, and asking open-ended questions. Health Promotion and Maintenance

A distraught parent informs the nurse of bleeding in a 1-day-old girl. What is an appropriate response by the nurse after assessing a small amount of bloody mucus in the newborn's diaper? 1. "Laboratory work will need to be completed to determine your newborn's hormone levels." 2. "The health care provider will prescribe a dose of medication to stop the bleeding." 3. "We will continue to monitor the amount, color, and consistency of the drainage." 4. "What visitors have been present since the baby was born?"

3 Mammary gland enlargement, non-purulent vaginal discharge (leukorrhea), and mild uterine withdrawal bleeding (pseudomenstruation) are benign transient findings commonly seen in newborns; these are physiologic responses to transplacental maternal estrogen exposure. Reassurance should be provided. Monitoring the amount, color, and consistency is the appropriate action (Option 3). (Options 1 and 2) The blood-tinged mucus will cease within a few days after birth when hormone levels return to normal. No additional workup or medications are indicated. (Option 4) Pseudomenstruation is a physiological process and is not caused by trauma or abuse. Educational objective: Mammary gland enlargement, non-purulent vaginal discharge (leukorrhea), and mild uterine withdrawal bleeding (pseudomenstruation) are benign transient findings commonly seen in newborns; they are physiologic responses to transplacental maternal estrogen exposure. Reassurance should be provided. Health Promotion and Maintenance

The parents of a 4-year-old tell the nurse that the child won't go to sleep at night due to fear of tigers living under the bed. Which response by the nurse is most helpful? 1. "Have you recently visited the zoo? Maybe the tigers looked scary." 2. "If you agree with your child, the fears could continue through this developmental stage." 3. "Night fears are common at this age. Look under the bed with your child." 4. "This is very unusual. Maybe the child saw something scary on TV."

3 Preschool children (age 3-5) are magical thinkers. Night fears are common during this period, and distinguishing between reality and fantasy is difficult. It is appropriate for parents to acknowledge the child's fears. A preschooler would be comforted and fears would be allayed if the parents looked under the bed and reassured the child that no tigers were there (Option 3). (Option 1) This reply does not educate the parents about normal growth and development. It is not a therapeutic response. (Option 2) Fantasy fears are normal during the preschool years. They are not common during other developmental periods. (Option 4) The parents should be told that magical thinking is common during the preschool period. This is not an accurate or therapeutic response. Educational objective: Magical thinking is common during the preschool period. It is not unusual for a child to have an imaginary friend, and parents should be taught that this is a normal part of development. Magical thinking satisfies children's questions about the world they live in. Health Promotion and Maintenance

Which of the following statements made by the mother of a child recently diagnosed with celiac disease indicates a need for further teaching? 1. "I will need to read the labels of all processed foods." 2. "It is okay if my child eats rice, corn, and potatoes." 3. "My child can have small amounts of foods containing wheat as long as she remains symptom free." 4. "My child will need to be on a gluten-free diet for the rest of her life."

3 The following are important principles to teach clients with celiac disease: 1. All gluten-containing products should be eliminated from the diet. These include wheat, barley, rye, and oats. 2. Rice, corn, and potatoes are gluten free and are allowed on the diet. 3. Deficient vitamins (mainly fat-soluble vitamins), iron, and folic acid should be replaced. 4. Processed foods (eg, chocolate candy, hot dogs) may contain "hidden" sources of gluten such as modified food starch, malt, and soy sauce. Food labels should indicate that the product is gluten free. 5. Clients will need to be on a gluten-free diet for the rest of their lives. Eliminating gluten from the diet reduces the risk of nutritional deficiencies and intestinal cancer (lymphoma). 6. Eating even small amounts of gluten will damage the intestinal villi although the client may have no clinical symptoms. All sources of gluten must be eliminated from the diet (Option 3). Educational objective: All sources of gluten must be eliminated from the diet of a client with celiac disease; consuming small amounts, even in the absence of clinical symptoms, will increase the risk of damage to the intestinal villi. Clients can have foods containing rice, corn, and potatoes. They should read food labels and follow the diet for the rest of their lives. Basic Care and Comfort

The parent of a 1-year-old says to the nurse, "I would like to start toilet training my child as soon as possible." What information does the nurse provide to the parent that correctly describes a child's readiness for toilet training? 1. "A good time to start toilet training is when your child can dress and undress autonomously." 2. "When your child can sit on the toilet until urination occurs, you can start toilet training." 3. "Your child may be ready to start toilet training when able to communicate and follow directions." 4. "Your child will be ready to start toilet training at about age 15 months."

3 Toilet training is a major developmental achievement for the toddler. The degree of readiness progresses relative to development of neuromuscular maturity with voluntary control of the anal and urethral sphincters occurring at age 18-24 months. Bowel training is less complex than bladder training; bladder training requires more self-awareness and self-discipline from the child and is usually achieved at age 2½-3½ years. In addition to physiological factors, developmental milestones rather than the child's chronological age signal a child's readiness for toilet training. These include the ability to: - Ambulate to and sit on the toilet - Remain dry for several hours or through a nap - Pull clothes up and down - Understand a two-step command - Express the need to use the toilet (urge to defecate or urinate) - Imitate the toilet habits of adults or older siblings - Express an interest in toilet training (Option 1) In order to achieve toilet training, the child will need to be able to pull clothing up and down but not necessarily dress and undress autonomously. (Option 2) Having the child sit on the toilet until urination occurs is not appropriate and will not facilitate bladder control; any urination that occurs is accidental and not due to sphincter control. However, the child should have the ability to remain on the toilet for about 5 - 8 minutes without getting off or crying. (Option 4) Age 15 months is too early to begin toilet training; voluntary control of the anal and urethral sphincters does not occur until age 18-24 months. Educational objective: Readiness for toilet training is dependent on the child's ability to voluntarily control the anal and urethral sphincters, which usually occurs at age 18-24 months. Other developmental and behavioral indicators of toilet training readiness include the child's ability to express the urge to defecate or urinate, understand simple commands, pull clothing up and down, and walk to and sit on the toilet. Health Promotion and Maintenance

A 10-year-old weighs 99 lb (44.9 kg) and has a BMI of 24.8 kg/m2 (>95th percentile). Which is the most important assessment for the nurse to make before initiating a weight loss plan? 1. Child's pattern of daily physical activity 2. Family's eating habits 3. Family's financial resources for purchasing healthy foods 4. Family's readiness for change

4 Before initiating a treatment plan for weight loss, it is most important to make certain that the child and family are ready for change. Attempting to engage the family and child in weight loss strategies and dietary changes before they are ready could easily result in frustration, treatment failure, and reluctance to try new approaches in the future. The nurse needs to explore the reasons and desire for weight loss by assessing: - Motivation and confidence - Willingness to change behaviors and food choices - Perceived importance of a weight loss treatment plan - Confidence in ability to take on healthier eating habits (Option 1) Physical activity is an important component of a weight loss treatment plan, but it is not the priority nursing assessment. (Option 2) The family's eating habits will have a strong influence on the child's ability to make changes and need to be assessed. However, it is more important to assess the family's readiness for change. (Option 3) Assessing the family's financial resources is important in planning education about healthy food choices, but it is not the priority nursing action. Educational objective: Before initiating a treatment program that requires a client and family to make major lifestyle and behavior changes, the nurse needs to assess readiness for change. Motivation and a desire for change are the keys to successful weight loss. Health Promotion and Maintenance

The nurse is discussing child safety with the parents of a 12-month-old who is just beginning to walk. Which statement by the parents indicates a need for further instruction? 1. "Our swimming pool is fenced in with a lock on the gate." 2. "We have installed childproof gates at the top and bottom of our stairs." 3. "We need to lower the mattress in our child's crib." 4. "When we can't be watching, we put our child in a mobile child walker."

4 Due to the relatively high incidence of injuries associated with child walkers, the American Academy of Pediatrics has recommended a ban on the manufacture and sale of mobile infant walkers. Accidents associated with child walkers include: - Rolling down stairs (the most common cause of injury) - Burns - children can reach high in a walker, enabling them to grab hot pot handles, reach heaters and fireplaces, or grab a hot cup of liquid off a counter or table - Drowning - a child can fall into a bathtub or pool while in a mobile walker - Poisoning - the child can reach higher objects Even if a parent is close by and watching a child in a walker, an accident may not be preventable. Children can move quickly and the parent or caregiver may not be able to respond quickly enough. Safer alternatives to mobile baby walkers include stationary walkers (no wheels) and play areas. If parents or caregivers insist on using a baby walker, they should be advised to choose one that meets the American Society for Testing and Materials safety standards. Walkers with braking mechanisms stop if at least one wheel drops off the riding surface. (Option 1) This is an appropriate action; swimming pools should be surrounded by fences with childproof locks to prevent accidental drowning. Wading pools and all water containers should be emptied after each use. (Option 2) This is an appropriate action; childproof gates should be installed on stairs and at the entrances to rooms that could pose danger to a child. (Option 3) This is an appropriate action; as children grow taller and can stand, they may be able to crawl over the crib rails and fall. Educational objective: Mobile baby walkers are associated with injuries such as falls and drowning as they can easily tip over. Children can also reach higher places while in a baby walker, enabling them to pull hot objects and dangerous substances off counters and tables. Safety and Infection Control

A 14-year-old is scheduled for surgery to treat scoliosis. The child will be hospitalized for about a week and then discharged home to recuperate for 3-4 weeks before returning to school. What is the best activity the nurse can recommend to promote age-specific growth and development during this time? 1. Attending selected after-school events and social activities 2. Keeping up with schoolwork 3. Reading teen magazines 4. Visits from friends

4 During adolescence, being with a peer group is part of the process of achieving individual identity, the most important developmental task at this age. An adolescent's friends have more influence than parents, teachers, or any other adults. Social relationships and activities help to provide a sense of belonging, acceptance, and approval. Having face-to-face visits and spending time with friends will help counteract feelings of isolation and loneliness during the client's recuperative period. In addition, the client is at risk for body image disturbance related to the scoliosis and surgery. The client may be particularly sensitive about body image and needs understanding and acceptance from peers. (Option 1) The client can attend school functions or social activities with friends when off all pain medication and when the spine has healed sufficiently. (Option 2) It is important for the client to keep up with schoolwork, but it is not a priority for recovery. (Option 3) Reading teen magazines can be a diversionary activity and may help distract the client from any pain, but it is not a priority. Educational objective: Friends play a significant role in the adolescent's quest for identity and provide a source of support, belonging, and understanding. Interacting with friends during recuperation after surgery is important to help counteract feelings of loneliness and isolation. Health Promotion and Maintenance

The public health nurse has received a referral to make a follow-up home visit to a 1-year-old recently diagnosed with failure to thrive (FTT). Which intervention is the priority nursing action for this child? 1. Assess overall parenting skills 2. Complete a 24-hour dietary intake 3. Measure the child's height, weight, and head circumference 4. Observe the child feeding

4 FTT is generally defined as weight less than 80% of ideal for age and/or depressed weight for length, correcting for gestational age, sex, and special medical conditions. The underlying cause of FTT is inadequate dietary intake; contributing factors include a disturbance in feeding behavior and psychosocial factors. Observing the child feeding or when hungry will provide the nurse the opportunity to identify potential factors contributing to insufficient intake. The nurse can observe the type of food being offered, the quantity of food consumed, how the child is held or positioned while being fed, the amount of time for feeding, the parent's response to the child's cues, the tone of the feeding, and the interaction between the child and the parent. (Option 1) As part of the home visit, the nurse will assess overall parenting skills. It is most important for the focus of the visit to be on the nutritional intake of the child and the feeding experience. (Option 2) A 24-hour dietary intake is an assessment tool to obtain information regarding nutritional intake. However, because the child's intake would be reported by the parent, it may not be accurate and does not provide information about what takes place during the feeding itself. (Option 3) This is an appropriate nursing action, but it provides no information about the factors contributing to the child's insufficient intake. Educational objective: FTT is generally defined as weight less than 80% of ideal for age and/or depressed weight for length, correcting for gestational age, sex, and special medical conditions. Observation of the child while being fed may provide information related to the cause of inadequate dietary intake, including disturbances in feeding behavior and psychosocial factors. Basic Care and Comfort

The nurse is performing a well-child assessment on a sleeping 2-month-old client. Organize the assessment in the correct order based on the developmental age of the client. All options must be used. 1. Assess pupillary response 2. Auscultate heart and lungs 3. Elicit Moro reflex 4. Observe skin color and respiratory pattern 5. Palpate fontanelles and abdomen

4. Observe skin color and respiratory pattern 2. Auscultate heart and lungs 5. Palpate fontanelles and abdomen 1. Assess pupillary response 3. Elicit Moro reflex The sequence of examination steps for infants requires a different approach than the typical head-to-toe order used for older clients. The steps are altered to accommodate the developmental needs of the infant, minimize the infant's stress, and increase assessment accuracy. The correct order of assessment in infants is: - Before handling the infant, the nurse first observes the infant for activity level, skin color, and respiratory rate and pattern to obtain findings during a calm state (Option 4). - Auscultation is performed next while the infant is still quiet, allowing the nurse to hear sounds clearly (Option 2). - Palpation and percussion are then performed while the infant remains relatively still. This allows the nurse to accurately assess the abdomen while the abdominal muscles are relaxed. The fontanelles are also palpated while the infant is calm, as crying can cause temporary bulging (Option 5). - Traumatic procedures (eg, examine eyes, ears, mouth) are performed near the end of the assessment after completing any procedures that require accurate observation or counting (Option 1). - Elicitation of the Moro reflex (ie, reflexive startle and cry to a sudden dropping or jarring motion) is performed last because the infant is usually awake and moving around by this point (Option 3). Educational objective: When assessing an infant, the nurse should observe, auscultate, palpate, and then perform traumatic procedures (eg, examine eyes, ears, mouth). Elicitation of the Moro reflex should be performed last. Health Promotion and Maintenance

A 3-month-old client has stopped breathing. Identify the area where the nurse should check the client's pulse. Left-clicking the mouse will place an X to show the answer before submitting the question.

The American Heart Association's guidelines for infant cardiopulmonary resuscitation (CPR) are used on children age <1 year. To check a pulse on an infant, the nurse should palpate the brachial artery by placing 2 or 3 fingers halfway between the shoulder and elbow on the medial aspect of the arm. The pulse should be assessed for 5-10 seconds to determine its presence and quality before CPR is initiated. The brachial pulse is preferred in infants as the brachial artery is close to the surface and is easily palpable. The carotid pulse can be difficult to assess due to a child's shorter neck. Extending an infant's neck to attempt to palpate the carotid pulse can cause injury. This pulse is recommended for clients age >1 year. The femoral pulse may be used for all clients; however, it is often not easily accessible for palpation due to diapers and clothing. The radial pulse is used in responsive clients age >1 year. It is not a recommended method of pulse detection in an unresponsive client as a weak or thready pulse is difficult to palpate at this location. Educational objective: According to the infant cardiopulmonary resuscitation guidelines of the American Heart Association, the brachial artery is used to detect a pulse in an unresponsive client age <1 year.

The nurse is assessing a 3-month-old during a well-child visit. Which developmental finding should the nurse expect to observe in the client? 1. Infant cries and clings to parent when members of the health team come near 2. Infant kicks legs, smiles, and coos when a familiar face comes into view 3. Infant transfers a ball from one hand to the other hand 4. Infant turns from the back to the abdomen

2 Developmental milestones (eg, motor, sensory, verbal, cognitive) are known patterns of growth and development noted in most children by a specific age. These milestones are used as a general assessment guide, although each child has a unique pattern of development. By age 3 months, the infant recognizes familiar items and faces (Option 2). Any 3-month-old who does not respond to familiar faces may have visual impairment or an underlying neurological disorder (eg, autism). (Option 1) Stranger anxiety is part of the infant's normal social and cognitive development and usually begins around age 6 months. (Option 3) Transferring objects from one hand to the other hand is a fine motor skill that usually develops between age 6 and 9 months. Failure to develop this skill may indicate neuromuscular or developmental delays. (Option 4) A 3-month-old is usually not strong enough to roll from the back to the front. Infants should be able to turn from the abdomen to the back at around age 4 months and then from the back to the abdomen by age 6 months. Failure to roll over by age 6 months may indicate slower-than-normal neck, leg, back, and arm muscle development and should be investigated. Educational objective: Developmental milestones (known patterns of growth and development) are used as a general assessment guide. By age 3 months, the infant should recognize familiar items and faces. Any 3-month-old who does not respond to familiar faces (eg, by cooing) may have visual impairment or an underlying neurological disorder (eg, autism).

The nurse in a clinic is obtaining a developmental history of an 18-month-old during a well-child visit. Which activities should the child be able to perform? Select all that apply. 1. Calls self by name 2. Goes up stairs while holding a hand 3. Stacks 6 blocks in a tower 4. Turns 2 pages in a book at a time 5. Twists doorknob to open doors

2, 4 A toddler's development centers on both fine and gross motor skills. By 18 months, the toddler can manage stairs while holding a hand and turn 2 or 3 pages in a book. The direction in development is on improving the skill of locomotion (Options 2 and 4). (Options 1, 3, and 5) A 24-month-old should be able to build a tower of 6 or 7 blocks, call self by name, and use a doorknob to open a door. Educational objective: An 18-month-old is developing both fine and gross motor skills, which include going up stairs while holding a hand and turning 2 or 3 pages in a book.

When monitoring an infant with a left-to-right-sided heart shunt, which findings would the nurse expect during the physical assessment? Select all that apply. 1. Clubbing of fingertips 2. Cyanosis when crying 3. Diaphoresis during feedings 4. Heart murmur 5. Poor weight gain

3, 4, 5 Congenital heart defects that cause blood to shunt from the higher pressure left side of the heart to the lower pressure right side (eg, patent ductus arteriosus, atrial septal defect, ventricular septal defect) increase pulmonary blood flow. Left-to-right shunting results in pulmonary congestion, causing increased work of breathing and decreased lung compliance. Compensatory mechanisms (eg, tachycardia, diaphoresis) result from sympathetic stimulation. Clinical manifestations of acyanotic defects may include: - Tachypnea - Tachycardia, even at rest - Diaphoresis during feeding or exertion (Option 3) - Heart murmur or extra heart sounds (Option 4) - Signs of congestive heart failure - Increased metabolic rate with poor weight gain (Option 5) (Option 1) Clubbing of the fingertips is associated with chronic hypoxia caused by decreased pulmonary circulation as occurs with right-to-left heart defects. (Option 2) Right-to-left congenital heart defects (eg, cyanotic defects) impede pulmonary blood flow (eg, tetralogy of Fallot, transposition of the great vessels) and cause cyanosis, which is evident shortly after birth and during periods of physical exertion. Educational objective: Left-to-right cardiac shunts (eg, patent ductus arteriosus, atrial septal defect, ventricular septal defect) result in excess blood flow to the lungs. Manifestations include heart murmur, poor weight gain, diaphoresis with exertion, and signs of heart failure.

The public health nurse conducts a teaching program for parents of infants. Which statement by a participant indicates that teaching has been successful? 1. "After age 6 months, it is safe to use honey to sweeten my infant's formula." 2. "I should wait until my infant is 1 year old to introduce egg products." 3. "I will switch my 1-year-old to low-fat milk instead of commercial formula." 4. "My infant should be able to pick up small finger foods by age 10 months."

4 The pincer grasp, a thumb to forefinger movement, develops at age 8-10 months. This is the time to start offering small finger foods, such as crackers or cut-up pieces of nutritious foods. Caregivers should inform their health care provider if the infant does not achieve this significant milestone in fine motor development (Option 4). (Option 1) Formula should never be sweetened. Honey (especially raw or wild) should not be offered to children age <12 months because their immature gut systems are susceptible to Clostridium botulinum (botulism) infection. (Option 2) Common allergenic foods (eg, eggs, fish, peanut products) may be introduced along with other foods starting at age 4-6 months. Previous guidelines recommended delaying introduction of these foods until age 12 months. However, recent evidence suggests that delaying introduction of these foods may actually increase the risk for food allergy. (Option 3) Infants should be transitioned to whole milk, not low-fat milk, at age 12 months. Due to rapid growth, a child's brain requires the nutrition from the fat found in whole milk. Educational objective: The pincer grasp should be present by age 10 months. Offering small finger foods allows the infant to develop fine motor skills. The child will also enjoy the ability to self-feed and explore a variety of nutritious foods.

The parents of a 5-year-old ask the school nurse for advice on how to tell their child about being adopted. Which developmentally appropriate thought about adoption by the child does the nurse counsel the parents to anticipate? 1. Feels responsible for being placed for adoption 2. Imagines what life would be like with a different family 3. Is unable to conceptualize differences between adoptive and biological parents 4. Worries about what peers will say or think

1 Children age 3-6 (preschool) are in Piaget's preoperational stage of cognitive development. At age 5, children are not able to fully understand cause and effect and will therefore ascribe inappropriate causes to phenomena (eg, scraped knee was caused by earlier misbehavior). Five-year-olds are developmentally capable of understanding adoption on a basic level; however, it may be difficult for them to understand the concept of having another family. The child might notice that friends are not adopted. Preschool-age children may also believe they are responsible for being adopted and may develop separation issues and fear abandonment. (Option 2) School-age children may imagine how life would be different if they were with their biological parents. Self-esteem issues begin to develop around this time with the possible sense of loss of the biological family. School-age children may be sensitive to physical differences between themselves and their adoptive family. (Option 3) Children age 3 and under are unable to understand differences between adoptive and biological parents. The age at which to begin discussions about being adopted is debatable, but use of positive language is always encouraged. The terms "given up" or "put up" for adoption or someone's "real" parents are inappropriate and should be avoided. (Option 4) Adolescents have abstract thinking abilities that enable introspection about their adoption. They typically do not like differing from their peers. Open and honest communication is important at this age. Educational objective: Preschool-age children (3-6 years) are in Piaget's preoperational stage of cognitive development. They become increasingly verbal but are unable to understand cause and effect, often ascribing inappropriate causes to phenomena; therefore, the adopted child may feel responsible for being adopted. Health Promotion and Maintenance

The school nurse is conducting an educational session for middle school teachers that is designed to heighten awareness of school bullying. The nurse recognizes that further instruction is needed when one of the teachers makes which comment? 1. "Bullying is a normal part of childhood growth and development." 2. "Children with physical disabilities are more vulnerable to bullying." 3. "Most children who are victims of a school bully do not tell an adult about it." 4. "The most common form of bullying is verbal aggression, such as insults and intimidation."

1 Despite increased media coverage, articles, and discussion about school bullying, myths and "old" attitudes toward school bullying persist. A number of parents and teachers continue to perceive bullying as a normal part of childhood, with the attitude of "kid will be kids." Some may even believe that bullying is "fun" and will help make those who are bullied stronger over time. These beliefs are one of the most common reasons why teachers and parents may ignore bullying behavior. The consequences of bullying are lasting harm and distress, including anxiety, depression, school adjustment problems, sleep difficulties, and even death. (Option 2) This is a true statement. Children who bully often target those who seem different or are physically weak. Children with disabilities are easy targets. (Option 3) This is a true statement. Most victims of bullying are afraid to tell an adult for fear that the bullying will get worse. They may also feel embarrassed that they appear to be a "weakling." (Option 4) Bullying includes physical, verbal, and psychological aggression. Studies indicate that verbal abuse (eg, yelling obscenities, derogatory remarks, intimidation) is the most common type of bullying. Physical bullying tends to decrease from middle school to high school, but verbal bullying intensifies. Verbal attacks are more difficult to identify as they occur when adults are out of earshot. Educational objective: Bullying is not a part of normal childhood growth and development. It is abusive behavior that can have lasting and harmful physical and psychological effects on its victims. Health Promotion and Maintenance

A nurse is discussing the concept of parallel play with parents of toddlers. Which statement should the nurse include to describe this type of play? 1. "Children play near other children but without significant interaction." 2. "Children playing together are strongly influenced by each other's choice of toy." 3. "The child primarily plays alone or with familiar people, such as parents." 4. "When playing in a group, one child will take on a follower role."

1 Play is an important developmental task of childhood and reflects the child's physical, social, and emotional health. Parallel play is independent play near other children with minimal group interaction and is typical of toddlers (age 12-36 months) (Option 1). Toddlers engaging in parallel play may share toys and verbalize thoughts, but they primarily focus on doing their own activities rather than directly interacting with others in organized play. (Option 2) Toddlers engaging in parallel play may play with similar toys but are not directly influenced by the choices of other children. Preschoolers (age 3-5 years) are more likely to interact with each other and borrow each other's toys in associative play. (Option 3) During solitary play, children play alone while focusing on their own activities but also enjoy interactions with familiar people (eg, parents) or objects (eg, favorite toy). This type of play is typical of infants (age <12 months). (Option 4) During cooperative play, one or two children direct the activity and assign roles while others follow. This type of play is typical of school-age children (age 6-12 years) and may involve a formal game or task (eg, building a castle from blocks). Educational objective: Toddlers typically engage in parallel play, which is characterized by independent play alongside other children. Minimal interaction is observed between children during parallel play because toddlers focus on their own activities rather than interact with others in organized play. Health Promotion and Maintenance

The school nurse assesses an 8-year-old with a history of asthma. The nurse notes mild wheezing and coughing. Which action should the nurse perform first? 1. Assess the client's peak expiratory flow 2. Call the health care provider (HCP) 3. Educate the client about avoiding triggers 4. Notify the client's parents

1 Symptoms of an asthma exacerbation include wheezing, chest tightness, dyspnea, cough (may be nocturnal, dry, or productive), and retractions. A cough is often the earliest sign of an asthma exacerbation in children. Bronchospasm leads to CO2 trapping and retention. The bronchospasm forces the client to work harder to exhale and the expiratory phase becomes prolonged. The nurse needs to further assess this client to validate the severity of the exacerbation before implementing an intervention. By assessing the client's peak expiratory flow, the nurse can determine the severity of the symptoms. The nurse will also need to assess the client's respiratory rate and lung sounds. (Option 2) Additional information is needed before notifying the HCP to determine the severity of the client's current condition. (Options 3 and 4) The client's parents do need to be notified and discuss asthma triggers with the nurse. However, these are not a priority as the client is currently symptomatic. Educational objective: The nurse must determine the severity of a client's condition before implementing an intervention. By assessing this client's peak expiratory flow, the nurse can determine the severity of the asthma symptoms.

The parent of a 2-year-old tells the nurse at the well-child clinic, "I am concerned because my child does not like to be cuddled, does not respond when called by name, and does not make eye contact when being fed." What is the priority question for the nurse to ask when completing the health history? 1. "How many words can your child say?" 2. "Is your child potty trained?" 3. "What are your child's favorite foods?" 4. "What kind of toys does your child like to play with?"

1 The concerns presented by this child's parent are suggestive of a developmental delay and very possibly autism spectrum disorder (ASD). ASD is a complex neurodevelopmental disorder characterized by the onset of abnormal functioning before age 3. The 2 core symptoms of ASD are abnormalities in social interactions and communication (verbal and nonverbal), and patterns of behavior, interests, or activities that can be restricted and repetitive. Social skills, especially communication, are delayed more significantly than other developmental functioning and are the focus during client assessment. The vast majority of children diagnosed with ASD lack the acquisition of communication skills during the first 2 years of life. A healthy 2-year-old should have a vocabulary of about 300 words and should be able to string 2 or more words together in a meaningful phrase. Assessing this child's language abilities would be the priority. (Option 2) Assessing any 2-year-old's progress in toilet training is appropriate. However, it is not the priority assessment given the parent's concerns. (Option 3) A nutrition assessment is part of every well-child visit, but it is not the priority in this situation. (Option 4) Although not the priority assessment, it would be important to ask the parent about the child's play activities. Children with ASD often have a restricted interest in and preoccupation with a single toy, exhibit repetitive behaviors when playing with the toy, and insist on the same play routine. Educational objective: The 2 core symptoms of autism spectrum disorder are abnormalities in social interactions and communication (verbal and nonverbal), and patterns of behavior, interests, or activities that can be restricted and repetitive. Social skills, especially communication, are delayed more significantly than other developmental functioning. Health Promotion and Maintenance

The nurse is conducting a psychosocial developmental checkup on a 2-year-old child. What is the priority assessment finding that should be reported to the primary health care provider? 1. Does not talk or respond to being talked to or read to 2. Likes to imitate others by playing house and talking on the telephone 3. Rides a Big Wheel and plays with a softball and bat 4. Says "no" to everything and throws temper tantrums

1 Toddlers experience a phenomenal growth of language skills. They have many ways of communicating, some of them nonverbal, but they enjoy and learn by being talked to and read to. When toddlers do not enjoy these interactions or are not expressing themselves verbally, speech and hearing deficits should be explored. Many deficits in speech and hearing are correctable, or therapy may enhance quality of life. (Option 2) Imitating others is a normal pattern of psychosocial development for toddlers. They adapt to their role in the family unit and society by imitating the same-sex parent. (Option 3) Riding a Big Wheel as fast as possible and hitting a softball with a soft bat helps the toddler to relieve stress in a constructive and learning environment. (Option 4) Toddlers are trying to express themselves and gain independence over their own bodies and actions. Temper tantrums are also a way of relieving stress. They say "no" to express their independence. Educational objective: If toddlers are not expressing themselves verbally and do not enjoy being talked to or read to, speech and hearing deficits should be explored. Health Promotion and Maintenance

Which nursing interventions should be included in the plan of care for a newborn with suspected esophageal atresia (EA) and tracheoesophageal fistula (TEF)? Select all that apply. 1. Keep the infant nothing by mouth (NPO) status 2. Maintain the infant supine with the head elevated 30 degrees 3. Place suction equipment by the infant's bed 4. Prepare for urgent gastrostomy tube placement to start feedings 5. Refer family to palliative care team

1, 2, 3 In the most common form of EA/TEF, the upper esophagus ends in a blind pouch and the lower esophagus connects to the primary bronchus or the trachea through a small fistula. EA/TEF can usually be corrected surgically. Clinical manifestations include frothy saliva, choking, coughing, and drooling. Clients may also develop apnea and cyanosis when feeding. Aspiration is the greatest risk for clients with EA/TEF. Priority nursing interventions for infants with suspected EA/TEF include maintaining NPO status, positioning the client supine, elevating the head at least 30 degrees, and keeping suction equipment by the bed to clear secretions from the mouth. If surgery must be staged or delayed due to the infant's condition, the priority is to maintain a clear airway and prevent aspiration. (Option 4) This client will likely require parenteral nutrition prior to surgery. A gastrostomy tube may be placed to allow for release of air and drainage of gastric contents to prevent aspiration; however, feedings or irrigations through the tube are contraindicated until after surgical correction of the TEF. (Option 5) Surgical correction is successful in most cases of EA/TEF. Infants diagnosed with extreme forms or with additional congenital anomalies may require referral to palliative care services if surgical correction fails. Educational objective: Priority nursing interventions to prevent aspiration in infants with EA/TEF include maintaining NPO status, positioning the child supine with the head elevated at least 30 degrees, and keeping suction equipment available by the bed. Physiological Adaptation

A nurse is caring for a 1-month-old client who is being evaluated for congenital hypothyroidism. The nurse should recognize which of the following as clinical manifestations of hypothyroidism in infants? Select all that apply. 1. Difficult to awaken 2. Dry skin 3. Frequent, loose stools 4. Hoarse cry 5. Tachycardia

1, 2, 4 Congenital hypothyroidism occurs when abnormal development of the thyroid gland causes complete or decreased secretion of thyroid hormone (TH). Untreated hypothyroidism can cause severe intellectual disability in infants if undetected. Screening occurs after birth for all infants in the United States and Canada to prevent disability and encourage early treatment (ie, levothyroxine). TH plays an important role in growth, development, and regulation of many bodily functions (eg, heat production, muscle tone, skin function, cardiac function, metabolism). Clinical manifestations in affected infants reflect the pathophysiology of decreased TH and may include: - Difficulty awakening, lethargy, or hyporeflexia due to alterations in central nervous system function (Option 1) - Dry skin due to alterations in skin function (Option 2) - Hoarse cry caused by swelling of the vocal cords due to fluid retention (Option 4) - Constipation due to slowed metabolism - Bradycardia due to the effect of TH on cardiac function (Options 3 and 5) Hyperthyroidism (Graves disease) is an autoimmune condition related to increased production of TH. Neonatal Graves disease is uncommon and usually occurs secondary to maternal hyperthyroidism. Tachycardia and increased bowel motility (frequent or loose stools) are features of hyperthyroidism and are related to an increase in metabolic processes. Educational objective: Congenital hypothyroidism is a partial or complete loss of thyroid function that affects growth, development, and regulation of bodily functions. Clinical manifestations in affected infants may include dry skin, hoarse cry, or difficulty awakening beginning a few months after birth. If untreated, intellectual disability may occur.

A nurse on a pediatric unit is reviewing interventions for a toddler with a practical nurse who will be caring for this child. Which of the following are appropriate activities to minimize the effect of hospitalization on a toddler? Select all that apply. 1. Integrate preferred snack foods in the day's routine 2. Plan quiet play prior to usual nap time 3. Point out body changes that may occur 4. Post a daily schedule by the child's bed 5. Provide 1 or 2 options when choosing toys

1, 2, 5 Toddlers (age 1-3) display an egocentric approach as they strive for autonomy. They attempt to control their experiences through intense emotional displays, such as temper tantrums or forceful negative responses (eg, "no!"). Hospitalization results in loss of a toddler's usual routines and rituals, often resulting in regressive behavior. The toddler may also be frequently separated from the parents, leading to separation anxiety. Nursing care activities should be similar to home routines, such as providing preferred snacks and anticipating nap time. The toddler should be given options rather than asked yes/no questions to limit the potential negative responses. It is also important to encourage participation and presence of the parents whenever possible. (Option 3) This is an appropriate activity when working with an adolescent. Adolescents are often very concerned with outward changes that may occur as a result of illness or surgery. (Option 4) This is an appropriate activity when working with school-age children after they have grasped the concept of time. Toddlers have not yet reached this level of cognition. Educational objective: Toddlers react to the experience of hospitalization with a display of intense emotions, regressive behaviors, and manifestations of separation anxiety. Nursing care centers on integrating home routines into planned activities.

A 15-year-old client with type 1 diabetes mellitus (DM) is admitted to the pediatric intensive care unit with a blood glucose level of 460 mg/dL (25.5 mmol/L). The nurse understands that which factor is contributing to this client's noncompliant behavior? 1. Client is depressed and wants to die 2. Client's psychosocial developmental stage 3. Lack of supervision by the client's caregivers 4. Limited understanding of the disease process

2 Certain behaviors are common in the adolescent period, ages 11 (early adolescence) to 20 (late adolescence). Teenagers engage in risk-taking behaviors and want to be just like their peers. Adolescents with chronic disease may have difficulty managing their illness due to a false sense of security and the belief that nothing bad can happen to them. (Option 1) Although the client may not be managing the disease well currently, there is nothing to indicate that the child is suicidal. (Option 3) It is unrealistic to expect the parents of an adolescent with DM to monitor the child's diet. Teenagers have developed their own code of behavior and need independence. (Option 4) An adolescent is fully able to understand disease management, including risks of noncompliance, but may choose to ignore the issue out of a desire to be like peers. Educational objective: Adolescence in psychosocial development is marked by risk-taking behaviors, a sense of invincibility, the need for independence, and a strong connection to peers. Health Promotion and Maintenance

The nurse is gathering data on a 5-week-old admitted with a suspected diagnosis of pyloric stenosis. The nurse should expect to find which laboratory value? 1. Blood pH of 7.1 2. Hematocrit of 57% (0.57) 3. Potassium of 5.2 mEq/L (5.2 mmol/L) 4. White blood cells of 28,500/mm3 (28.5 x 109/L)

2 In pyloric stenosis, a hypertrophied pyloric muscle causes postprandial projectile vomiting secondary to an obstruction at the gastric outlet. An olive-shaped mass may be palpated in the epigastric area just to the right of the umbilicus. Emesis is nonbilious (formula in/formula out) and leads to progressive dehydration. Infants will be hungry constantly despite regular feedings. A hematocrit of 57% (0.57) is elevated and indicative of hemoconcentration caused by dehydration (Option 2). Elevated blood urea nitrogen is also a sign of dehydration. (Option 1) The stomach contains acid, which becomes depleted with excess vomiting (or during nasogastric [NG] suctioning), leading to metabolic alkalosis (increased bicarbonate and pH of >7.45). (Option 3) A potassium level of 5.2 mEq/L (5.2 mmol/L) is considered slightly elevated. However, vomiting or prolonged NG suctioning would cause hypokalemia, not hyperkalemia. (Option 4) A white blood cell count of 28,500/mm3 (28.5 x 109/L) is elevated, indicating infection. However, pyloric stenosis is not an infectious process. Educational objective: Hypertrophic pyloric stenosis results in recurrent projectile vomiting, which leads to dehydration and hypokalemic metabolic alkalosis. Dehydration is manifested by hemoconcentration (elevated hematocrit) and elevated blood urea nitrogen. Reduction of Risk Potential

The nurse cares for a 4-year-old who is on long-term, strict bed rest. Which toy is most appropriate to provide diversion and minimize developmental delays? 1. Board games 2. Puppets 3. Soap bubbles 4. Stacking and nesting toys

2 Play is an integral part of a child's mastery of emotional, social, and physical development. When a child is hospitalized, play can also serve as a diversion and a way to express stress and anxiety. Preschoolers enjoy play that enables them to imitate others and be dramatic. They have rich imaginations and enjoy make-believe. Their play often centers on imitating adult behaviors by playing dress up and using housekeeping toys, telephones, medical kits, dolls, and puppets. Quiet play appropriate for the preschooler includes finger paints, crayons, illustrated books, puzzles with large pieces, and clay. Through playing with objects such as dolls or puppets, preschoolers can often process fears and anxieties that are difficult for them to express. (Option 1) Board games are appropriate for children of school age, when play becomes more complex and competitive. (Option 3) Soap bubbles are appropriate for toddlers, who learn from tactile play and environmental exploration. (Option 4) Stacking and nesting toys are appropriate for toddlers who are developing fine motor skills. Educational objective: Play serves as an important part of children's emotional, social, and physical development. It is important that they be provided with toys that can help them achieve developmental tasks. Appropriate toys for preschoolers are those that encourage imitation of adults, such as dolls, puppets, imaginative toys, dress-up clothing, medical kits, cars, and planes. Health Promotion and Maintenance

The nurse is teaching the parents of a 4-month-old who has developed positional plagiocephaly (flat head syndrome). Which statement by the parents indicates a need for further teaching? 1. "I should alternate head positions when the infant is supine." 2. "I should place the infant in the prone position during naps." 3. "I will minimize the amount of time the infant is in a car seat." 4. "I will place interesting toys opposite the affected side."

2 Positional plagiocephaly (flat head syndrome) occurs when an infant's soft, pliable skull is placed in the same position for an extended time. Positional plagiocephaly has become common due to the Safe to Sleep (formerly Back to Sleep) campaign, which advocates for infants to sleep in the supine position to prevent sudden infant death syndrome (SIDS). The risk of SIDS outweighs the benefit of a shapely head; the infant should not be placed in the prone position to sleep, even for a daytime nap (Option 2). Plagiocephaly can usually be prevented or corrected by: - Frequently alternating the supine infant's head position from side to side (Option 1) - Minimizing the amount of time an infant's head rests against a firm surface (eg, car seat) (Option 3) - Placing pictures and toys opposite the favored (affected) side to encourage turning the head (Option 4) - Placing the infant in the prone position for 30-60 min/day ("tummy time") Educational objective: Positional plagiocephaly, or flattening of the skull, can develop when infants spend a lot of time in the same position. Positioning techniques (eg, "tummy time," alternating the head position) can prevent or correct plagiocephaly. Infants should always be placed in the supine position to sleep. Basic Care and Comfort

The nurse is caring for a preschool-age child whose grandparent died 3 days ago. Which intervention is inappropriate? 1. Assign the same nurses and caregivers to the child each day 2. Avoid mentioning the loved one's death in the child's presence 3. Explain the importance of being with the child to the parents 4. Schedule time each day for age-appropriate play

2 The preschool-age (3-5 years) child's view of death is related to their developmental stage. They believe death is temporary and reversible, similar to a prolonged nap. The child may ask repeatedly when the deceased individual will return, or they may feel guilty and responsible for the death because of their wishes or thoughts (magical thinking). Talking about the death in simple, accurate terms as often as needed helps the preschool-age child to process their loss. Avoiding discussion of the loved one's death is not therapeutic and may increase anxiety or cause confusion (Option 2). (Option 1) Familiar faces are comforting to the child, and consistently assigning the same nurses and caregivers promotes therapeutic relationships and trust. (Option 3) When considering the idea of death, preschool-age children have significant fear of separation from their parents. Therefore, it is appropriate to explain the importance of remaining with the child as much as possible to the parents. (Option 4) Play allows the child to cope with grief and provides an outlet to express or work through feelings/experiences that the child may not be able to vocalize. Educational objective: Therapeutic interventions for preschool-age children who are experiencing the death of a loved one include providing familiarity (eg, same nurses, parental presence), ensuring that time each day is devoted to play, and speaking openly to the child about the death as often as needed. Health Promotion and Maintenance

The nurse is assessing an infant with intussusception. Which of the following clinical findings should the nurse expect? Select all that apply. 1. Palpable olive-shaped mass in epigastrium 2. Palpable sausage-shaped abdominal mass 3. Projectile vomiting without visualized blood 4. Screaming and drawing of the knees up to the chest 5. Stool mixed with blood and mucus

2, 4, 5 Intussusception is an obstructive gastrointestinal disorder caused when a segment of the bowel slides, or telescopes, into another section. This typically occurs in infants and children age <6. Once the bowel telescopes in, pressure increases within the bowel, causing ischemia and leakage of blood and mucus into the lumen of the bowel. Classic clinical manifestations of intussusception include episodes of sudden, crampy abdominal pain; a palpable sausage-shaped abdominal mass; and red, "currant jelly" stools (Options 2 and 5). Other manifestations include inconsolable crying with the knees drawn up to the chest and vomiting (Option 4). The child may appear normal and calm between painful episodes. (Options 1 and 3) A palpable, epigastric, olive-shaped mass and nonbloody projectile vomiting (ie, up to 3-4 feet [~1 meter]) are clinical manifestations often seen with pyloric stenosis. Projectile vomiting may also be a symptom of elevated intracranial pressure. However, intussusception typically causes bilious, nonprojectile vomiting and involves a sausage-shaped mass. Educational objective: Classic symptoms of intussusception include sudden, crampy abdominal pain; a palpable sausage-shaped abdominal mass; "currant jelly" stools; inconsolable crying with the knees drawn up to the chest; and bilious, nonprojectile vomiting. An olive-shaped mass is characteristic of pyloric stenosis. Projectile vomiting is frequently associated with pyloric stenosis or increased intracranial pressure. Physiological Adaptation

A nurse is performing an assessment of a 12-month-old child. Which of the following findings would the nurse expect? Select all that apply. 1. Approaches strangers with ease 2. Birth weight is tripled 3. Can skip and hop on one foot 4. Fully developed pincer grasp 5. Sits from a standing position

2, 4, 5 The first 12 months of life are characterized by rapid growth and development. By age 12 months, the child's birth weight should be about tripled (Option 2). A 12-month-old child should have mastered the gross motor skill of sitting down from a standing position without assistance (Option 5). The pincer grasp (ie, use of the thumb and forefingers to pick up objects) is an important fine motor skill that should also be fully developed by this age (Option 4). (Option 1) Stranger anxiety is well developed by age 8 months and continues into the toddler years. At age 12 months, the child typically prefers the parents and exhibits fear when separated. (Option 3) The gross motor skills of skipping and hopping on one foot do not usually occur until around age 4. Educational objective: Assessment findings of a 12-month-old child should include a weight that is approximately triple the birth weight. Expected motor skills include the ability to sit from a standing position without assistance and to use a fully developed pincer grasp. Health Promotion and Maintenance

The nurse is reinforcing education with the parents of a 2-year-old child about diet choices to promote growth. The family observes a strict vegan diet. Which of the following statements by the nurse are appropriate? Select all that apply. 1. "Diets consisting of legumes as the only protein source are sufficient for growth." 2. "It is important to feed your child fortified breads and cereals to help with iron intake." 3. "Preparing meals with vegetables and fruits will ensure sufficient vitamin B12 intake." 4. "Try to pair foods high in iron with foods high in vitamin C to aid iron absorption." 5. "Your child may require calcium and vitamin D supplementation due to lack of dairy intake."

2, 4, 5 With careful monitoring of nutritional intake, a vegan diet (ie, excluding all animal-derived products [eg, meat, dairy, eggs]) can be appropriate for clients in all age groups. Pediatric clients consuming a vegan diet are at increased risk for nutritional deficiencies (eg, protein, calories, calcium, vitamin D, iron, vitamin B12) due to rapid growth and development. Nurses educating clients about preventing nutritional deficiencies in vegan diets should include information about: - Iron: Plant sources of iron, which are in smaller quantities and difficult to absorb, should be supplemented with fortified cereals and breads to decrease risk of iron-deficient anemia (Option 2) - Vitamin C: Iron absorption is improved when dietary sources of iron and vitamin C are taken together (Option 4) - Calcium: Without animal sources of calcium (eg, dairy, eggs, fish), vegan diets require supplementation of calcium and vitamin D for bone health (Option 5) (Option 1) Many plant-based proteins (eg, legumes, grains) do not individually contain all the essential amino acids to support growth and tissue repair; therefore, vegan clients will require further teaching on combinations of protein sources. (Option 3) Fruits and vegetables do not provide vitamin B12. The nurse should educate the parents on the need for multivitamins or fortified grains as quality vitamin B12 sources. Educational objective: Pediatric clients consuming a vegan diet are at risk for dietary deficiencies (eg, iron, protein, calories, vitamin B12, calcium, vitamin D). Parent education about supplementation and adequate food sources of these nutrients is necessary. Health Promotion and Maintenance

The nurse is performing a physical assessment on a 2-year-old with cold symptoms and a fever at home of 101.7 F (38.7 C). The parent is concerned about the child's ability to cooperate during the examination. Place the components of assessment in the order the nurse would perform them. All options must be used. 1. Auscultate the child's heart and lungs 2. Interact with the parent in a friendly manner 3. Measure the child's height and weight 4. Play with the child using a finger puppet 5. Take the child's vital signs

2. Interact with the parent in a friendly manner 4. Play with the child using a finger puppet 3. Measure the child's height and weight 1. Auscultate the child's heart and lungs 5. Take the child's vital signs Always complete the assessment by performing the least invasive parts first and then progressing to the most invasive. By first establishing a rapport with the parent (Option 2), the nurse will elicit the child's trust and cooperation. Playing with the child will help the child relax and perceive the nurse as less of a threat (Option 4). Measuring the child's height and weight should be performed next (Option 3). Auscultation of the heart and lungs should then be performed. Allowing the child to play with the equipment first will make this part of the assessment easier (Option 1). Taking vital signs can be difficult as a blood pressure cuff can be perceived as painful (Option 5); once the child is upset, it becomes difficult to continue with the assessment. A temperature of 101.7 F (38.7 C) is not serious in a child, especially if there are signs and symptoms of an upper respiratory infection. Educational objective: Performing a physical assessment in a toddler can be challenging. The nurse should establish a rapport with the parent and then attempt to gain the child's trust. Playing with the child can make the experience easier on the nurse, parents, and child. The nurse should always perform the least invasive procedures first, explain them in simple terms, and praise the child throughout the assessment. Health Promotion and Maintenance

A 15-year-old parent brings a 4-month-old infant for a well-baby checkup. The parent tells the nurse that the baby cries all the time; the parent has tried everything to keep the infant quiet but nothing works. What is the priority nursing action? 1. Advise the parent to give a pacifier whenever the infant cries 2. Ask the parent to describe what is done to "keep the baby quiet" 3. Assess the infant's pattern and frequency of crying 4. Explore the parent's support system

3 During the first 3-4 months of life, it is not unusual for an infant to cry 1-3 hours a day in response to being hungry, thirsty, tired, in pain, bored, or lonely. A very young, first-time parent may not have an appreciable understanding of normal infant behavior and may perceive normal crying as excessive. It is most important for the nurse to assess the infant's pattern and quality of crying to better understand whether it is normal behavior or a sign of something more serious that requires further evaluation and treatment. The nurse needs to determine: - What "all the time" means - When the "all the time" crying started - What makes the crying worse and what makes it better - The quality of the crying (tone, pitch, loudness) - Length and quality of periods of silence (Option 1) A pacifier would be appropriate to calm and soothe this infant. However, the nurse needs to first assess the pattern and quality of the crying along with the methods the parent is already using. (Option 2) Finding out what the parent is already doing to comfort the child is part of the nursing assessment. In this case, however, it is more important to determine if the crying is normal or abnormal. (Option 4) Exploring the parent's support system is an appropriate nursing action to determine if the parent has anyone to turn to when frustrated in caring for the infant. However, it is not the most important assessment. Educational objective: When a parent tells the nurse that an infant cries "all the time," the priority nursing action is to assess the pattern, quality, and frequency of the child's crying. This will help the nurse determine if the crying is normal infant behavior or a sign of a more serious condition that requires further evaluation and treatment. Health Promotion and Maintenance

A 9-year-old has terminal cancer, but the parents do not want the child to know the prognosis. The child has been asking questions such as what dying is like and whether the child will die. Which action by the nurse is most appropriate? 1. Encourage the child to ask the parents these questions 2. Notify the health care provider (HCP) about the child's questions 3. Reassure the child that everyone is trying to help the child get better 4. Tell the parents about the child's questions

4 A 9-year-old's understanding of death is the same as that of an adult. The parents try to "protect" the child, but the child senses the truth at some level and wants to discuss it. A child may be aware of impending death even before being told. Not being told may make the child feel isolated. Children sometimes feel a need to "protect" parents because they fear that their understanding will burden them. The nurse can offer self or other appropriate people to talk to the child if the parents cannot do it themselves. However, the nurse should first discuss the child's concerns with the parents and not talk with the minor child on the nurse's own initiative. Discussing the child's questions about death would support the parents' autonomy and advocate for the child's needs. (Option 1) The child is probably hesitant to talk about death because the child senses the parents' reluctance to discuss it. It is worth considering why the child is asking others instead of the parents in the first place. It might be helpful for the nurse to bridge the communication gap initially. (Option 2) Although the HCP should be informed, the parents need to handle the situation with their minor child. (Option 3) This action minimizes the child's concerns and does not deal with the issue raised. The child is terminally ill. Educational objective: The nurse's role with a dying client is to aid communication. When a dying child asks about death, the parents should know about the child's concerns.

The clinic nurse is asked by the mother of a 15-month-old, "I am worried about my child's thumb sucking and its effects on tooth alignment. What should I do?" What is the nurse's best response? 1. "As long as your child's thumb sucking stops by age 2-3 years when all of the primary teeth have erupted, there is little concern." 2. "Because your child already has teeth, it is important to implement a plan to stop the thumb sucking as soon as possible." 3. "Newer research shows that thumb sucking has little effect on a child's teeth." 4. "The risk for misaligned teeth occurs when thumb sucking persists after eruption of permanent teeth."

4 Rooting and sucking are a part of an infant's natural reflexes. Nonnutritive sucking assists in helping the infant to feel secure. Some parents become very concerned about their infants sucking fingers, thumbs, or a pacifier and try to stop the behavior. As a rule, if thumb sucking stops before the permanent teeth begin to erupt, misalignment of the teeth and malocclusion can be avoided. Parents should be taught that teasing and punishing a child for using a pacifier or sucking the thumb is not an effective method for getting the child to stop. This can increase the child's anxiety and cause the child to increase the behavior. (Options 1, 2, and 3) These options are incorrect. Use of a pacifier or thumb sucking prior to eruption of the permanent teeth does not tend to cause dental issues such as teeth misalignment or malocclusion. Educational objective: The risk of teeth misalignment and malocclusion occurs when a child uses a pacifier or sucks the thumb after the eruption of the permanent teeth. Health Promotion and Maintenance

A 2-year-old child is brought to the emergency department for a severe sore throat and fever of 102.9 F (39.4 C). The nurse notes that the child is drooling with distressed respirations and inspiratory stridor. What action should the nurse take first? 1. Assess an accurate temperature with a rectal thermometer 2. Directly examine the throat for the presence of exudates 3. Obtain intravenous access for anticipated steroid administration 4. Position the child in tripod position on the parent's lap

4 This is a classic description of epiglottitis (supraglottitis). It is an inflammation by bacteria of the tissues surrounding the epiglottis, a long, narrow structure that closes off the glottis during swallowing. Edema can develop rapidly (as quickly as a few minutes) and obstruct the airway by occluding the trachea. There has been a 10-fold decrease in its incidence due to the widespread use of the Hib (Haemophilus influenzae type B) vaccine. The classic symptoms include a high-grade fever with toxic appearance, severe sore throat, and the 4 Ds—dysphonia (muffled voice), dysphagia (difficulty swallowing), drooling, and distressed respiratory effort. The tripod position opens the airway and helps air flow. The child should be allowed to assume a position of comfort (usually sitting rather than lying down). The priority nursing response is to protect the airway. (Option 1) No invasive procedure should be done that could cause the child to cry until the airway is secure. Knowing the temperature is not a priority. (Options 2 and 3) When drooling is present, the airway becomes the primary concern. No visual inspection, invasive procedure, or anxiety-provoking activity should be done until the airway is secure due to the risk of laryngospasm and respiratory arrest. Educational objective: Children with potential epiglottitis should be allowed a position of comfort without any invasive or anxiety-provoking procedures (eg, phlebotomy, pharyngeal examination, epiglottal cultures) until the airway is secure with intubation or a surgical airway.

The nurse provides teaching for the parents of a 6-year-old client diagnosed with nocturnal enuresis. Which of the following instructions will the nurse include? Select all that apply. 1. Allow the child to wear a diaper at bedtime to avoid accidents 2. Encourage the child to help change soiled pajamas and linens 3. Prepare a calendar with the child for logging wet and dry nights 4. Restrict the child's fluids to 8 oz (240 mL) with each meal 5. Wake the child at a specified time each night to void

Involuntary bedwetting at night in a child beyond the age of expected bladder continence is known as nocturnal enuresis. Certain medications (eg, oral desmopressin) may be useful in treating nocturnal enuresis; however, nonpharmacologic techniques should be attempted first. The nurse should educate the child's parents about the following strategies: - Limit the child's intake of caffeine and sugar - Instruct the child to void before going to bed - Avoid punishing, scolding, or ridiculing the child - Encourage the child to assist with changing soiled pajamas and linens, which helps them feel more control over the situation, but provide reassurance that it is not punishment (Option 2) - Use positive reinforcement to motivate the child (eg, calendar showing wet and dry nights, rewards) (Option 3) - Awaken the child nightly at a specified time to void (Option 5) - Use an enuresis alarm (ie, a moisture-sensitive alarm worn on sleep clothes that awakens the child when voiding occurs) (Option 1) The nurse should remind parents to avoid disposable training pants or diapers at bedtime because these discourage motivation to get up and void during the night. (Option 4) Rather than restrict fluids throughout the day, parents should restrict fluids only after the evening meal (ie, take small sips). Educational objective: Nocturnal enuresis, or involuntary bed-wetting at night, is managed with a variety of interventions. Initial strategies for addressing nocturnal enuresis should include positive reinforcement, awakening the child at a specified time to void, and encouraging the child to assist with wet linen changes.

The emergency department triage nurse is assessing 4 pediatric clients. Which client is a priority for further diagnostic workup and definitive care? 1. 1-year-old with ventriculoperitoneal shunt who has "lethargy" and pulse of 78/min 2. 3-year-old with history of meningocele who has unilateral ear pain and urinary incontinence 3. 6-year-old with muscular dystrophy who has "flu-like" symptoms and temperature of 100.4 F (38 C) 4. 8-year-old with history of cerebral palsy who has foot injury and spastic clonus

1 A ventriculoperitoneal shunt is used to treat hydrocephalus and is usually placed at age 3-4 months. Blockage and infection are complications of shunt placement. Blockage results in signs of increased intracranial pressure (ICP). The normal pulse range for a 1-year-old is 100-160/min. A pulse of 78/min is considered bradycardia, a part of Cushing's triad (bradycardia, slowed respiration, widened pulse pressure). (Option 2) Meningocele is a saclike protrusion through a bony defect that contains meninges and cerebrospinal fluid; it is corrected with surgery. In some children, residual bowel and bladder incontinence can result despite surgery. If bowel and bladder control is obtained but incontinence reoccurs, the child should be evaluated for infection (a common complication). Although the child with ear pain (eg, otitis media) may need antibiotics, this is not urgent and the client with neurological signs is the priority. (Option 3) Muscular dystrophy is an inherited condition of muscle fiber degeneration and muscle wasting. Respiratory and cardiac problems are the leading causes of mortality. These clients should take precautions to prevent respiratory infections (eg, pneumococcal and influenza vaccination, avoiding contact with infected individuals). This client is the second priority after the client with increased ICP and possible brain herniation. (Option 4) Cerebral palsy is a permanent disorder of movement, with abnormal muscle tone, lack of physical coordination, spasticity, and compromised posture development causing activity limitation. Clonus is a series of involuntary, rhythmic, muscular contractions and relaxations. Spasticity/clonus is an expected finding in a client with cerebral palsy. Foot injury in this client is not a priority. Educational objective: A ventriculoperitoneal shunt is used to treat hydrocephalus. Complications include blockage (with signs of increased intracranial pressure [ICP]) and infection. The normal pulse range for a 1-year-old is 100-160/min; bradycardia can be a sign of increased ICP.

The nurse is caring for a 4-year-old client with cystic fibrosis who uses a high-frequency chest wall oscillation (HFCWO) vest for chest physiotherapy. After reinforcing education with the client's parents, which statement by a parent requires further teaching? 1. "I will allow my child to have a snack while using the HFCWO vest to encourage cooperation." 2. "I will give my child the nebulized bronchodilator treatment during therapy with the HFCWO vest." 3. "I will perform manual chest percussion on my child if the HFCWO vest is broken or unavailable." 4. "My child will use the HFCWO vest once in the morning, once in the evening, and as needed."

1 Chest physiotherapy (CPT) describes techniques of airway clearance, which is an important component of treatment for clients with cystic fibrosis that loosens and drains thick respiratory secretions. CPT can be performed by percussing (ie, clapping) the chest with a cupped hand or by wearing an inflatable high-frequency chest wall oscillation (HFCWO) vest. The HFCWO vest inflates and deflates rapidly, causing vibration over the chest wall and mobilizing secretions into the large airways that the child can expectorate. The HFCWO vest's rapid vibrations may induce nausea and vomiting in some clients. Therefore, the client should avoid meals and snacks 1 hour before, during, or 2 hours following CPT to prevent gastrointestinal upset (Option 1). The nurse may suggest other more appropriate ways to ensure compliance with CPT, such as allowing the child to watch a favorite television show or reading the child a story while wearing the HFCWO vest. (Option 2) Nebulized bronchodilators are often given before or during CPT treatments to open the airways and mobilize secretions. (Option 3) CPT can be administered using various methods, including percussion (ie, clapping) of the chest wall with cupped hands. (Option 4) CPT should be performed at least twice a day, and more often if needed. Educational objective: Chest physiotherapy (CPT) is an important component of treatment for clients with cystic fibrosis. A high-frequency chest wall oscillation vest is a common method of performing CPT, but treatments should occur 1 hour before or 2 hours after eating to avoid gastrointestinal upset (eg, nausea, vomiting).

A child with autism spectrum disorder is being admitted to an acute care unit. Which is the most important nursing action? 1. Placing the child in a private room away from the nurses' station 2. Placing the child in a private room near the playroom 3. Placing the child in a semi-private room near the nurses' station 4. Placing the child in a semi-private room with another child with autism spectrum disorder

1 Children with autism spectrum disorder (ASD) often exhibit sensory processing problems; they may be hyper- or hypo-sensitive to sounds, lights, movement, touch, taste, and smells. A calming environment with minimal stimulation should be provided; a private room away from the nurses' station is the best location. The nurse can also facilitate a calming environment by: - Using a quiet or monotone voice when speaking to the child - Using eye contact and gestures carefully - Moving slowly - Limiting visual clutter - Maintaining minimal lighting - Providing the child with a single object to focus on (Option 2) A private room is an appropriate placement; however, the noise and activity from the playroom may be distracting to the child with ASD. (Option 3) A semi-private room near the nurses' station is likely to have a stimulating environment due to the noise, lighting, and work pace in the area. (Option 4) Placing the child in a semi-private room with another child with ASD does not promote a calming environment. Educational objective: Because children with autism spectrum disorder often exhibit sensory processing problems, they need a calming environment with minimal stimulation.

A newborn is being evaluated for possible esophageal atresia with tracheoesophageal fistula. Which finding is the nurse most likely to observe? 1. Choking and cyanosis during feeding 2. Concave (scaphoid) abdomen 3. Diminished lung sounds 4. Projectile vomiting after feeding

1 Esophageal atresia (EA) and tracheoesophageal fistula (TEF) consist of a variety of congenital malformations that occur when the esophagus and trachea do not properly separate or develop. In the most common form of EA/TEF, the upper esophagus ends in a blind pouch and the lower esophagus connects to the primary bronchus or the trachea through a small fistula. EA/TEF can usually be corrected with surgery. Clinical manifestations of EA/TEF include frothy saliva, coughing, choking, and drooling. Clients may also develop apnea and cyanosis during feeding (Option 1). Aspiration is the greatest risk for clients with EA/TEF, and newborns who demonstrate signs of the condition are immediately placed on nothing by mouth (NPO) status. (Option 2) A newborn with EA/TEF may have a distended abdomen due to the buildup of air in the stomach via the fistula from the trachea to the lower esophagus. A concave (ie, scaphoid) abdomen is associated with a congenital diaphragmatic hernia due to the migration of abdominal organs to the thoracic space. (Option 3) Diminished lung sounds are not an ordinary sign of EA/TEF unless aspiration pneumonia develops. These may be an indication of a diaphragmatic hernia or pneumothorax. (Option 4) A newborn with EA/TEF may experience apnea, choking, and cyanosis due to aspiration of fluid while eating. Projectile vomiting after feeding is a classic manifestation of hypertrophic pyloric stenosis. Educational objective: Clinical manifestations of EA/TEF include frothy saliva, coughing, choking, drooling, and a distended abdomen. Clients may also develop apnea and cyanosis while feeding. These findings must be reported to the health care provider for further evaluation.

The nurse is reviewing discharge instructions with the parents of a child who just had a tracheostomy. Which statement made by the parents indicates teaching has been effective? 1. "I will always travel with two tracheostomy tubes, one of the same size and one a size smaller." 2. "I will immediately change the tracheostomy tube if my child has difficulty breathing." 3. "I will provide deep suctioning frequently to prevent any airway obstruction." 4. "I will remove the humidifier if my child starts developing more secretions."

1 In the event of an accidental decannulation or another urgent need to change a tracheostomy tube, the most important action is to quickly replace the tube as it is the client's only means to ventilate. Clients should always carry two spare tracheostomy tubes, one the same size and one a size smaller. If the tube is not easily replaced or is meeting resistance, the smaller tube should be used. (Option 2) Changing a tracheostomy tube is a high-risk procedure that should be done only if respiratory distress is noted and other interventions (eg, suctioning) have failed. Mucus plugs (ie, thickening and buildup of mucus due to dehydration) are one of the most common causes of respiratory distress. (Option 3) A tracheostomy should be suctioned frequently to maintain airway patency. However, deep suctioning should be reserved for clients in respiratory distress due to the risk of injury. Tracheostomy tubes should be suctioned to the specified depth using a measurement marked on the tube, to provide safe, effective suctioning. (Option 4) Humidification is crucial for clients with a tracheostomy as the upper airway, which provides natural humidity for inhaled air, is bypassed. Humidification helps keep secretions thin and reduces formation of mucus plugs. The humidifier should not be removed if the child develops more secretions as this is the intended effect. Educational objective: Clients with a tracheostomy should always carry two spare tubes, one the same size and one a size smaller, to ensure that the tube can be replaced quickly and effectively.

The nurse is providing discharge instructions to the parent of a child with Kawasaki disease. The nurse informs the parent that the presence of which symptom should be immediately reported to the health care provider? 1. Fever 2. Irritability 3. Knee pain 4. Skin peeling

1 Kawasaki disease (KD) is a systemic vasculitis of childhood that presents with ≥5 days of fever, nonexudative conjunctivitis, lymphadenopathy, mucositis, hand and foot swelling, and a rash. First-line treatment consists of IV immunoglobulin and aspirin to prevent coronary artery aneurysms. When children with KD are discharged home, parents are instructed to monitor them for fever by checking the temperature (orally or rectally) every 6 hours for the first 48 hours following the last fever. Temperature should also be checked daily until the follow-up appointment. If the child develops a fever, the health care provider should be notified as this may indicate the acute phase of KD recurrence. The child may require additional treatment with IV immunoglobulin to prevent development of coronary artery aneurysms and occlusions. (Option 2) Irritability is a hallmark finding in a child with KD, especially during the acute phase (due to fever and inflammation). Parents should be advised that irritability can last up to 2 months. (Option 3) Temporary joint pain and other manifestations of arthritis (eg, stiffness, decreased range of motion) may occur and persist for several weeks. Parents should be informed that range of motion exercises and warm baths will help reduce these symptoms and minimize discomfort. (Option 4) Desquamation (skin peeling) of the hands and feet is an expected finding in KD. Parents should be informed that the peeling itself is not painful but that the new skin underneath may be red and sore. Educational objective: Once children with KD are discharged home, parents should be instructed to check their temperature every 6 hours for the first 48 hours following the last fever and then daily until the follow-up visit. The health care provider should be notified if the child has fever as this may indicate a need for further treatment.

The nurse is caring for a 2-year-old who is receiving a saline enema for treatment of intussusception. Reporting which client finding to the health care provider (HCP) is most important? 1. Passed a normal brown stool 2. Passed a stool mixed with blood 3. Stopped crying 4. Vomited a third time

1 Most cases of intussusception are successfully treated without surgery using hydrostatic (saline) or pneumatic (air) enema. The nurse will monitor for passage of normal brown stool, indicating reduction of intussusception. If this occurs, the HCP should be notified immediately to modify the plan of care and stop all plans for surgery. (Option 2) In intussusception, the stools are mixed with blood and mucus, giving a characteristic "currant jelly" appearance. This is an expected finding. (Option 3) Pain in intussusception is typically intermittent. It occurs every 15-20 minutes, along with screaming and drawing up of the knees. Therefore, if a child stops crying, it may not be due to reduction of intussusception. (Option 4) Intense pain causes spasms of the pyloric muscle that lead to vomiting after each episode. Vomiting tends to resolve once the intussusception is reduced. Educational objective: Reduction of intussusception is often performed with a saline or air enema. The HCP should be notified if there is passage of a normal stool as this indicates reduction of the intussusception. All plans for surgery should be stopped and the plan of care should be modified. Additional Information Pharmacological and Parenteral Therapies NCSBN Client Need

A nurse is reviewing the laboratory values for a 3-year-old client with nephrotic syndrome. The nurse interprets the results to most clearly reflect which physiologic process related to nephrotic syndrome? Click on the exhibit button for additional information. Labs: Serum albumin 2.0 g/dL (20 g/L) Serum total cholesterol275 mg/dL (7.1 mmol/L) Urinalysis, protein3+ 1. Glomerular injury 2. Hepatic impairment 3. Inherited hypercholesterolemia 4. Malnutrition

1 Nephrotic syndrome is a collection of symptoms resulting from various causes of glomerular injury. Below are the 4 classic manifestations of nephrotic syndrome: 1. Massive proteinuria - caused by increased glomerular permeability 2. Hypoalbuminemia - resulting from excess protein loss in the urine 3. Edema - specifically periorbital and peripheral edema and ascites; caused by low serum protein and albumin as fluid is pulled into interstitial spaces and body cavities 4. Hyperlipidemia - related to increased compensatory protein and lipid production by the liver Additional symptoms include decreased urine output, fatigue, pallor, and weight gain. The most common cause of nephrotic syndrome in children is minimal change nephrotic syndrome, which is generally considered idiopathic. Less common secondary causes may be related to systemic disease or infection, such as glomerulonephritis, drug toxicity, or acquired immunodeficiency syndrome. (Option 2) Ascites and edema are often associated with liver disease. However, these symptoms result from fluid shifts related to hypoalbuminemia in nephrotic syndrome. (Option 3) Lipid levels (normal total cholesterol <200 mg/dL [5.2 mmol/L]) can increase with nephrotic syndrome as the liver produces increased lipids and proteins to compensate for protein loss. (Option 4) Although low serum albumin (normal 3.5-5.0 g/dL [35-50 g/L]) could result from malnutrition, hypoalbuminemia in nephrotic syndrome is related to massive proteinuria (negative to trace protein on urinalysis is usually considered normal). Educational objective: Nephrotic syndrome is a collection of symptoms resulting from glomerular injury. The 4 characteristic manifestations are proteinuria, edema, hypoalbuminemia, and hyperlipidemia.

The nurse is providing care to a 9-year-old client who is awaiting surgery. Which intervention is developmentally appropriate for this client's plan of care? 1. Discuss the procedure with the client using simple diagrams with correct anatomical terminology 2. Explore the client's perception of how the surgery will positively affect their future 3. Focus primarily on the client's feelings and concerns regarding surgical scar appearance 4. Provide initial education about the procedure to the client immediately before it is performed

1 Planning care during a pediatric hospitalization requires the nurse to consider the child's stage of psychosocial and cognitive development. For the school-age child (age 6-12 years), developing a sense of industry (ie, confidence in skills and abilities) is a primary psychosocial need, and cognitive development is marked by concrete thinking (ie, based on actual objects or activities). During preprocedural education, the nurse should foster a sense of industry by involving the child in discussions about the procedure, interacting with the child directly, and using correct anatomical terminology. In addition, the use of simple diagrams helps to meet the child's need for concrete learning (Option 1). (Option 2) A school-age child has a concrete way of thinking and may not think abstractly about the future until adolescence. (Option 3) Adolescents are more aware of body image than school-age children. Exploring feelings and concerns regarding the appearance of a surgical scar is not a primary focus. (Option 4) Toddlers (age 1-3 years) should receive initial teaching about a procedure immediately before it occurs due to a limited concept of time. School-age children can be prepared in advance of a procedure (eg, a day before), which gives them time to process. Educational objective: During preprocedural education, the nurse should use developmentally appropriate methods of teaching. Using simple diagrams with correct anatomical terminology appropriately meets the psychosocial (ie, sense of industry) and cognitive needs (ie, concrete thinking) of school-aged children.

A nurse is planning to complete a physical examination of a toddler. Which approach is an appropriate intervention by the nurse? 1. Encourage the parent to be involved with the child 2. Engage in physical contact by removing the toddler's outer clothing first 3. Have medical equipment lying on a counter within view 4. Perform an examination in a head-to-toe order

1 The nurse should plan to assess the toddler client in a nonthreatening environment, taking time to develop rapport prior to beginning the examination. This can be achieved by talking to the toddler about favorite objects and slowly initiating contact. Parent involvement, such as holding the child and assisting the child with examination activities, reduces anxiety and encourages cooperation in toddler clients. Age-appropriate games or toys may be used if needed to gain the client's cooperation. (Option 2) Use minimal physical contact initially, and have the parent remove the outer clothing. (Option 3) Medical equipment may appear frightening to a toddler and should remain out of sight until needed. It may also be beneficial to allow the child to inspect and touch new pieces of equipment as they are used. (Option 4) It is best to order a physical examination for a toddler from least to most invasive, which commonly means assessing ears, nose, and mouth toward the end of a visit. Head-to-toe ordered assessments are more appropriate for school-age children. Educational objective: The nurse should allow a parent to interact with the toddler and assist with the examination process to encourage client cooperation. Examination of a toddler should proceed from least to most invasive, allowing the client to inspect pieces of equipment before use. Use minimal physical contact initially.

Which pediatric respiratory presentation in the emergency department is a priority for nursing care? 1. Client with an acute asthma exacerbation but no wheezing 2. Client with bronchiolitis with low-grade fever and wheezing 3. Client with runny nose with seal-like barking cough 4. Cystic fibrosis client with fever and yellow sputum

1 When an acute asthma exacerbation occurs, the child has rapid, labored respirations using accessory muscles. The child often appears tired due to the ongoing effort. In the case of severe obstruction (from airway narrowing as a result of bronchial constriction, airway swelling, and copious mucus), wheezing/breath sounds are not heard due to lack of airflow. This "silent chest" is an ominous sign and an emergency priority. In this situation, the onset of wheezing will be an improvement as it shows that air is now moving in the lungs. (Option 2) Bronchiolitis is associated with the respiratory syncytial virus (RSV). Cell debris clumps and clogs the airways. Air can get in but has difficulty getting out. Mild symptoms include low-grade fever, wheezing, tachypnea, and poor feeding; severe infections have more serious distress, including signs of hypoxia. Treatment is supportive. This child should be isolated and will receive supportive care, but the child with no air movement/wheezing is a priority. (Option 3) Croup or laryngotracheobronchitis is a viral inflammation and edema of the epiglottis and larynx. Symptoms include runny nose, tachypnea, inspiratory stridor, and a seal-like barking cough. The child will be treated with medications and oxygen (if needed). The child is still moving air. (Option 4) In clients with cystic fibrosis, fever with yellow or green sputum can be indicative of an infection. The child will receive antibiotics but is not a priority as there are no signs of respiratory distress. Educational objective: Inability to hear any breath sounds or wheezing in an acute asthma client ("silent chest") is an ominous sign and requires emergency intervention.

The nurse is reviewing the plan of care for a 4-year-old client who will receive daily dressing changes for an infected leg wound. Which of the following interventions should the nurse include in the plan of care for a preschool-age child? Select all that apply. 1. Allow the child's parents to stay during the procedure 2. Emphasize that dressing changes are not punishment for misbehavior 3. Encourage the child to voice questions and concerns about the procedure 4. Have the child place bandages on a doll when reinforcing education 5. Introduce the child to other clients with the same health condition

1, 2, 3, 4 Planning care during pediatric hospitalization requires the nurse to consider the child's stage of psychosocial and cognitive development. For the preschool-age child (3-5 years), developing a sense of initiative (ie, start and accomplish tasks, learn new things) is a primary psychosocial need, and cognitive development is marked by preoperational thinking (ie, egocentrism, poor causality, continuing language development). During preprocedural education, the nurse should: - Promote a sense of security and reduce fear by allowing the parents to stay with the child during the procedure (Option 1) - Address misconceptions related to preoperational thinking (ie, state that the procedure is not punishment for misbehavior) (Option 2) - Foster initiative by encouraging the child to ask questions, voice concerns, and participate during dressing changes (Option 3) - Enhance the child's learning ability and confirm the child's understanding of the procedure by allowing the child to imitate the procedure using a doll or toy equipment (Option 4) (Option 5) Peer support from other clients undergoing similar procedures is a coping technique that is more appropriate for adolescents. Educational objective: Interventions that meet the psychosocial (ie, sense of initiative) and cognitive (ie, preoperational thinking) needs of preschool-age children during preprocedural education include allowing parents to remain with the client during the procedure, emphasizing that the procedure is not punishment, encouraging the child to ask questions, and utilizing toys to communicate about the procedure.

A nurse is teaching the parents of an infant with tetralogy of Fallot. Which of the following actions should the nurse include to reduce the incidence of hypercyanotic spells? Select all that apply. 1. Encourage smaller, frequent feedings 2. Offer a pacifier when the infant begins to cry 3. Promote a quiet period upon waking in the morning 4. Swaddle the infant during procedures 5. Turn the infant frequently during sleep

1, 2, 3, 4 Tetralogy of Fallot is a complex heart defect that results in decreased pulmonary blood flow, mixing of oxygenated and unoxygenated blood, and inadequate blood flow into the left side of the heart. Hypercyanotic episodes (ie, "tet" spell) occur when unoxygenated blood enters the systemic circulation, resulting in cyanosis and hypoxemia. Tet spells usually occur during stressful or painful procedures; on waking; and with hunger, crying, and feeding. Home interventions to reduce the incidence of tet spells include: - Providing a calm environment, particularly on waking (Option 3) - Soothing and quieting the infant when crying or distressed - Offering a pacifier (Option 2) - Swaddling or holding the infant during procedures or times of stress (Option 4) - Providing frequent smaller feedings to reduce frustration due to hunger and limit sucking fatigue (Option 1) During an acute tet spell, the infant may be placed in the knee-chest position to improve pulmonary blood flow by increasing systemic vascular resistance; older children may assume a squatting position. Intermittent oxygen can also be used to treat the spell, if necessary. (Option 5) Tet spells occur more often during stressful situations or on waking, so sleep should not be interrupted whenever possible. Educational objective: Hypercyanotic or tet spells usually occur during stressful or painful procedures; on waking; and with hunger, crying, and feeding. Providing a calm environment; reducing hunger with small, frequent meals; and swaddling during procedures can help prevent hypercyanotic spells.

A 6-year-old client was diagnosed with type 1 diabetes mellitus 2 years ago. The nurse would like to encourage the client to participate in disease management. Which of the following diabetes care tasks are appropriate for the child to perform? Select all that apply. 1. Choose insulin injection site with parental oversight of rotation schedule 2. Push plunger of insulin syringe after a parent inserts and stabilizes the needle 3. Select and clean the site for finger-stick blood glucose testing 4. Use a chart to determine insulin dose based on glucometer reading 5. Verbalize two or three signs and symptoms of hypoglycemia

1, 2, 3, 5 The nurse should offer school-aged children (age 6-12) as much opportunity as possible to participate in care to promote psychosocial development (industry versus inferiority) and provide a sense of control. Parents should transfer management of care to the child in small steps based on the child's skill level and cognitive ability. School-aged children are in the concrete operational stage of development and are most successful performing simple, concrete tasks with a limited number of steps. Appropriate diabetes management tasks for school-aged children include: - Choosing and cleaning a finger for blood glucose testing before a parent or caregiver performs the puncture (Option 3) - Selecting the site for insulin injection, with a parent or caregiver verifying appropriate site rotation (Option 1) - Pushing the syringe plunger to administer insulin after a parent or caregiver inserts the needle (Option 2) - Identifying signs and symptoms of hypoglycemia and hyperglycemia (Option 5) (Option 4) Adjusting insulin doses based on glucose readings is too complicated for school-aged children, and mistakes can be life-threatening. Children develop the cognitive ability to analyze test results and adjust insulin doses at approximately age 14. Educational objective: Children should participate in their diabetes management. The school-aged child can choose and clean a finger for a blood glucose reading, select the insulin injection site, push the plunger to administer insulin after parents or caregivers insert the needle, and identify signs of hypoglycemia and hyperglycemia.

The nurse is reviewing anticipatory guidance with the parents of a 6-month-old infant with phenylketonuria. Which statements by the nurse are appropriate? Select all that apply. 1. "A low-phenylalanine diet is required." 2. "Meat and dairy products should not be introduced into the diet." 3. "Phenylketonuria is self-limiting and usually resolves by adulthood." 4. "Special infant formula is required." 5. "Tyrosine should be removed from the diet."

1, 2, 4 Phenylketonuria (PKU) is one of a few genetic inborn errors of metabolism. Individuals with PKU lack the enzyme (phenylalanine hydroxylase) required for converting the amino acid phenylalanine into the amino acid tyrosine. As unconverted phenylalanine accumulates, irreversible neurologic damage can occur. A low-phenylalanine diet is essential in the treatment of PKU (Option 1). Phenylalanine cannot be entirely eliminated from the diet as it is an essential amino acid and necessary for normal development. The diet must meet nutritional needs while maintaining phenylalanine levels within a safe range (2-6 mg/dL [120-360 µmol/L] for clients age <12). There is no known age at which the diet can be discontinued safely, and lifetime dietary restrictions are recommended for optimal health (Option 3). Management of the client with PKU includes: - Monitoring serum levels of phenylalanine - Including synthetic proteins and special formulas (eg, Lofenalac, Phenyl-Free) in the diet (Option 4) - Eliminating high-phenylalanine foods (eg, meats, eggs, milk) from the diet (Option 2) - Encouraging the consumption of natural foods low in phenylalanine (most fruits and vegetables) (Option 5) Restriction of dietary tyrosine is not necessary. Tyrosine levels in clients with PKU may be normal or slightly decreased. Educational objective: Phenylketonuria requires lifetime dietary restrictions. Infants should be given special formulas (eg, Lofenalac). For children and adults, high-phenylalanine foods (eg, meats, eggs, milk) should be restricted and replaced with protein substitutes.

The nurse on a pediatric unit is caring for a preschooler who exhibits separation anxiety when the parents go to work. Which interventions should the nurse implement? Select all that apply. 1. Encourage the parents to leave the child's favorite stuffed animal 2. Establish a daily schedule similar to the child's home routine 3. Give the child time to calm down alone when visibly upset 4. Provide frequent opportunities for play and activity 5. Remove visual reminders of the parents from the room

1, 2, 4 Some of the first stressors faced by children from infancy through the preschool years are related to illness and hospitalization. Separation anxiety, also known as anaclitic depression, particularly affects children age 6-30 months. There are 3 stages of separation anxiety: protest, when the child refuses attention from others, screams for the parent to return, and cries inconsolably; despair, when the child is withdrawn, quiet, uninterested in activities or meals, and displays younger behavior (eg, use of pacifier, wetting the bed); and detachment, when the child suddenly appears happy and interested in building relationships. Nursing care of hospitalized clients experiencing separation anxiety focuses on maintaining a calm environment and a supportive demeanor to build trust between the nurse and the child, and encouraging connection with family and familiar environments, even when they are absent. Key interventions include: - Encouraging the parents to leave favorite toys, books, and pictures from home - Establishing a daily schedule that is similar to the child's home routine - Maintaining a close, calming presence when the child is visibly upset - Facilitating phone or video calls when parents are available - Providing opportunities for the child to play and participate in activities (Option 3) When the child is visibly upset, it is important to provide a calming presence and implement strategies to reduce the child's anxiety. Leaving the child alone at such times can further increase stress. (Option 5) Providing pictures of the child's family is actually beneficial, as it reminds the child of something familiar and safe. Educational objective: Toddlers and preschool-age children experience separation anxiety in response to the stress of illness and hospitalization. Key nursing interventions to alleviate separation anxiety include encouraging the presence of favorite items, establishing a daily routine, providing opportunities for play, facilitating phone calls with the parents, and providing support when the child is upset.

The nurse is caring for a newborn who has a lumbar myelomeningocele with an intact sac. Which of the following nursing actions are appropriate? Select all that apply. 1. Avoids taking the newborn's temperature rectally 2. Covers the sac with a moist, sterile, nonadherent dressing 3. Ensures that the newborn's diaper is placed loosely across the sac 4. Frequently assesses the sac for leaks, infection, or irritation 5. Places the newborn in a prone position with the head turned to one side

1, 2, 4, 5 A myelomeningocele is a thin, membranous sac that contains meninges, spinal cord, nerve roots, and spinal fluid and develops on the outside of the body (eg, lumbar area) when the neural tube does not fuse correctly. Appropriate nursing actions include avoiding rectal temperature measurements, which is contraindicated in all newborns, especially if myelomeningocele is present; covering the sac with a moist, sterile dressing to prevent drying; assessing the sac for leaks, redness, and irritation; and positioning the newborn so that the sac does not rupture (ie, prone position) (Options 1, 2, 4, and 5). Diapers increase the risk for trauma to the sac and may be contraindicated until the defect is repaired and healing (Option 3).

The parents of a 2-year-old client ask how they can help their child cope with hospitalization. Which of the following suggestions should the nurse give the parents? Select all that apply. 1. Follow as many home routines as possible 2. Organize a visit from a playgroup friend 3. Sleep in the child's hospital room at night 4. Take child on regular visits to the playroom 5. Tell the child they did not cause the illness

1, 3, 4 Hospitalization for toddlers (ie, 12-36 months) is particularly difficult due to separation anxiety and a limited ability to cope with stress. Toddlers thrive on home rituals and routines, which bring stability and reassurance. Hospitalization can severely disrupt these routines, triggering frustration and temper tantrums. Caregivers should maintain as many home routines as possible (eg, sleeping, eating) to help the child cope with unfamiliar hospital surroundings and procedures (Option 1). Parents should also stay with the child as much as possible, including overnight (ie, rooming-in), to provide consistency and alleviate separation anxiety (Option 3). Play, an important part of a child's emotional and social well-being, is an effective coping mechanism for children of all ages to deal with the stress of being away from home. The playroom is a safe place for children to act out their fears and anxieties related to illness and hospitalization (Option 4). (Option 2) A visit from friends is not likely to provide much comfort to a toddler and may actually cause additional stress. Adolescents, who are driven by peer interaction, would be more likely to benefit from this strategy. (Option 5) Preschool-aged children (3-5 years) have egocentric and magical thinking, which may cause them to think that their illness is due to something they have done or thought. Toddlers do not think this way. Educational objective: Coping mechanisms used by hospitalized toddlers include following homes rituals and routines, having parents stay with the child (including overnight), and using the playroom for relief of anxiety and fear.

The nurse is planning care for an 8-year-old client with mild cognitive impairment who is hospitalized for diagnostic testing. Which of the following interventions are appropriate to include in the plan of care? Select all that apply. 1. Consistently assign the same nurse and unlicensed assistive personnel to care for the client 2. Give direct procedural education and explanations to the parent rather than the client 3. Provide appropriate toys based on developmental level rather than chronological age 4. Reinforce parental limit-setting measures for preventing self-injurious behavior 5. Use a picture board to facilitate communication and promote understanding of procedures

1, 3, 4, 5 Clients with cognitive impairment (intellectual disability) are diagnosed prior to adulthood and have limited levels of intellectual functioning and adaptive skills for their chronological age. Manifestations may include a decreased ability to perform abstract or logical reasoning, interpret complex ideas, and learn by experience. Cognitive impairment results in developmental delays of varying levels (eg, mild, moderate) and types (eg, cognitive, physical, social, emotional, behavioral) and requires the nurse to assess the client's skills and abilities and provide individualized care. Appropriate nursing interventions for a client with cognitive impairment include: - Promoting the staff's understanding of client behavior/needs and maintaining a familiar environment for the client by consistently assigning the same staff (eg, nurse) for care (Option 1) - Fostering playtime by providing toys that are developmentally appropriate, not necessarily age appropriate (Option 3) - Preventing self-injury by reinforcing the parents' limit-setting measures (eg, time-outs) and positively reinforcing good behavior (Option 4) - Facilitating communication and learning by using visual demonstration (eg, picture board) rather than complex explanations (Option 5) (Option 2) The nurse should involve parents in preprocedural education but avoid excluding the client; explaining procedures using methods appropriate for the client's cognitive ability is encouraged. Educational objective: Appropriate nursing interventions when caring for a pediatric client with cognitive impairment include providing consistency in staff assignments, providing toys appropriate for the client's developmental (not chronological) age, preventing self-injurious behavior (eg, reinforce parental limit setting), and using visual demonstration (eg, picture board) and simple explanations to facilitate communication and learning.

The school nurse is teaching a class of 10-year-old children about prevention of dental caries. Which recommendations would be part of the nurse's teaching plan? Select all that apply. 1. Chew sugar-free gum 2. Drink fruit drinks/juices instead of sugary, carbonated beverages 3. Include milk, yogurt, and cheese in dietary intake 4. Minimize consumption of sweet, sticky foods 5. Rinse mouth with water after meals when brushing is not possible

1, 3, 4, 5 Dental caries (ie, cavities) form when bacteria (eg, Streptococcus mutans) digest carbohydrates in the mouth, producing acids that break down tooth enamel and cause mineral loss. Oral hygiene and dietary intake are significant factors contributing to the development of caries. Clients should increase intake of cariostatic foods, which have an inhibitory effect on the progression of dental caries (eg, dairy products, whole grains, fruits and vegetables, sugar-free gum containing xylitol) (Options 1 and 3). Cariogenic foods increase the risk for cavities and should be avoided. These include refined, simple sugars; sweet, sticky foods such as dried fruit (eg, raisins) and candy; and sugary beverages (eg, colas and other carbonated beverages, fruit drinks/juices) (Option 4). Additional practices to prevent dental caries include: - Brushing after meals - Flossing at least twice a day - Rinsing the mouth with water after meals or snacks (Option 5) - Drinking tap water rather than bottled water (most tap water sources add fluoride to promote dental health, whereas most bottled water does not contain fluoride) - Finishing meals with a high-protein food (Option 2) Fruit drinks/juices contain high amounts of simple sugars; substituting these for other sugary beverages does not prevent dental caries. Whole fruits are better choices. Educational objective: Risk for dental caries can be reduced by avoiding highly cariogenic foods (eg, refined, simple sugars; sugary beverages; sweet, sticky foods), increasing intake of cariostatic foods (eg, dairy products, whole grains, fruits and vegetables), and maintaining oral hygiene (eg, brushing teeth, rinsing after meals).

The home health nurse is visiting an infant who recently had surgery to repair tetralogy of Fallot. Which of the following signs of heart failure should the nurse teach the parents to report to the health care provider? Select all that apply. 1. Cool extremities 2. Increase in appetite 3. Puffiness around the eyes 4. Reduction in number of wet diapers 5. Weight gain

1, 3, 4, 5 Following repair of tetralogy of Fallot, clients often develop chronic pulmonary regurgitation. Insufficient flow into the pulmonary vasculature causes the right ventricle to work harder, leading to right ventricular hypertrophy and a subsequent reduction in right ventricular function and cardiac output. The decrease in forward blood flow causes blood to back up into venous circulation, resulting in heart failure. Clinical manifestations of heart failure include: - Pale, cool extremities due to reduced perfusion to the systemic circulation (Option 1) - Periorbital edema (puffiness around the eyes) and rapid weight gain due to systemic venous congestion and fluid retention (Options 3 and 5) - Reduction in the number of wet diapers due to reduced perfusion to the kidneys (Option 4) (Option 2) Infants with heart failure generally have decreased appetite and feeding due to dyspnea and fatigue. Educational objective: Following repair of tetralogy of Fallot, clients often develop chronic pulmonary regurgitation, leading to right ventricular hypertrophy, decreased cardiac output, and heart failure. Clinical manifestations of heart failure include signs of poor perfusion (eg, pale, cool extremities; reduced urinary output) and fluid overload (eg, periorbital edema, rapid weight gain).

A home health nurse is managing care for an adolescent client with cystic fibrosis. Which of the following potential complications should the nurse consider when developing a nursing care plan? Select all that apply. 1. Chronic hypoxemia 2. Diabetes insipidus 3. Frequent respiratory infections 4. Obesity 5. Vitamin deficiencies

1, 3, 5 Cystic fibrosis (CF) is an inherited disorder (autosomal recessive) characterized by thickened secretions due to impaired chloride and sodium channel regulation that causes exocrine gland dysfunction. Management of a client with CF should primarily address potential complications related to the following body systems: > Pulmonary: Alterations in respiratory secretions (ie, thick sputum) make it difficult to clear the airway and can result in frequent respiratory infections and sinusitis (Option 3). Frequent infections and inflammation damage lung tissue and may lead to chronic hypoxemia (Option 1). > Gastrointestinal: Thickened secretions obstruct the release of pancreatic enzymes, causing malabsorption of fat-soluble vitamins (eg, A, E, D, K) and other nutritional deficiencies (Option 5). High-protein, high-calorie foods and supplemental enzymes with meals are necessary. > Reproductive: Thickened reproductive secretions (eg, seminal fluid, cervical mucus) or the absence of the vas deferens in men contributes to CF-related infertility. (Option 2) Diabetes mellitus, not diabetes insipidus, is a potential complication for clients with CF due to pathologic pancreatic changes (eg, fibrosis). (Option 4) Due to impaired gastrointestinal absorption, weight loss and failure to thrive are more common and a greater concern than obesity. Educational objective: Cystic fibrosis is an inherited disorder that results in impaired exocrine gland function and is characterized by thickened secretions that affect the pulmonary, gastrointestinal, and reproductive systems. When planning care, the nurse should monitor for priority concerns, including development of respiratory infections, chronic hypoxemia, nutritional deficiencies, and abnormal growth (failure to thrive).

A nurse is discussing the fine motor abilities of a 10-month-old infant with the infant's parent. Which are developmentally appropriate skills for an infant of this age? Select all that apply. 1. Grasps a small doll by the arm 2. Stacks 3 wooden blocks 3. Transfers small objects from hand to hand 4. Turns single pages in a book 5. Uses a basic pincer grasp

1, 3, 5 Fine motor skills of infants develop around the ability to grasp and pick up objects. By 3 months, infants will reflexively grasp a rattle placed in their hand. At 5 months, they are able to voluntarily clasp it with their palm. Around 7 months, infants are able to transfer an object from one hand to the other. By 8-10 months, infants have replaced the palmar grasp with a crude pincer grasp (use of thumb, index, and other fingers) to pick up round oat cereal and other finger foods. By 11 months, this develops into a neat pincer grasp (use of thumb and index finger). (Options 2 and 4) By 12 months, infants may attempt to turn multiple book pages at once, and they also begin attempts to stack 2 blocks. These skills require finer muscle control than is expected of a 10-month-old. Educational objective: Fine motor skills of infants develop around the ability to grasp objects. Voluntary grasping with the palm begins around 5 months, followed by the ability to transfer an object between hands by 7 months and the development of a crude pincer grasp (using the thumb, index, and other fingers) around 8-10 months.

A nurse is speaking with the parent of a toddler who believes the child has a hearing deficit. Which findings support this suspected diagnosis? Select all that apply. 1. Behavior appears withdrawn 2. Intelligible speech began at age 12 months 3. Monotone speech 4. Seems attentive, nods, and smiles when given directions 5. Speaks with a loud voice

1, 3, 5 Hearing impairment in children may be related to family history, an infection, use of certain medications, or a congenital disorder. Toddlers with hearing deficits may appear shy, timid, or withdrawn, often avoiding social interaction. They may seem extremely inattentive when given directions and appear "dreamy." Speech is usually monotone, difficult to understand, and loud. Increased use of gestures and facial expressions is also common. (Option 2) Children typically begin to use well-formed syllables such as "mama" and "dada" by approximately age 7 months. A referral for a hearing test should be made if there is an absence of well-formed syllables by age 11 months or intelligible speech is not present by 24 months. (Option 4) Lack of attentiveness and appropriate response when given a direction is characteristic of a toddler who has a hearing impairment. Educational objective: Hearing impairment in infants delays development of intelligible speech. As these infants become toddlers, they often have a loud voice and monotone speech that is difficult to understand. They appear shy, timid, and inattentive.

The nurse is caring for a 7-month-old client during a well-child visit. Which of the following gross motor skills should the nurse expect to identify at this age? Select all that apply. 1. Bears full weight on feet with support 2. Moves from lying down to a sitting position 3. Pulls up into a standing position from sitting 4. Sits using hands for extra support 5. Walks while holding on to furniture

1, 4 Childhood development usually occurs in an orderly and predictable manner, with more complex skills being acquired as age increases. Fine (eg, grasp) and gross (eg, posture, balance, movement) motor skills are assessed during routine well-child visits to identify normal development and detect delays. During infancy, gross motor development begins with head and neck control and progresses to skills such as turning over, bearing weight on the arms in a prone position, sitting with the head erect, standing, crawling (ie, abdomen touching floor), creeping (ie, abdomen lifted off floor), and walking. By age 7 months, infants should be able to bear their full weight while standing with caregiver support and sit with minimal support from their hands (ie, tripod sitting) (Options 1 and 4). (Option 2) By age 7 months, infants can roll over, but the ability to move from a prone to a sitting position is not expected until age 10 months. (Option 3) Some infants learn to pull themselves up into a standing position early, but this is not expected until age 9-10 months. (Option 5) Walking while holding on to furniture is not expected until age 11 months. Educational objective: Childhood development of gross motor skills usually follows a predictable pattern, with more complex skills being acquired as age increases. A 7-month-old client should be able to sit with minimal support and bear their full weight while standing with caregiver support.

The nurse assessing a 2-year-old should expect the child to be able to perform which actions? Select all that apply. 1. Build a tower with blocks 2. Draw a square 3. Hop on one foot 4. Say own name 5. Walk without help

1, 4, 5 Nurses play an important role in identifying appropriate growth and development in all clients. Children who do not meet key developmental milestones for their age should be reported to the health care provider (HCP) to determine the need for further testing. Developmental milestones that a 2-year-old toddler should meet include: - Motor skills: Walks alone, builds block towers, draws lines, kicks a ball - Language: Knows 300+ words, uses 2- to 3-word phrases, states name - Cognitive/social skills: Engages in parallel play, imitates others, exerts independence (Option 2) Normally, a child will develop the ability to draw or copy a square later during the preschool years (age 3-5). (Option 3) A 2-year-old client will not yet demonstrate the balance required for this activity. The ability to hop and stand on one foot for 5-10 seconds develops during the preschool years (age 3-6). Educational objective: Developmental assessment findings in 2-year-old clients include the ability to build block towers, say their own name, and walk without assistance. The nurse should notify the HCP if a child is not meeting age-appropriate developmental milestones so the child can be referred for further testing. Health Promotion and Maintenance

The nurse is assisting with an education conference for graduate nurses about infant CPR. Which of the following statements are appropriate to include in the teaching? Select all that apply. 1. "A single rescuer responding to an unwitnessed infant arrest should perform 2 minutes of CPR before retrieving a defibrillator." 2. "Depth of chest compressions for infants should be half the depth of the anterior-posterior chest diameter." 3. "Rescuers should place the heel of one hand on the lower sternum when delivering chest compressions to infants." 4. "The ratio of chest compressions to breaths during CPR by a single rescuer is 15:2 for infants." 5. "You should assess the infant's brachial pulse for no longer than 10 seconds."

1, 5 Although rare, cardiac arrest in infants can occur and usually stems from a respiratory etiology. The American Heart Association provides guidelines for basic life support of infants (<12 months), including certain CPR modifications (eg, the location of pulse check) and the timing of emergency services notification and retrieval of the automatic external defibrillator (AED). The rescuer should check the infant's brachial pulse for no longer than 10 seconds (Option 5). During an unwitnessed collapse, a single rescuer should shout for nearby help, activate the emergency response system (eg, call emergency services via mobile device if located outside a health care setting), and then provide approximately 2 minutes of CPR at a rate of at least 100 compressions/min before retrieving the AED (Option 1). (Option 2) The rescuer should deliver chest compressions to an infant at a depth equal to one-third of the chest's anterior-posterior diameter (ie, ~1.5 in [4 cm]) and allow for recoil between compressions. (Option 3) The rescuer should perform infant chest compressions using either two fingers or two thumbs on the sternum just below the nipple line. (Option 4) Single rescuers performing infant CPR should use a 30:2 compression-to-breath ratio. A compression-to-breath ratio of 15:2 is used when two rescuers are involved. Educational objective: The American Heart Association provides guidelines for basic life support of infants, including initial client evaluation (eg, assess brachial pulse) and retrieval of automatic external defibrillator (ie, after 2 min of CPR during an unwitnessed collapse with a single rescuer).

A 7-month-old infant is admitted to the unit with suspected bacterial meningitis after receiving an initial dose of antibiotics in the emergency department. Frequent assessment of which of the following is most important in the plan of care? 1. Babinski reflex 2. Fontanel assessment 3. Pulse pressure 4. Pupillary light response

2 Bacterial meningitis is inflammation of the meninges of the brain and spinal cord caused by infection. General manifestations in infants and children age <2 include fever, restlessness, and a high-pitched cry. One common acute complication of bacterial meningitis is hydrocephalus, an increase in intracranial pressure (ICP) resulting from obstruction of cerebrospinal fluid flow. Increased ICP can progress to permanent hearing loss, learning disabilities, and brain damage. Bulging/tense fontanels and increasing head circumference are important early indicators of increased ICP in children. Frequent assessment for developing complications is vital for any client with suspected bacterial meningitis. (Option 1) The Babinski reflex can be present up to age 1-2 years and is a normal, expected finding; it does not indicate meningitis. (Option 3) Pulse pressure is the difference between systolic and diastolic blood pressures. Widening of pulse pressure is one of the signs of Cushing's triad (systolic hypertension with widened pulse pressure, bradycardia, respiratory depression). These signs occur very late if increased ICP is not treated. Fontanel assessment provides an earlier indication of increased ICP. (Option 4) Because meningitis clients are sensitive to light (photophobia), frequent assessment of pupillary light response will be uncomfortable. Severely increased ICP may alter pupillary response; however, this is a late complication of hydrocephalus. Fontanel assessment provides an earlier indication of a developing problem. Educational objective: Infants with bacterial meningitis can develop hydrocephalus. Bulging/tense fontanels and increasing head circumference are important early indicators of increased ICP in children and should be monitored to prevent long-term complications.

The registered nurse is teaching the parent of a 6-year-old about behavioral strategies for treating fecal incontinence due to functional constipation. Which statement by the parent indicates a need for further teaching? 1. "I will give my child a picture book to look at during toilet time." 2. "I will give my child a reward for each bowel movement while sitting on the toilet." 3. "I will keep a log of my child's bowel movements, laxative use, and episodes of soiling." 4. "I will schedule regular toilet sitting time for my child."

2 Fecal incontinence (ie, encopresis, soiling) refers to the repeated passage of stool in inappropriate places by children age ≥4 years. In more than 80% of cases, it is due to functional constipation (retentive type); in about 20% of cases, it may be caused by psychosocial triggers (nonretentive type). Management of fecal incontinence/constipation primarily includes 3 components: Disimpaction followed by prolonged laxative therapy, dietary changes (increased fiber and fluid intake), and behavior modification. Behavioral strategies are used to promote and restore regular toileting habits and to gain the child's cooperation and participation in the treatment program. Behavioral interventions include the following: - Regularly schedule toilet sitting times 5-10 minutes after meals for 10-15 minutes (Option 4) - Provide a quiet activity for the child during toilet sitting, which will help pass the time and make the experience more "enjoyable" (Option 1) - Initiate a reward system to boost the child's participation in the treatment program; the reward would be given for effort, not for success of evacuation in the toilet (children with retentive encopresis have dysfunctional anal sphincters and little control over bowel movements; giving a reward for something the child has no control over would not be effective) (Option 2) - Keep a diary or log of toilet sitting times, stooling, medications, and episodes of soiling to evaluate the success of the treatment (Option 3) Educational objective: A reward system is one of the behavioral strategies used in the treatment of functional incontinence (due to constipation). The reward is given to encourage the child's involvement in the treatment to restore normal bowel function. Rewards are given for the child's effort and participation, not for having bowel movements while sitting on the toilet.

The clinic nurse performs assessments on four infants. The nurse should alert the health care provider to see which client first? 1. 3-month-old whose posterior occiput appears flattened 2. 4-month-old who has sclera visible above the iris (sunset eyes) 3. 6-month-old who has vomited twice and has had 8 wet diapers in the last 24 hours 4. 9-month-old whose toes fan out and big toe dorsiflexes when plantar surface is stroked

2 Hydrocephalus is an increase in intracranial pressure (ICP) that results from obstruction of cerebrospinal fluid flow. Increased ICP can progress to brain damage and death. Signs of increased ICP in children include bulging fontanelles, increasing head circumference, and sunset eyes (or setting-sun sign) (sclera visible above the iris). Sunset eyes occur when periaqueductal structures are compressed from increased ICP, paralyzing the upward gaze. This is a late sign of increased ICP that requires timely treatment (eg, shunt placement) and is the priority (Option 2). (Option 1) Positional plagiocephaly (flat head syndrome) occurs when an infant is placed in the same position (eg, supine) for an extended period of time and the pliable skull molds to the surface (flattens). Parents can intervene to avoid or correct plagiocephaly (eg, periodically repositioning the head during sleep, tummy time). Minor skull deformation is not a priority. (Option 3) Eight wet diapers in 24 hours is within the normal range (6-10 diapers/day or approximately 1 diaper every 4 hours), indicating that the infant is likely producing >1 mL/kg/hr urine output and is not dehydrated, despite vomiting. (Option 4) The Babinski reflex (ie, toes fan outward and the big toe dorsiflexes with stimuli) is expected in infants and is a normal finding up to age 1 year. However, its presence beyond this age can indicate neurologic disease. Educational objective: The presence of sunset eyes (sclera visible above the iris) is a late sign of increased intracranial pressure and a priority to report to the health care provider.

An overweight toddler is diagnosed with iron deficiency anemia. Which is the most likely explanation for the anemia? 1. Excessive intake of meat products 2. Excessive intake of milk 3. Gastrointestinal blood loss 4. Impaired iron transfer from the mother

2 Iron deficiency anemia is the most common chronic nutritional disorder in children. There are many risk factors for iron deficiency, including insufficient dietary intake, premature birth, delayed introduction of solid food, and consumption of cow's milk before age 1 year. One common cause in toddlers is excessive milk intake, over 24 oz/day. In addition to becoming overweight, toddlers who consume too much milk develop iron deficiency due to the likely exclusion of iron-rich foods in favor of milk, a poor source of available iron. Treatment of iron deficiency anemia includes oral iron supplementation and increased consumption of iron-rich foods (eg, leafy green vegetables, red meats, poultry, dried fruit, fortified cereal). It is also important to limit milk intake (16-24 oz/day) in toddlers to ensure a balanced diet. (Option 1) Red meat and other meat products are considered good sources of dietary iron. However, clients may be at risk for obesity if meat consumption exceeds protein and caloric needs. (Option 3) Gastrointestinal blood loss, which can occur if infants under age 1 year are fed cow's milk, is a potential cause of iron deficiency anemia. However, excessive milk intake is a more common cause, particularly in clients over age 1 year. (Option 4) Impaired or decreased iron transfer is a potential cause of iron deficiency anemia, particularly in preterm infants or infants born in multiples. However, iron stores received from the mother are typically depleted by age 5-6 months (2-3 months for preterm infants); after this point, iron must be acquired through dietary sources. Because this client is a toddler (age 1-3 years), impaired iron transfer is not a likely cause of the current anemia. Educational objective: Iron deficiency anemia is the most common nutritional disorder in children. Risk factors include premature birth, cow's milk before age 1 year, and excessive milk intake in toddlers. Prevention and treatment are achieved through proper nutrition (eg, meat, leafy green vegetables, fortified cereal) and supplementation.

The nurse is planning care for a child being admitted with Kawasaki disease and should give priority to which nursing intervention? 1. Apply cool compresses to the skin of the hands and feet 2. Monitor for a gallop heart rhythm and decreased urine output 3. Prepare a quiet, non-stimulating, and restful environment 4. Provide soft foods and liberal amounts of clear liquids

2 Kawasaki disease (KD) is a childhood condition that causes inflammation of arterial walls (vasculitis). The coronary arteries are affected in KD, and some children develop coronary aneurysms. The etiology of KD is unknown; there are no diagnostic tests to confirm the disease, and it is not contagious. KD has 3 phases: 1. Acute - sudden onset of high fever that does not respond to antibiotics or antipyretics. The child becomes very irritable and develops swollen red feet and hands. The lips become swollen and cracked, and the tongue can also become red (strawberry tongue). 2. Subacute - skin begins to peel from the hands and feet. The child remains very irritable. 3. Convalescent - symptoms disappear slowly. The child's temperament returns to normal. Initial treatment consists of IV gamma globulin (IVIG) and aspirin. IVIG creates high plasma oncotic pressure, and signs of fluid overload and pulmonary edema develop if it is given in large quantities. Therefore, the child should be monitored for symptoms of heart failure (eg, decreased urinary output, additional heart sounds, tachycardia, difficulty breathing). (Option 1) During the acute phase (swollen hands and feet), skin discomfort can be eased with cool compresses and lotions. No treatment is needed in the subacute phase (skin peeling), but the new skin might be very tender. (Option 3) The child will be very irritable during the acute phase of KD. A non-stimulating, quiet environment will help to promote rest. After a KD episode, it is important for parents to understand that their child's irritability may last for up to 2 months and that follow-up appointments for cardiac evaluation are important. (Option 4) During the acute phase (painful swollen lips and tongue), the child should be given soft foods and clear liquids as these are tolerated best. Educational objective: Kawasaki disease causes inflammation of the arterial walls and can lead to scarring of the coronary arteries or development of coronary aneurysms. Treatment consists of aspirin and substantial infusion of IV gamma globulin. The affected child must be monitored for signs of heart failure.

A newborn has a large myelomeningocele. What nursing intervention is priority? 1. Assess the anus for muscle tone 2. Cover the area with a sterile, moist dressing 3. Measure the occipital frontal circumference 4. Place the newborn supine with the head of the bed elevated

2 Myelomeningocele occurs when the neural tube fails to fuse properly during fetal development. An outpouching of spinal fluid, spinal cord, and nerves covered by only a thin membrane occurs, typically in the lumbar area. The newborn is at high risk for infection at this area. A priority nursing intervention is to cover the area with a sterile, moist dressing to decrease the risk of infection until surgical repair can occur. (Option 1) Assessing for an anal wink will assist in the assessment of the level of neurologic deficit but is not a priority intervention. (Option 3) Myelomeningocele may decrease the absorption of cerebrospinal fluid, which would place the newborn at risk for hydrocephalus from the excess cerebrospinal fluid. An occipital frontal circumference is needed as a baseline measurement but is not a priority. (Option 4) The newborn would be placed in the prone position (with face turned to the side) to prevent rupture of the myelomeningocele. Educational objective: The newborn with a myelomeningocele is at risk for infection. Covering the myelomeningocele with a sterile, moist dressing is indicated to decrease the risk of infection at the site. The infant should be placed on the abdomen (prone) with the face turned to the side.

The nurse is caring for a newborn with patent ductus arteriosus. Which assessment finding should the nurse expect? 1. Harsh systolic murmur 2. Loud machine-like murmur 3. Soft diastolic murmur 4. Systolic ejection murmur

2 Patent ductus arteriosus (PDA) is an acyanotic congenital defect more common in premature infants. When fetal circulation changes to pulmonary circulation outside the womb, the ductus arteriosus should close spontaneously. This closure is caused by increased oxygenation after birth. If a PDA is present, blood will shunt from the aorta back to the pulmonary arteries via the opened ductus arteriosus. Many newborns are asymptomatic except for a loud, machine-like systolic and diastolic murmur. The PDA will be treated with surgical ligation or IV indomethacin to stimulate duct closure. (Option 1) A harsh systolic murmur is heard in the setting of ventricular septal defect, an opening between the ventricles of the heart. Ventricular septal defect is an acyanotic defect. (Option 3) A diastolic murmur is heard in mitral stenosis and aortic regurgitation but not in PDA. (Option 4) A systolic ejection murmur is heard in pulmonic stenosis. Right ventricular hypertrophy will develop if this defect is not repaired. In adults, systolic ejection murmur is usually due to aortic stenosis. Educational objective: The ductus arteriosus of a newborn should close spontaneously when fetal circulation changes to pulmonary circulation. If the ductus arteriosus remains open, blood will shunt from the aorta to the pulmonary arteries. The child will be acyanotic but will have a machine-like murmur heard on both systole and diastole.

The parents of a hospitalized preschooler are concerned because their toilet-trained child has started wetting the bed. Which response by the nurse is most helpful? 1. "Discipline your child by taking away playroom privileges." 2. "It is normal for your child to regress while hospitalized." 3. "Restricting fluids at nighttime will solve this problem." 4. "Your child is acting out due to the hospitalization."

2 Regression during hospitalization is a normal response to the stress of an unfamiliar environment, the fear and pain of invasive procedures, and the change in a child's normal routine. Toilet-trained children may start bed-wetting, and children who gave up the bottle or pacifier may ask for it. It is important for the nurse to explain that this behavior is completely normal and that the child will gain back previous milestones after discharge. (Option 1) Firm discipline would be counterproductive at this time. Punishment by restricting playtime would create more stress for the child. (Option 3) Limiting fluids at nighttime, voiding before bedtime, and involving the child in planning (eg, changing wet linens) are all appropriate interventions for enuresis. However, the first step is to reassure the parents and then teach them therapeutic interventions. (Option 4) Misbehaving is not an unusual behavior for a preschooler. Acting out would not be due exclusively to the hospitalization. Educational objective: Hospitalization can be very stressful for a child. Regressive behaviors during hospitalization are a normal response to changes in routine. The nurse should inform the caregivers that this behavior is temporary and that the child will regain lost milestones rapidly after discharge.

The triage nurse is assessing an unvaccinated 4-month-old infant for fever, irritability, and open-mouthed drooling. After the infant is successfully treated for epiglottitis, the parents wonder how this could have been avoided. Which response by the nurse would be most appropriate? 1. "It's impossible to know for sure what could have caused this episode." 2. "Most cases of epiglottitis are preventable by standard immunizations." 3. "We are still waiting for the formal report from the microbiology laboratory." 4. "There is nothing you could have done; the important thing is that your child is safe now."

2 The majority of cases of epiglottitis are caused by Haemophilus influenza type B (HiB), which is covered under the standard vaccinations given during the 2- and 4-month visits. Epiglottitis is rarely seen in vaccinated children. (Option 1) This statement is technically true, but it is not helpful to the parents and misses a critical teaching moment for them. (Option 3) It is reasonable to attribute the cause of the infant's epiglottitis to missing the vaccinations for Haemophilus influenza type B. (Option 4) This statement is both unhelpful and inaccurate as the child is still at risk for further preventable illness. Educational objective: Cases of epiglottitis are preventable, and parents should always be educated on the risks of foregoing vaccinations for their children.

A client diagnosed with acute glomerulonephritis has pitting edema in both lower extremities, blood pressure of 170/80 mm Hg, and proteinuria. When developing a plan of care for this client, the nurse should include which most accurate indicator of fluid loss or gain? 1. Blood pressure measurements 2. Daily weight measurements 3. Intake and output measurements 4. Severity of pitting edema

2 The most accurate indicator of fluid loss or gain in an acutely ill client is weight, as accurate intake and output and assessment of insensible losses may be difficult (Option 3). A 2.2-lb (1-kg) weight gain is equal to 1,000 mL of retained fluid. (Option 1) Blood pressure measures the amount of pressure exerted on the arterial walls due to factors such as peripheral artery constriction or dilation, not just fluid volume status. (Option 4) Pitting edema is not an accurate indicator as the fluid may shift from intravascular to interstitial spaces without an overall change in fluid gain or loss throughout the body. Educational objective: The most accurate indicator of fluid loss or gain in an acutely ill client is daily weight.

A nurse is assessing a 1-month-old infant with an atrial septal defect (ASD). Which assessment finding does the nurse expect? 1. Muffled heart tones 2. Murmur 3. Cyanosis 4. Weak femoral pulses

2 The nurse would expect to hear a murmur with an atrial septal defect. This defect is an abnormal opening between the right and left atria, allowing blood from the higher pressure left atrium to flow into the lower pressure right atrium. The back-and-forth flow of blood between the 2 chambers causes a vibration that is heard as a murmur on auscultation. ASD has a characteristic systolic murmur with a fixed split second heart sound. Some clients may also have a diastolic murmur. (Option 1) Muffled heart tones are not typical in ASD. Muffled heart tones that are heard postsurgical intervention are concerning for cardiac tamponade. (Option 3) Atrial and ventricular septal defects are acyanotic congenital heart defects because the blood from the high pressure left side (oxygenated blood) goes to the low pressure right side. (Option 4) Weak lower and strong upper extremity pulses are present in coarctation of the aorta. Educational objective: In a child with atrial septal defect, the nurse would expect to hear a heart murmur on auscultation of heart sounds.

The nurse has received report on 4 pediatric clients on a telemetry unit. Which client should the nurse assess first? 1. Adolescent client with coarctation of the aorta and diminished femoral pulses 2. Infant client with ventricular septal defect with reported grunting during feeding 3. Newborn client with patent ductus arteriosus and a loud machinery-like systolic murmur 4. Preschool client with tetralogy of Fallot who has finger clubbing and irritability

2 Ventricular septal defect (VSD) is a congenital abnormality in which a septal opening between ventricles causes left-to-right shunting, leading to excess blood flow to the lungs. This places the client at risk for congestive heart failure (CHF) and pulmonary hypertension. Clinical manifestations of VSD include a systolic murmur auscultated near the sternal border at the third or fourth intercostal spaces, and hallmark CHF signs (eg, diaphoresis, tachypnea, dyspnea). The client is currently showing signs of increased respiratory exertion (eg, grunting) and requires further assessment for CHF (Option 2). (Option 1) Coarctation of the aorta (COA) is an abnormal aortic narrowing that results in decreased cardiac output. The client will exhibit elevated pulse pressure in the upper extremities and diminished pressures in the lower extremities. Further assessment is needed, but this client is not the current priority. (Option 3) A systolic murmur with a machine sound and poor feeding are expected, nonurgent findings in clients with patent ductus arteriosus (PDA). PDA commonly resolves within 48 hours and requires no intervention in full-term newborns. (Option 4) Tetralogy of Fallot (TOF) is a cyanotic congenital heart defect commonly manifested by signs of irritability and clubbing of fingers due to oxygen saturation chronically remaining between 65-85% until the client can undergo surgical repair. Further evaluation of the client's oxygenation is necessary but not urgently required. Educational objective: Ventricular septal defect is a cardiac abnormality, with a septal opening between ventricles, that may progress to congestive heart failure (CHF). The client should be closely monitored for respiratory exertion and signs of CHF (eg, dyspnea, tachypnea).

What socioeconomic indicators would the nurse identify as risk factors for a 2-month-old infant to develop failure to thrive (FTT)? Select all that apply. 1. Both caregivers work outside the home 2. Infant lives only with mother, who is currently unemployed 3. Infant's primary caregiver has cognitive disabilities 4. Parents are socially and emotionally isolated 5. Parents live together but are not married

2, 3, 4 FTT, or growth failure, is a state of undernutrition and inadequate growth in infants and young children. Most cases of FTT are related to an inadequate intake of calories, which can be tied to many different etiologies. Physiologic risk factors for FTT include preterm birth, breastfeeding difficulties, gastroesophageal reflux, and cleft palate. Socioeconomic risk factors include: - Poverty - most common - Social or emotional isolation - parents may lack the support system needed to assist them with the problems of child rearing - Cognitive disability or mental health disorder - Lack of nutritional education - parents may not have knowledge of proper feeding techniques or appropriate calorie intake based on age and size of the child (Option 1) There is no known relationship between caregivers working outside the home and FTT. Caregivers who are fully employed may be more able to provide adequate food resources. (Option 5) There is no indication that unmarried parents pose a higher risk for an infant to develop FTT. More important protective factors include having a stable environment and living with 2 parents. Educational objective: FTT is a state of undernutrition and inadequate growth found in infants and young children. Physiologic risk factors for FTT include preterm birth, breastfeeding difficulties, gastroesophageal reflux, and cleft palate. Socioeconomic risk factors include poverty, social or emotional isolation, caregivers with cognitive disabilities or mental health disorders, and lack of nutritional education. Health Promotion and Maintenance

The nurse has provided teaching about home care to the parent of a 10-year-old with cystic fibrosis. Which of the following statements by the parent indicates that teaching has been effective? Select all that apply. 1. "Chest physiotherapy is administered only if respiratory symptoms worsen." 2. "I will give my child pancreatic enzymes with all meals and snacks." 3. "I will increase my child's salt intake during hot weather." 4. "Our child will need a high-carbohydrate, high-protein diet." 5. "We will limit our child's participation in sports activities."

2, 3, 4 In clients with cystic fibrosis (CF), a defective protein responsible for transporting sodium and chloride causes exocrine gland secretions to be thicker and stickier than normal. Viscous respiratory secretions accumulate, resulting in impaired airway clearance and a chronic cough. Clients eventually develop chronic lung disease, which predisposes them to recurrent respiratory infections. Pancreatic enzyme secretion, needed for digestion and absorption of nutrients, is also impaired because thick secretions block pancreatic ducts. Therefore, the client needs supplemental enzymes with all meals and snacks (Option 2). The client also requires multiple vitamins and a diet high in carbohydrates, protein, and fat to help meet nutritional requirements for growth (Option 4). Sweat gland abnormalities prevent sodium and chloride reabsorption, causing increased salt loss, dehydration, and hyponatremia during times of significant perspiration. Therefore, parents should increase the child's salt intake and fluids during hot weather, exercise, or fever (Option 3). (Option 1) Regardless of symptoms, clients should incorporate chest physiotherapy (eg, percussion, vibration, postural drainage) into their daily routine to improve mucus clearance and lung function. (Option 5) The parents should encourage physical activity as tolerated, which helps to thin secretions and remove them from airways and improves muscle strength and lung capacity. Educational objective: Cystic fibrosis causes increased viscosity of exocrine gland secretions. Clients require pancreatic enzyme supplements with meals and snacks; a diet high in carbohydrates, protein, and fat; and increased salt intake during times of significant perspiration. Clients should also incorporate chest physiotherapy and exercise into their daily routine.

A nurse is caring for a 3-month-old infant who has bacterial meningitis. Which clinical findings support this diagnosis? Select all that apply. 1. Depressed anterior fontanelle 2. Frequent seizures 3. High-pitched cry 4. Poor feeding 5. Presence of the Babinski sign 6. Vomiting

2, 3, 4, 6 Bacterial meningitis is an inflammation of the meninges in the brain and spinal cord that is caused by specific types of bacteria, including group B streptococcal, meningococcal, or pneumococcal pathogens. Clinical manifestations of bacterial meningitis in infants age <2 include: - Fever or possible hypothermia - Irritability, frequent seizures - High-pitched cry - Poor feeding and vomiting - Nuchal rigidity Bulging fontanelle possible but not always present One of the most common acute complications of bacterial meningitis in children is hydrocephalus. Long-term complications include hearing loss, learning disabilities, and brain damage. Due to the severity of potential complications, prompt identification and immediate treatment are vital for any client with suspected bacterial meningitis. (Option 1) Infants with bacterial meningitis may have bulging fontanelles due to an increase in intracranial pressure. Depressed fontanelles indicate severe dehydration. (Option 5) The Babinski reflex can be present up to 1-2 years and is a normal expected finding; it does not indicate meningitis. Educational objective: Bacterial meningitis is inflammation of the meninges in the brain and spinal cord caused by bacterial infection. Key characteristics of bacterial meningitis in infants under age 2 include frequent seizures, a high-pitched cry, poor feeding, nuchal rigidity, and possible bulging fontanelles.

The nurse is preparing for the admission of a 9-year-old client with new-onset tonic-clonic seizures. It is important for the nurse to ensure that what is in the room? Select all that apply. 1. Oral bite prevention device 2. Oxygen delivery system 3. Padding on the bed siderails 4. Soft arm and leg restraints 5. Suction equipment

2, 3, 5 Client safety is a priority when caring for a client with seizure activity. Protecting the airway and improving oxygenation includes turning the client on the side and providing oxygen and oral suctioning as needed. Padding the bed siderails provides the client protection and decreases the potential for injury from the metal in the event that the head or extremities hit the siderails during seizure activity. (Option 1) During seizure activity, nothing should be placed in the client's mouth. Placing objects in the mouth could result in injury to the client or health care provider. Maintaining an open airway is important and can be accomplished by turning the client on the side and providing oral suctioning to the inside of the cheeks as necessary. (Option 4) A client should never be restrained during seizure activity. Restraints could cause muscle or tissue injury. Educational objective: Turning the client on the side, providing oxygen and suctioning as needed, and padding the siderails or removing objects that are near the client can decrease the risk for injury during a seizure. Avoid restraints.

The nurse is reviewing the medical record of a 4-year-old client with failure to thrive. Which of the following risk factors likely contribute to the client's condition? Select all that apply. 1. Child is the youngest of four children in the home 2. One parent is incarcerated for spousal abuse 3. One parent was diagnosed with anorexia nervosa prior to having children 4. One parent works a full-time job outside the home 5. Parents are concerned about not having enough money to buy food

2, 3, 5 Failure to thrive (FTT) describes a client with poor growth due to inadequate caloric intake, inadequate food absorption, or excess caloric expenditure. In children, a weight that is <80% of ideal weight for height, weight that is below the 3rd to 5th percentile on growth charts, or persistent decrease in growth over time on growth charts support the diagnosis of FTT. Causes of FTT are typically multifactorial but may be related to certain medical conditions (eg, low birth weight, prematurity, congenital anomalies) or influenced by psychosocial risk factors, including: - Domestic violence in the home and/or history of child neglect or abuse (Option 2) - Caregiver or child with negative attitudes toward food (eg, fear of obesity, anorexia, food restriction) (Option 3) - Poverty or food insecurity (which is the greatest risk factor) (Option 5) - Disordered feeding behaviors (eg, unstructured mealtimes) (Option 1) Children are not at risk for FTT based on birth order or number of siblings. (Option 4) Children with a parent who works outside the home do not have an increased risk for FTT. Educational objective: Failure to thrive (FTT) describes a client with poor growth due to inadequate caloric intake, inadequate food absorption, or excess caloric expenditure. In children, psychosocial risk factors for FTT include lack of structured mealtimes, domestic violence, negative attitudes toward food, and poverty.

The pediatric nurse cares for a 16-year-old client who is scheduled for an appendectomy in the morning. Which of the following interventions are appropriate to support the client's psychosocial needs? Select all that apply. 1. Create a strict daily schedule for the client while hospitalized 2. Encourage the client to have peers visit while hospitalized 3. Ensure parental presence during any client procedure 4. Include the client as an active participant when planning care 5. Support the client in discussing concerns about body image changes

2, 4, 5 Pediatric clients are at increased risk for impaired psychosocial integrity during stressful experiences (eg, hospitalization, surgical procedures, medical treatment) and require developmentally appropriate care based on their age to assist with managing stress. Unaddressed or ineffectively managed developmental needs may lead to or worsen the client's anxiety, disobedient behavior, and/or social withdrawal. Developmentally appropriate nursing care for an adolescent client includes: - Encouraging interaction with peers (eg, hospital visits, internet communication), which supports the developmental need for social connection and support and reduces stress and anxiety (Option 2) - Involving the client in care planning to address the developmental needs for control and independence (Option 4) - Assisting the client to discuss emotions or fears related to treatment (eg, changes in body image, disability, possibility of death) to improve coping, support the developmental need for understanding, and decrease anxiety (Option 5) (Option 1) Strict scheduling by the nurse reduces the adolescent's perception of control and independence, which may increase stress. Adolescents should be allowed to determine their daily schedule when possible. (Option 3) Loss of privacy (eg, forced parental presence) can increase anxiety in the adolescent client. Adolescents should be asked if they want parents present for procedures and what level of parental involvement they prefer. Educational objective: Nursing care for the hospitalized adolescent client needs to be developmentally appropriate and promote the elimination of stressors. The nurse should encourage adolescent clients to interact with peers, discuss emotions or fears about treatments, and involve the client in decision-making regarding the plan of care.

In the emergency department, a pediatric client is placed on mechanical ventilation by means of an endotracheal tube. Several hours later, the nurse enters the room and finds the client in respiratory distress. It is most important for the nurse to take which of these actions? 1. Assess the client for intercostal retractions 2. Assess the client's blood pressure in both arms 3. Auscultate the client's lung sounds 4. Observe the color of the client's fingernail beds

3 A client experiencing respiratory distress while receiving mechanical ventilation should be assessed for proper ventilation first. The nurse needs to determine if the mechanical ventilation equipment is still properly placed in the trachea. An endotracheal tube (ET) can become displaced with movement. By assessing the client's lung sounds, the nurse can quickly determine if ET placement has been compromised (Option 3). Airway is the priority for this client. By auscultating the client's lung sounds, the nurse can determine if the client has an open airway. (Option 1) This is an assessment of the client's breathing, which is not the priority at this time. (Option 2) This is an assessment of the client's circulation, which is not the priority at this time. (Option 4) This is an assessment of the client's circulation, which is not the priority at this time. Educational objective: Clients with respiratory distress should be assessed for a patent airway first. The nurse should assess the client's airway to determine if it is present or needs to be established.

A nurse is teaching the parent of a child who has a new diagnosis of absence seizures. Which statement by the parent indicates understanding of the teaching? 1. "My child may experience incontinence." 2. "My child may seem confused afterwards." 3. "My child may stare and seem inattentive." 4. "My child will notice unusual odors prior to the event."

3 Absence seizures occur in children age 4-12 and usually disappear at puberty. Clinical manifestations include a brief loss of consciousness and an appearance of inattention or daydreaming (the absence attack) without loss of postural body tone. However, slight loss of tone may lead to dropping objects held in hands. Most absence seizures last less than 10 seconds and often go unrecognized. Following an attack, behavior and awareness return immediately to normal. The child does not experience a postictal period but usually has no recollection that a seizure has occurred. A child may have multiple absence seizures each day. Treatment includes the use of anticonvulsant medication(s). (Options 1, 2, and 4) Altered sensory perceptions (eg, awareness of odors [aura]), postictal confusion, and incontinence are clinical manifestations of complex partial or tonic-clonic seizures. Educational objective: Absence seizures are characterized by a brief loss of consciousness and an appearance of inattention or daydreaming without loss of postural tone. Most absence seizures last less than 10 seconds. The seizures occur in children age 4-12, and multiple seizures may occur daily.

The nurse is caring for a 10-year-old diagnosed with osteomyelitis. What is the best activity the nurse can suggest to promote age-specific growth and development during hospitalization? 1. Fantasy play with puppets 2. Invite friends to come visit 3. Provide missed schoolwork 4. Watch favorite movies

3 According to Erikson's stages of psychosocial development, school-age children deal with the conflict of industry versus inferiority. Attaining a sense of industry (competence) is the most significant developmental goal for children age 6-12. Parents should therefore be encouraged to provide a hospitalized child with missed school work on a regular basis. This will help the child keep up with school demands, learn new skills, cope with the stressors of hospitalization, and avoid a sense of inferiority. (Option 1) Fantasy play with puppets is more appropriate for a preschool-age child as imaginary play and magical thinking peak during this stage of development. (Option 2) Although school-age children enjoy spending time with friends, peer relationships are significantly more important during the adolescent period. (Option 4) Watching television is a good diversion for all hospitalized children, but it does not promote age-specific growth and development. Educational objective: According to Erikson's stages of psychosocial development, school-age children deal with the conflict of industry versus inferiority. During this stage, unlike other developmental stages, learning is a priority and completing school work provides a sense of accomplishment and satisfaction. It is therefore important that parents provide hospitalized school-age children with missed school work on a regular basis.

When performing developmental screenings in the well-child clinic, the registered nurse understands that which child is at highest risk of developing autism spectrum disorder? 1. 2-year-old who has a vocabulary of 10 words 2. 3-year-old who received measles, mumps, and rubella immunization at age 1 year 3. 4-year-old whose 10-year-old sibling has the disorder 4. 5-year-old whose parents were age 42 at the time of birth

3 Although the cause of autism spectrum disorder (ASD) is unknown, numerous studies indicate that it has a strong genetic component. The underlying genetic source is unknown in the majority of cases; however, researchers hypothesize that genetic factors predispose to an autism phenotype and that genetic expression is influenced by environmental factors. (Option 1) A 2-year-old with a vocabulary of a few words only is a concern; however, there are a number of factors and/or conditions that could cause language or overall developmental delay. (Option 2) There is no scientific evidence that the measles, mumps, and rubella vaccine or thimerosal-containing vaccines (eg, influenza) are linked to ASD. (Option 4) Retrospective studies have linked parents of older age to autism; however, this association is inconclusive. Educational objective: There is strong scientific evidence of a genetic component to autism spectrum disorder (ASD). As a result, parents who have a child with ASD are at higher risk of having another child with this disorder. There is no evidence that supports a link between vaccines and ASD, and studies on associations between advanced parental age and the disorder are inconclusive.

The nurse assesses a child who has been treated for an acute asthma exacerbation. Which client assessment is the best indicator that treatment has been effective? 1. Episodes of spasmodic coughing have decreased 2. No wheezes are audible on chest auscultation 3. Oxygen saturation has increased from 88% to 93% 4. Peak expiratory flow rate has dropped from 212 L/min to 127 L/min

3 Asthma is a chronic condition characterized by inflammation, swelling, and narrowing of the airways in the lungs. The client having an acute attack will experience chest tightness, wheezing, uncontrollable coughing, rapid respirations, retractions, and anxiety and panic. Treatment of an acute attack can include nebulized breathing treatment with a short-acting beta-agonist medication such as albuterol, and oral or IV corticosteroids. Oxygen saturation is the best indicator of treatment effectiveness as it reflects gas exchange. (Option 1) Decreased coughing may indicate improvement, but it is more subjective than measurement of oxygen saturation. In addition, it may be a sign of client exhaustion and worsening asthma. (Option 2) The absence of wheezes may indicate resolution of the attack or progression of airway swelling to the point of little air flowing through the lungs. (Option 4) Peak expiratory flow rate, by measuring how much air a person can exhale, indicates the amount of airway obstruction. Following treatment for an acute asthma attack, an increase, not a decrease, in peak expiratory flow would be expected. Educational objective: Improvements in oxygen saturation and peak expiratory flow are the best indicators of treatment effectiveness during an acute asthma attack.

Several children are brought to the emergency room after a boating accident in which they were thrown into the water. The children are now 6 hours post admission to the clinical observation unit. Which client should the nurse evaluate first? 1. Client who did not require CPR but now has a new oxygen requirement of 2 L via nasal cannula to maintain a saturation of 95% 2. Client who did not require CPR but was coughing on arrival to the hospital and is now crying inconsolably and asking for the mother 3. Client who received CPR for 2 minutes on the scene and whose respiratory rate has now dropped from 61/min to 18/min 4. Client who was briefly submerged in water and received rescue breaths on the scene and is now irritable and refusing food and drink

3 Clients with morbidity related to immersion in water are described as having submersion injury. Even if an individual was submerged for a very brief time, it is possible that water may have been aspirated, which can lead to respiratory compromise. Observation for at least 6 hours is recommended as the majority of significant respiratory problems will manifest in this time period. A marked decrease in respiratory rate or increased work of breathing may indicate respiratory fatigue, and immediate intervention is needed (Option 3). Impending respiratory failure is the immediate priority. (Option 1) A new oxygen requirement is an important symptom; however, this child has good oxygen saturation with the nasal cannula and is therefore not the immediate priority. (Option 2) This child who is coughing and emotionally distressed should be seen and comforted by the nurse but is not the priority. (Option 4) Irritability can be an early sign of hypoxia in a toddler. This child should be assessed promptly but is not the immediate priority. Educational objective: Clients who have sustained submersion injury should be evaluated immediately and observed for at least 6 hours for new or worsening respiratory failure. Changes in respiratory pattern or rate, oxygen saturation, and level of consciousness can signal impending respiratory failure, which can be life threatening.

The clinic nurse is interviewing the parents of a 6-month-old client about the infant's diet. Which statement by the parents is most concerning? 1. "Because apples are healthy, we make apple pie and feed small, soft bites to our baby." 2. "If our baby refuses to finish foods, we continue to offer small bites, so food isn't wasted." 3. "Infant oatmeal sweetened with fresh honey is our baby's favorite breakfast." 4. "We found that the food in TV dinners can be easily pureed and is convenient."

3 Clostridium botulinum spores in honey or soil can colonize an infant's immature gastrointestinal system and release a toxin that causes botulism, a rare but serious illness. The toxin attacks the neuromuscular system, causing progressive muscle paralysis that can potentially lead to respiratory failure and death. Initial manifestations may include constipation, generalized weakness, difficulty feeding, and decreased gag reflex. Iron-fortified infant cereals (eg, oatmeal) mixed with formula or breastmilk are appropriate for infants >6 months; however, honey (especially raw or wild) is not recommended for infants age <12 months due to the risk of botulism (Option 3). (Option 1) Although apple pie adds excessive amounts of fat and sugar to the infant's diet, this is not the priority over honey, which can be life-threatening. (Option 2) Small portions (<1 tablespoon) of solid food are appropriate for infants. Continuing to feed an infant who acts disinterested in food (eg, turns away) can contribute to future obesity. The nurse should explore this behavior further; however, the priority is to address the danger of feeding an infant honey. (Option 4) TV dinners and various canned foods have high sodium and sugar content and therefore are not the best sources of nutrition for an infant. However, TV dinners are not immediately life threatening. Educational objective: Clostridium botulinum spores in honey can colonize an infant's (age <12 months) immature gastrointestinal system and release a toxin that causes botulism, a rare but potentially life-threatening illness.

The nurse is assessing a 3-year-old client in the emergency department and finds dyspnea, high fever, irritability, and open-mouthed drooling with leaning forward. The parents report that the symptoms started rather abruptly. The client has not received age-appropriate vaccinations. Which set of actions should the nurse anticipate? 1. 20-gauge needle insertion at the mid-axillary line for pleural aspiration 2. 4 L oxygen at 100% per nasal cannula with bilevel positive airway pressure (BPAP) ventilation standing by 3. Intubation in the operating room with a prepared tracheotomy kit standing by 4. Nebulized racemic epinephrine with pediatric anesthesiologist standing by

3 Epiglottitis should be considered first in a 3-7-year-old child with acute respiratory distress, toxic appearance (eg, sitting up, leaning forward, drooling), stridor, and high-grade fever. Tachycardia and tachypnea are also present. This is a pediatric emergency and should be managed with endotracheal intubation; however, intubation of such clients is difficult, and preparation for possible tracheostomy is also standard. The complications of epiglottitis are serious and include sudden airway obstruction. (Option 1) This is a recommended therapy for spontaneous tension pneumothorax, which is demonstrated by tracheal deviation; absent lung sounds; and severe, abrupt hypotension and dyspnea. (Option 2) Neither oxygenation nor BPAP is acceptable in acute epiglottitis as the trachea can close completely from edema. (Option 4) This is the appropriate therapy for croup, not epiglottitis. Croup is notably distinct for the hacking cough, which does not occur in epiglottitis. Educational objective: When assessing a client with symptoms suggestive of epiglottitis (eg, acutely ill, drooling, leaning forward, dyspnea), the nurse should prepare for an emergency airway.

The nurse receives change of shift report on 4 clients. Which client should the nurse assess first? 1. 6-month-old with respiratory syncytial virus and pulse oximetry of 90% 2. 1-year-old with otitis media and a temperature of 102.5 F (39.2 C) rectally 3. 2-year-old with suspected epiglottitis 4. 3-year-old who has a barking-type cough

3 Epiglottitis, a sudden-onset medical emergency due to Haemophilus influenzae, causes severe inflammatory obstruction above and around the glottis. The affected child will typically progress from having no symptoms to having a completely occluded airway within hours. Sitting in a tripod position (upright and leaning forward with the chin and tongue sticking out) is a classic presentation. The child will likely drool and be very restless and anxious secondary to airway obstruction and hypoxia. Throat inspection should not be done until emergency intubation is readily available (if necessary). (Option 1) Oxygen saturation ≥90% is the treatment goal for bronchiolitis caused by respiratory syncytial virus. (Option 2) This temperature is an expected finding in the setting of otitis media and does not carry the urgency of airway impairment. (Option 4) A barking-type cough is seen in viral croup syndromes. The resonant hoarse cough is secondary to narrowed airways. Croup is typically mild but can become life-threatening if the airway swells excessively. This child would need to be assessed next. Educational objective: Epiglottitis is a medical emergency as the child can rapidly progress from being asymptomatic to having a completely occluded airway. Emergency intubation equipment should be readily available.

The nurse assesses 4 infants. Which assessment finding would require follow-up by the health care provider? 1. 3-week-old whose anterior fontanelle bulges with crying 2. 4-week-old whose posterior fontanelle is soft 3. 6-month-old with birth weight of 7 lb 3 oz (3.3 kg) who now weighs 12 lb (5.4 kg) 4. 12-month-old with birth weight of 6 lb 4 oz (2.8 kg) who now weighs 20 lb (9.1 kg)

3 Infant growth is fast paced during the first year of life, with birth weight doubling by age 6 months and tripling by age 12 months. During the first year, birth length increases by approximately 50%. At birth, head circumference is slightly more than chest circumference, but these equalize by age 12 months. (Options 1 and 2) At birth, the infant has non-ossified membranes called fontanelles; these "soft spots" lie between the bones of the cranium. The 2 most noticeable are the anterior and posterior fontanelles, which are soft and non-fused. Fontanelles should be flat, but slight pulsations noted in the anterior fontanelle are normal as is temporary bulging when the infant cries, coughs, or is lying down. The posterior fontanelle fuses by age 2 months, and the anterior fontanelle fuses by age 18 months. (Option 4) This assessment shows tripling of the birth weight by age 12 months, a normal finding. Educational objective: Infants should double in birth weight by age 6 months and triple in birth weight by age 12 months. At birth, head circumference is slightly more than chest circumference, but these equalize by age 12 months. The posterior fontanelle fuses by age 2 months, and the anterior fontanelle fuses by age 18 months.

The clinic nurse is caring for several clients during well-child visits. The nurse should recognize which client as being the most at risk for anemia? 1. 1-month-old infant born at term gestation who exclusively breastfeeds 2. 2-month-old infant born at preterm gestation who exclusively receives iron-fortified formula 3. 3-month-old infant born at preterm gestation who is exclusively bottle-fed with breastmilk 4. 6-month-old infant born at term gestation who breastfeeds and eats iron-fortified infant cereal

3 Iron deficiency during infancy causes reduced hemoglobin production, resulting in anemia, decreased immune function, and delayed growth and development. During gestation, the fetus stores iron received from the mother; the amount of iron stored is dependent on the length of gestation. After birth, iron stores are progressively depleted and nutritional sources of iron are eventually required. Infants born at preterm gestation have less time in utero to accumulate iron. Preterm infants typically deplete iron stores by age 2-3 months and require additional iron supplementation (eg, oral iron drops, iron-fortified formula). Therefore, a 3-month-old infant born at preterm gestation who is exclusively receiving breastmilk is most at risk for anemia (Option 3). (Options 1 and 4) Infants born at term gestation have sufficient iron stores for the first 4-6 months of life. However, infants receiving exclusively breastmilk require iron supplementation (eg, oral iron drops) around age 4 months until food sources of iron (eg, iron-fortified infant cereal) are adequate around age 6 months. (Option 2) Although this client is at risk for anemia due to preterm gestation, the risk decreases due to intake of iron-fortified formula. The iron content of most infant formulas is adequate for the first 12 months of life. Educational objective: During gestation, the amount of iron a fetus stores is dependent on the length of gestation. Infants born at preterm gestation have lower iron stores at birth and are at an increased risk for iron-deficiency anemia. Iron supplementation (eg, oral iron drops, iron-fortified formula) is usually needed by preterm infants at an earlier age (2-3 months).

A 12-month-old is found to have a moderately elevated blood lead level. Which of the following is the most serious concern for this child? 1. Gastrointestinal bleeding 2. Growth retardation 3. Neurocognitive impairment 4. Severe liver injury

3 Lead poisoning still occurs in the United States, although not as often as in previous decades. A common source of exposure is lead-based paints found in houses built before 1978, when such paint was banned. Blood lead level (BLL) screenings are recommended at ages 1 and 2, and up to age 6 if not previously tested. Because lead poisoning particularly affects the neurological system, elevated BLLs (≥5 mcg/dL [0.24 µmol/L]) are dangerous in young children due to immature development of the brain and nervous system. A mild to moderate increase in BLL can manifest with hyperactivity and impulsiveness; prolonged low-level exposure can cause developmental delays, reading difficulties, and visual-motor issues. Extremely elevated BLLs can lead to permanent cognitive impairment, seizures, blindness, or even death. (Option 1) Gastrointestinal bleeding is a concern for clients with iron poisoning but has no link to lead toxicity. (Option 2) Although delays in physical growth can result from chronic lead toxicity, the danger of permanent damage to the neurological system is a higher priority, particularly for young children. Growth retardation more commonly occurs with chronic anemia or pituitary disorders. (Option 4) Lead poisoning is most threatening to the kidneys and neurological system; liver injury typically does not occur. Severe liver damage is closely associated with acetaminophen overdose or Reye syndrome. Educational objective: Lead poisoning can lead to many severe complications of the neurological system (eg, developmental delays, cognitive impairment, seizures). Elevated blood lead levels are particularly dangerous in young children due to immature development of the brain and nervous system.

The nurse has been providing care for the past month to a 7-year-old client recently diagnosed with type 1 diabetes mellitus. Initially, the family seemed devastated about the diagnosis and the client's parent stated, "Our lives will never be the same." Which statement made by the parent indicates that nursing interventions and education have been effective? 1. "Our child will not be able to participate in any sporting events." 2. "Our whole family will have to make sacrifices to deal with this disease." 3. "We are working to manage this disease so that it cannot control our child's life." 4. "We have set aside a place in the pantry for our child's special foods."

3 Parents experience a variety of emotions when a child is diagnosed with a chronic illness (eg, diabetes mellitus). Reactions include shock, denial, helplessness, anger, fear, anxiety, and often guilt about perceived contribution or failure to prevent the development of the disease. Parents' emotional response, adaptation, and coping strategies impact the child's perception of self and ability to manage the disease. In providing diabetes education, the nurse should emphasize that with planning and preparation, diabetes can be managed and controlled and the child can resume regular day-to-day activities and have an independent life (Option 3). (Option 1) Clients with diabetes can participate in a wide variety of sports and should continue pursuing age-appropriate physical activities. (Option 2) The diagnosis and management of diabetes in a child can affect the whole family; however, the word "sacrifice" suggests that the parent is feeling victimized by the disease. (Option 4) Nutritional management of diabetes does not require special foods. Nutrition education should emphasize healthy food choices, consistent amounts of carbohydrates, and a balance of food choices, with insulin and exercise for blood glucose control. Educational objective: The diagnosis of a chronic illness (eg, diabetes) in a child will have an impact on the entire family. When parents see themselves and the child as capable of being independent and in control of the disease, there is an increased likelihood that the disease will be managed and controlled and the child can have an independent life.

An 8-month-old infant is scheduled for a femorally inserted balloon angioplasty of a congenital pulmonic stenosis in the cardiac catheterization laboratory. Which finding should the nurse report to the health care provider that could possibly delay the procedure? 1. Auscultation of a loud heart murmur 2. Infant has been NPO for 4 hours 3. Infant has severe diaper rash 4. Slight cyanosis of the nail beds

3 Pulmonic stenosis causes increased pressure in the right side of the heart as the ventricle tries to push blood through the narrowed pulmonary area to the lungs. In severe pulmonic stenosis, higher pressure in the right side of the heart causes unoxygenated blood to travel to the left side through the foramen ovale (or other congenital defect) and into the systemic circulation, leading to chronic hypoxia and cyanosis and requiring repair (interventional catheterization or surgery). The presence of severe diaper rash should be reported to the health care provider (HCP). This could delay the procedure if the rash is in the groin area where access is planned for a femorally inserted arterial cannula. Yeast or bacteria may be present on the rash and could be introduced into the bloodstream with the arterial stick (Option 3). (Option 1) A loud heart murmur can be an expected finding in a child with pulmonic stenosis. (Option 2) Children are NPO for 4-6 hours or longer before the procedure. Younger children and infants may have a shorter period of NPO status and should be fed right up to the time recommended by the HCP. (Option 4) Cyanosis indicates severe pulmonic stenosis with right-to-left shunt and the need for interventional catheterization or surgery without delay. Educational objective: The nurse should report the presence of severe diaper rash in an infant who has an interventional catheterization procedure planned. The rash may delay the procedure due to possible contamination at the insertion site.

The nurse is caring for an infant with suspected meningitis and preparing to assist with a lumbar puncture. What is the appropriate nursing intervention? 1. Administer oxygen via nasal cannula for client comfort and safety 2. Clean area with povidone iodine in a circular motion moving outward 3. Hold the child with the head and knees tucked in and the back rounded out 4. Monitor and record vital signs every 15 minutes throughout the procedure

3 The optimal position for access during a lumbar puncture is to have the client's head and knees tucked in and the back rounded out. This provides the most room for the health care provider (HCP) to perform the procedure and allows for a good hold to keep the client still. A lumbar puncture is a sensitive procedure, and it is important to keep the child from moving during needle insertion. (Option 1) Unless the client has improper air exchange, oxygen administration is not needed. The nasal cannula will most likely bother the child and lead to unnecessary movement during needle placement. (Option 2) The HCP performing the lumbar puncture will feel the spine for correct needle placement and then sterilize and prepare the chosen area for needle insertion. (Option 4) Unless the client is unstable, there is no need to record vital signs every 15 minutes. The client should be awake and alert, and the procedure should be fairly short in duration. Educational objective: Performing a lumbar puncture on a child is a very sensitive procedure that requires accuracy. The correct position and ability to hold the child still are important to achieve the best result and minimize the risk for complications.

The clinic nurse is caring for a 3-year-old client. Which task, if not observed or reported by the parents as accomplished, will cause the nurse concern? 1. Catches a ball at least 50% of the time 2. Copies a square with a pencil or crayon 3. Eats with a spoon 4. Hops on one foot

3 Things that most children can do by a certain age are considered developmental milestones. These include the following areas of development: social/emotional, language/communication, cognitive, and physical. Each child develops in a unique pattern, and ages are considered as general guidelines for assessing development. Normally, a toddler develops the ability to use a spoon by 18 months. Therefore, a 3-year-old should be able to eat with a spoon. (Option 1) Catching a ball 50% of the time is a developmental expectation for a 4-year-old. (Option 2) A 4-year-old can copy or draw a square with a pencil or crayon. Copying shapes other than a circle is a developmental expectation for a 5-year-old. (Option 4) Hopping on one foot is a developmental expectation for a 4-year-old. Educational objective: A 3-year-old should be able to eat with a spoon.

A nurse in a pediatric clinic is performing a physical examination of a 30-month-old child. Which finding requires further evaluation? 1. Bladder and bowel control achieved 2. Chest circumference is greater than abdominal circumference 3. Current weight is 6 times greater than birth weight 4. Head circumference increased by 1 in (2.5 cm) in the past year

3 Weight gain slows during the toddler years with an average yearly weight gain of 4-6 lb (1.8-2.7 kg). By age 30 months, current weight should be approximately 4 times greater than birth weight. A toddler weighing 6 times the initial birth weight requires further evaluation. Family nutrition and meal habits should be discussed. (Option 1) A toddler achieves bowel and bladder sphincter control by age 24 months as bladder capacity increases. (Option 2) Chest circumference exceeds abdominal circumference after age 2, resulting in a taller and more slender appearance. (Option 4) Head circumference increases by 1 in (2.5 cm) during the second year and then slows to a growth rate of 0.5 in (1.25 cm) per year until age 5. Educational objective: Weight gain slows during the toddler years. By age 30 months, a toddler's weight should be approximately 4 times greater than the birth weight. Health Promotion and Maintenance

The registered nurse has completed a well-baby assessment of an 18-month-old. Which assessment findings prompted the nurse to make a referral for a formal developmental screening test? 1. Cannot climb steps by self, pulls a toy, turns the pages of a book 2. Is bottle fed, can hold a spoon, creeps down stairs 3. Throws a ball, is able to point to 2 or 3 body parts, cannot draw a picture 4. Uses 2 words, cannot hold a cup, can seat self in a small chair

4 An 18-month-old should have a vocabulary of 10 or more words and be able to hold and drink from a cup. Both of these types of delays (communication and language development, and fine motor skills) may be more apparent at age 18 months than at earlier ages. Either finding should prompt the nurse that further evaluation is needed. (Option 1) An 18-month-old can climb stairs with assistance, use a pull-toy, and turn the pages of a book. (Option 2) An 18-month-old may continue to be bottle fed at times, can hold and clumsily use a spoon, and can creep down stairs. (Option 3) An 18-month-old might be able to scribble but would not be able to draw a picture; an 18-month-old can throw a ball and point to body parts. Educational objective: An 18-month-old should have a vocabulary of at least 10 words and be able to use a spoon. Health Promotion and Maintenance

The nurse is reinforcing education to the caregivers of a 9-year-old client diagnosed with scarlet fever. The client has a history of type 1 diabetes mellitus. Which statement by the caregivers indicates that further teaching is needed? 1. "We will encourage extra fluid intake while our child is sick." 2. "We will increase the frequency of blood glucose checks." 3. "We will monitor our child's urine for ketones with each void." 4. "We will not administer insulin if our child is unable to eat."

4 An acute illness (eg, scarlet fever) in clients with type 1 diabetes may trigger the release of stress hormones, which leads to higher blood glucose and ketone levels (sometimes leading to ketoacidosis). Clients with type 1 diabetes do not produce any insulin (unlike those with type 2 diabetes), so clients should not skip administration of external insulin even if not eating. Insulin therapy should be continued as prescribed during an acute illness (Option 4). Additional sick-day management includes: - Increasing frequency of blood glucose level checks (every 1-4 hours) - Increasing or decreasing the dose of insulin as needed based on blood glucose levels - Maintaining adequate hydration - Testing for urinary ketones frequently (Options 1 and 3) Stress hormones released during illness cause increased insulin resistance and lead the body to break down fat for energy. Ketones are produced when fat is broken down, which can lead to diabetic ketoacidosis (DKA). The client's urine should be monitored frequently for ketones while the client is sick. Fluids are encouraged to clear ketones from the system and prevent dehydration. (Option 2) Blood glucose should be assessed frequently while the client is ill due to the potentially unpredictable and rapidly changing levels caused by illness and/or fasting. Educational objective: Clients with type 1 diabetes should not discontinue insulin usage during an illness. Encouraging fluids and monitoring glucose and ketone levels are priorities for this client.

The nurse in a clinic is caring for an 8-month-old with a new diagnosis of bronchiolitis due to respiratory syncytial virus (RSV). Which instructions can the nurse anticipate reviewing with the parent? 1. Administering a cough suppressant and antihistamine 2. Prophylactic treatment of family members 3. Temporary cessation of breastfeeding 4. Use of saline drops and a bulb syringe to suction nares

4 Bronchiolitis is a common viral illness of childhood that is usually caused by RSV. It typically begins with viral upper respiratory symptoms (eg, rhinorrhea, congestion) that progress to lower respiratory tract symptoms such as tachypnea, cough, and wheezing. Bronchiolitis is a self-limited illness and supportive care is the mainstay of treatment. Most children can be managed in the home environment. Breastfeeding should be continued and additional fluids offered if there is a risk of dehydration due to frequent coughing and vomiting (Option 3). Parents should be instructed to use saline nose drops and then suction the nares with a bulb syringe to remove secretions prior to feedings and at bedtime (Option 4). (Option 1) Medications such as cough suppressants, antihistamines, bronchodilators (eg, albuterol), and corticosteroids have not been found to be effective and are not recommended. (Option 2) Prophylactic treatment of family members is recommended for pertussis infection but not for RSV bronchiolitis. Educational objective: Bronchiolitis is a common viral illness of childhood that is usually caused by RSV. The focus of home care is on monitoring respiratory status and periodic nasal suctioning using saline nose drops to ease breathing. Additional fluids should be offered.

The school nurse evaluates a 9-year-old who is sweating, trembling, and pale. The client has type 1 diabetes managed with insulin glargine and NPH. What is the most appropriate action by the nurse? 1. Administer scheduled dose of NPH insulin 2. Give emergency glucagon IM injection 3. Give peanut butter and crackers 4. Provide 4 oz (120 mL) of a regular soft drink

4 Clients experiencing hypoglycemia may develop shakiness, palpitations, sweating, pallor, and altered mental status (eg, difficulty speaking, confusion). If manifestations of hypoglycemia are present, the nurse should check the client's blood glucose (BG) level immediately. A BG of <70 mg/dL (3.9 mmol/L) requires treatment; however, if glucose testing is not readily available, the client should be treated based on symptoms. Hypoglycemia treatment in a conscious client is administration of 15 g of a quick-acting carbohydrate (Option 4). After treatment, the nurse should recheck BG every 15 minutes, repeating treatment if it remains low. Quick-acting carbohydrate options include: > 4 oz (120 mL) of a regular soft drink or fruit juice > 8 oz (240 mL) of low-fat milk > 1 tablespoon (15 mL) of honey or syrup > 6 hard candies > Commercial dextrose products (Option 1) The nurse should hold the client's scheduled insulin until the client's BG is normal and symptoms resolve. (Option 2) An emergency glucagon IM injection is indicated if the client is somnolent, unconscious, seizing, or unable to swallow. (Option 3) After the client's BG improves, the client should eat a meal. However, if the next meal is more than an hour away, the nurse should give the client a serving of carbohydrate plus protein or fat (eg, peanut butter, cheese) to maintain glucose levels. Educational objective: Clients with diabetes mellitus should be monitored for signs of hypoglycemia (eg, shakiness, sweating, pallor, alterations in mental status). Conscious clients experiencing hypoglycemia should receive a snack of 15 g of a quick-acting carbohydrate.

The nurse assesses a pediatric client who was diagnosed with diarrhea caused by Escherichia coli. The nurse is most concerned with which finding? 1. Blood-streaked stools 2. Client drank fruit juice 3. Dry mucous membranes 4. Petechiae noted on the trunk

4 Hemolytic uremic syndrome (HUS) is a life-threatening complication of Escherichia coli diarrhea and results in red cell hemolysis, low platelets, and acute kidney injury. Hemolysis results in anemia, and low platelets manifest as petechiae or purpura. Therefore, the presence of petechiae in this client could indicate underlying HUS and needs further assessment. (Option 1) E coli bacteria infect people through contaminated food or water and attack the digestive system. Blood-streaked stool due to intestinal irritation is a common symptom associated with this illness. Treatment is aimed at preventing dehydration, and clients usually improve in about a week. (Option 2) Fruit juices are discouraged in acute diarrhea as they have high sugar (osmolality) and low electrolyte content. Continuing the client's normal diet (solid foods) is encouraged as it shortens the duration and severity of the diarrhea. (Option 3) Dry mucous membranes are a sign of dehydration, a common complication of any persistent diarrhea. Dehydration should be treated promptly, especially in children; however, as long as fluid is replenished, the condition is not life-threatening. Educational objective: Hemolytic uremic syndrome is a life-threatening complication of Escherichia coli diarrhea. Clinical features include anemia (pallor), low platelets (petechiae and purpura), and acute kidney injury (low urine output).

The nurse is providing health promotion education to the parent of a toddler. Which statement by the parent requires the nurse to clarify teaching? 1. "I will offer my child options rather than asking yes or no questions." 2. "I will wait at least 15 minutes after a play period to offer a meal to my child." 3. "If my child is having a tantrum, I will have them sit in a quiet area for a short time-out." 4. "If my child refuses a meal, I will have them stay at the table until they eat half the food."

4 Physiologic anorexia (ie, a decrease in nutritional need and appetite) occurs when the very high metabolic demands of infancy slow to keep pace with the moderate growth during toddlerhood. During this phase, toddlers are increasingly picky about their food choices and eating schedules. Parents sometimes fear the child is not consuming enough calories, but intake over several days usually meets nutritional and energy needs. Parents should avoid forcing food or pressuring toddlers to eat more, which can lead to poor eating habits in the future (Option 4). Strategies to promote intake for toddlers include the following: - Offering 2 or 3 high-quality food choices - Keeping food portions small (1-2 tablespoons per serving) - Exposing the child to new foods repeatedly - Avoiding distractions (eg, television, toys) during meals/snacks (Option 1) Parents can help toddlers gain a sense of control by providing options (eg, corn or peas) rather than asking yes or no questions. (Option 2) Toddlers may have difficulty sitting still at the table immediately after physical activity. Offering a 15-minute period to calm down promotes better eating habits. (Option 3) Tantrums are common as toddlers seek more independence. Parents can consider using time-outs in a quiet, controlled environment to help children calm down. Educational objective: Toddlers may develop a physiologic decrease in appetite due to reduced nutritional and metabolic demands. The nurse should reassure parents that intake over several days usually meets nutritional and energy needs and encourage parents not to force or pressure toddlers to eat more. Health Promotion and Maintenance

A 2-month-old infant has been admitted to the hospital with suspected shaken baby syndrome (abusive head trauma). In reviewing the infant's chart, the nurse expects to encounter which of these clinical findings? 1. A reported history of recent trauma 2. Abdominal bruising 3. External signs of trauma 4. Irritability and vomiting

4 Shaken baby syndrome (SBS) is a type of abusive head injury and is defined by the Centers for Disease Control and Prevention (CDC) as severe physical child abuse resulting from violent shaking of an infant by the arms, legs, or shoulders. The impact of the shaking causes bleeding within the brain or the eyes. It is not uncommon for the diagnosis of SBS to be missed as the clinical findings are often vague and nonspecific—vomiting, irritability, lethargy, inability to suck or eat, seizures, and inconsolable crying. Usually, there are no external signs of trauma except for occasional small bruises on the chest or upper arms where the child was held during the shaking episode. The most common reasons that caregivers seek medical attention for children with SBS are breathing difficulty, apnea, seizures, and lifelessness. Caregivers typically do not offer a history of trauma nor do they report the episodes of shaking. By contrast, children who have sustained unintentional head injury are typically brought for treatment out of concern by their caregivers even when the children are asymptomatic. (Option 1) Typically, a history of physical trauma is not reported by the parent or caregiver. (Option 2) Abdominal bruising is not an expected clinical finding of SBS. (Option 3) External signs of trauma are usually absent on physical examination of an infant with SBS. Minimal bruising on the extremities or chest may be present. Educational objective: Shaken baby syndrome is a form of child physical abuse resulting from violent shaking of an infant by the extremities or shoulder that causes bleeding within the brain and/or eyes. The clinical findings of shaken baby syndrome are nonspecific and include lethargy, vomiting, seizures, irritability, inability to eat, and inconsolable crying. Multiple and severe shaking episodes can result in breathing difficulty and lifelessness. Caregivers typically do not report a history of trauma.

A nurse is assisting a new mother as she is breastfeeding her infant. The infant has been diagnosed with tetralogy of Fallot. During feeding, the infant becomes cyanotic and is having difficulty breathing. What should be the nurse's first action? 1. Administer morphine to the infant 2. Administer oxygen via mask 3. Assess infant's vital signs and pulse oximetry 4. Place the infant in the knee-chest position

4 Tetralogy of Fallot is a congenital cardiac defect that typically has 4 characteristics: pulmonary stenosis, right ventricular hypertrophy, overriding aorta, and ventricular septal defect. This infant is experiencing a hypercyanotic episode, or "tet spell," which is an exacerbation of tetralogy of Fallot that can happen when a child cries, becomes upset, or is feeding. The child should first be placed in a knee-to-chest position. Flexion of the legs provides relief of dyspnea as this angle improves oxygenation by reducing the volume of blood that is shunted through the overriding aorta and the ventricular septal defect. (Option 1) Morphine may be considered if the dyspnea is not relieved by the knee-to-chest position. (Option 2) If oxygen saturation remains low, oxygen may need to be administered. (Option 3) Vital signs and pulse oximetry may be checked after the infant has been placed in the knee-chest position. Educational objective: To relieve a hypercyanotic episode, or "tet spell," the nurse should place the infant or child in the knee-chest position.

A nurse is caring for a child who is receiving oxygen at 2 L/min by nasal cannula and observes the current oxygen saturation and pulse plethysmographic waveform on the pulse oximeter. Which intervention should be the nurse's initial action? Click the exhibit button for additional information. SpO2 83% Plethysmographic irregular/erratic waveform 1. Auscultate the child's lung fields 2. Have the child take slow, deep breaths 3. Increase the oxygen flow rate to 3 L/min 4. Verify the position and integrity of the finger probe

4 The first action of the nursing process is assessment. The nurse should first evaluate the accuracy of the reading by evaluating the pulse plethysmographic waveform. Waveforms that are irregular or erratic may contain artifact caused by a loose, misapplied, or damaged pulse oximeter or by client movement (Option 4). After ensuring that the probe has been properly applied and positioned to provide an accurate reading, the nurse should perform a thorough physical assessment and intervene as appropriate. (Option 1) Auscultation of the lungs would be the next appropriate action to perform if pulse oximetry readings are deemed accurate. (Options 2 and 3) If the reading is deemed accurate after further assessment, the nurse should assist the client into the high Fowler position and encourage slow, deep breaths to promote ventilation. The nurse should increase the oxygen flow rate and notify the health care provider if the client's oxygen saturation does not improve with nursing interventions. Educational objective: When a low oxygen saturation with apparent artifact in the pulse plethysmographic waveform is observed, the nurse should discern the accuracy of the reading to prevent unnecessary treatment. If the pulse oximeter reading is accurate, the nurse should perform a thorough physical assessment and intervene as appropriate.

The nurse is teaching a class on nutrition and feeding practices for young children. What should the nurse recommend as the best snack for a toddler? 1. ½ cup orange juice 2. Dry, sweetened cereal 3. Raw carrot sticks 4. Slices of cheese

4 When choosing foods for a toddler (age 1-3 years), parents should consider the following factors: - Safety: Small, hard, sticky, or slippery foods (eg, hot dogs, whole grapes, nuts, raw carrot sticks, popcorn, peanut butter, hard candy, fruit snacks) pose a choking risk and should not be offered. - Nutrient density: Foods should contain valuable nutrients (eg, protein, vitamins) rather than just "empty calories" (eg, sugars). - Potential for foodborne illness: Children are at a higher risk for developing food-related infections, especially if given raw, unpasteurized foods (eg, partially cooked eggs, raw fish, raw bean sprouts). Healthy snacks for a toddler include pieces of cheese, whole-wheat crackers, banana slices, yogurt, cooked vegetables, and cottage cheese with thinly sliced fruit (Option 4). (Option 1) Although orange juice is a source of vitamin C, it contains a large amount of sugar and lacks fiber. Toddlers should have no more than 4-6 oz of 100% fruit juice per day. (Option 2) Sweetened cereals, especially those marketed toward children, can be high in sugar and low in nutrients. (Option 3) Raw carrot sticks are hard and pose a choking risk. Parents should serve carrots and other hard vegetables grated or cooked. Educational objective: Food for young children should contain valuable nutrients and pose little risk of choking or foodborne infection. An example of a healthy snack for a toddler is a slice of cheese.


Related study sets

Ch. 29 Development and Inheritance

View Set

Internet-Based Research - SBE (ID 510)

View Set

Chapter 4: How Cells Obtain Energy

View Set

Cultural Anthropology - Applied Perspective: Chapter 5: Methods in Cultural and Applied Cultural Anthropology

View Set

Chapter Four: Dispute Resolution

View Set

prepU: nose, ears, eyes, mouth, and throat

View Set

aPHR certification: compensation and benefits

View Set

_1_ Premature Death Loss Exposures

View Set

Biology B v.3 : 4. HUMAN ANATOMY AND PHYSIOLOGY

View Set